You are on page 1of 232

SCHOOL OF DISTANCE EDUCATION

UNIVERSITY OF KERALA

MASTER OF BUSINESS ADMINISTRATION


SECOND SEMESTER

(2018-19 ADMISSION ONWARDS)

MGT 202

OPERATIONS RESEARCH

PREPARED BY: EDITED BY:

VENUGOPAL.S DR. B.SHAJI


DEPUTY GENERAL MANAGER COORDINATOR
HLL LIFE CARE LTD (MBA) SDE
TRIVANDRUM

1
Syllabus for operation Research MBA

SDE S2 (2018 batch)

Unit I: Introduction to operation Research- objectives & business applications of


operations Research, Models

Introduction Linear programming Problem (LPP) formulation graphical method,


Simplex Method, Dual LPP, basic concepts of sensitivity analysis.

Unit II : Transportation models. North west control method (NWC), Least cost
method (LCM), Vogels approximation method, Hungarian Method, Minimisation of
cost

Unit III : Network models, critical path method (CPM), Programme evaluation
review technique (PERT) Crashing resource smoothing – Resource leveling

Unit IV : Queuing Theory M/M/I (System with single server) M/M/2 (system with
two servers). Monte carlosimulatation, Application in Queing + Inventory

Unit V : Decision theory : Decision making under risk and uncertainty,Game theory,
Decision tree ,ORsoftwares

References

1. Operation research Introduction Taha HA


2. Operation Research Theor&y Applications
3. Introduction to operations Research Hiller F.S&Gerala
4. Operations Research Principles practice-KantiSwarup& Gupta man mohan
5. Optimisation Method& OR system Analysis – Mital KV& Mohan C

2
Module 1

Operations research
Operations Research is the systematic and method oriented study of the basic structure,
functions and relationships in an organization.
Operation research is the application of scientific methods, techniques and tools to
problems involving the operation of a system so as to provide those in control of the
system with optimum solution to the problem
Operations research is scientific methods of providing executive department with a
quantitative basis for decision regarding the operations under the control.
Operation Research is an applied decision theory which involves a team approach of
experts from various disciplinessuch as scientists,Engineers ,Business executives
,psychologistsetc

Features (characteristics) of OR
1. System Orientation – All possible interactions in activities of organization
2. Interdisciplinary approach – Team of scientific from different faculties
3. Scientific approach :Uses scientific methods to solve complex problem.
4. Decision making : Management to take better decision
5. Optimization of objective: OR attempts to fund the basis and optimal solution to
problem using OR techniques. It tries to optimize a well defined function subject to
given constraints.
6. Mathematical Models and quantitative solution
OR uses models built by quantitative measurements of variables concerning a given
problem and drives a quantitative solution from the model.
7. Use of computer : OR often require a computer to solve the complex mathematical
model

Functions of operational Research


1. OR provides a tool for scientific analysis
2. OR provides solution for business problem
3. OR provides proper development of resource
4. OR assists in choosing an optimum strategy
5. OR facilities the process of decision making
6. OR helps in minimizing waiting servicing cost

3
Phases of operation Research
1. Formulating the problem: identifying defining a specifying the components of
model
2. Constructing the model : Objective function & constraints
3. Deriving the solution – Solution to the model
Analytical method, numerical method and simulation methods are used
4. Testing the validity : A model is said to be valid, if it can gave a reliable prediction
of system performance.
5. Controlling the solution : Control over solution of proper feedback of information
variables which deviated significantly
6. Implementing the results – Examination of solution to be adopted

Limitation of operation research

1. OR models do not consider qualitative factors or emotional factors


2. Mathematical models are applicable to only specific category of employees
3. It is a specialist job
4. Implementation of decisions is a delicate job
5. Incorporating the mode is a costly affair when basic data is subjected to frequent
changes

Role of operation research in decision making


Decision making is an important responsibility of today’s management and this can be
achieved through scientific approach to the problem. OR provides a quantitative basis for
decision making since it involves scientific method. By employing a systematic study of
the problem, building mathematical model representing with the model and predicting
future operation, OR helps business management in taking wise decision

OR techniques are useful in decision making in the following areas

1. Suggesting the best locations for factories &Warehouses


2. Suggesting most economic means of transportation
3. Identification of most profitable type or use of advertisement campaigns

OR techniques
1. Allocation model: Allocation of resources such that effectiveness is optimized.
Allocation problem can be solved by linear &non linearprogramming Technique
.Assignment models and Transportation models are special cases.
2. Sequencing : Placing items in a certain sequence
3. Waiting or queuing theory : Models involving waiting for services
4. Inventory models : These are models with regard to holding or storing resources aim
at optimum inventory level

4
5. Competitive strategy models (Game theory) : When two or more people competing
for a certain resources,. Game model is used to determine optimum strategy in a
competitive situation
6. Decision Theory: Concern with making sound decisions under conditions of
certainty, risks &uncertainty.
7. Network analysis :Involve determination of an optimum sequence of performing
certain operations concerning some job in order to minimize overall time or cost. Eg.
PERT, CPM, Gantt Charts
8. Simulation : Technique of testing a model which represent real life situation
9. Search models : The models concern itself with search problem
10. Replacement theory : These are models concerned with the situation that arise when
some items need replacement because the same may deteoriatewith time or break
down completely.

Scope of operations (Applications of OR)


OR has a wide scope in every day lines

OR has a very great impact on Economics, management Engineering and other social and
behavioral service

1. In defenceoperation :Techniques of OR does much help to select the optimum


strategy. Many of the defense operation involves scientific decision making. The
methods via work techniques for shortest path problems scheduling, algorithm for
vehicle routing and allocation techniques for shipping food grain can be applied in
defence establishments
2. Industrial application

Industrial management includes managing force funded areas such as production,


marketing, personnel and finance and other support services

Techniques used in production function are as follows

- Linear programming for aggregate planning


- Shop floor production scheduling
- Network based techniques for line balancing project management
- Inventory control techniques for planning and procuring raw materials
- Replacement analysis for equipment replacement decision
- Queuing theory for designing in process buffer stock

Marketing function

1. Linear programming for product mix problem


2. Game theory for order bidding decision
3. Distance network related techniques for shipping finished goods
4. HR function
1. Manpower planning

5
2. Queing theory for determining the size of maintenance crews
3. Scheduling techniques for man power scheduling
5. Finance function
1. Integra programming for capital for budgeting
2. Portfolio analysis
3. Investment analysis
6. Purchasing
1. Material transfer
2. Optimal buying
3. Optimal reordering
7. Research and development
1. Control of R&D projects
2. Product introduction planning

III Business Applications

1. Accounting : cash flow accounting, credit policy analysis ect


2. Construction : Allocation of resources to projects, determination of proper work force
Deployment of workforce
Project scheduling
3. Faculties planning : Factory size,ware, house location
4. Design a operate public system such as Govt. departments, postal system, bank,
highways, railways, airways, hospitals & educations, institutions

Models of OR

A model is an abstraction of reality. Models play a very important role in OR


Models in OR may be defined as an idealized representation of a real life situation
Eg. Road map a city to trace the shortest route from a given source to destination
Three dimensional view of a factory to plan the material movement in shop floor

Properties of a good model


1. It should be simple
2. It should be capable of adjustments with new formulation without having any
significant change in its frame
3. It should contain very few variables
4. A model should not take much time in its consultation

Advantages of a model

1. It describes problem more concisely


2. It provides some logical and systematic approach to the problem
3. It indicates limitation and scope of problem
4. It tends to make the overall structure of the problem more comprehensible

6
5. It enables the use of high powered mathematical techniques to analyse the
problem
6. It helps in finding avenues for new research and improvement in the system
Disadvantages of model

1. Models are only an attempt to understand an operation and should never be


considered as absolute in any sense.
2. The validity of any model can only be verified by carrying on experiment relevant
data characteristics

Types of models

Models are classified based on the following aspect

Model by function Models by Models by nature Models by extent


structure of environment of generality

Descriptive Normative or
models optimization
model

Iconic or Mathematical or Analogue


symbolic models models
Physical models

Deterministic Probabilistic or
model stochastic
models

Static models Heuristic model

Descriptive model: Describe and predict factsandrelationshipamong various activities of the


problem. These models do not have an objective function as a part of model to evaluate

7
decision alternatives. In this model, it is possible to get information as to how one or more
factors change as a result of changes in other factors.

Normative or optimization models


They are prescriptive in nature and develop objective decision rule for optimum solution

Models by structure
Iconic or physical models : They are pictorial representation of real system and have the
appearance of the real thing. Iconic models are obtained by enlarging or reducing the size of
the system. In other words they are images.
Eg .photographs, drawing model air planes, globes, map etc.
These models are easy to observe and difficult to manipulation
a. . Symbolic models
These model employ a set of mathematical symbol to represent the components of
real system. This symbolic models are some kind of mathematical equation or
inequalities reflecting the structure of system they represent.Eg : Inventory models,
Queuing models, allocation models ,sequencing models etc.
These models usually yield more accurate results under manipulation
b. Analogue models : In analogue models, one set of properties is used to represent
another set of properties. After the problem is solved, the solution is reinterpreted iin
terms of original system. Analogue models are easy to manipulate than iconic models
Eg.Control linear on a map are analogues of elevation as they represent the rise and
fall of heights
Deterministic models
In deterministic models, everything is defined and the results are certain
Eg. EOQ methods, economic lot size
Probabilistic model (Stochastic models)
In this model there is risk and uncertainty and thereforeinput and output variables
assume probability distribution. In this model at least one parameter or decision
variable is a random variable.
Eg. Game theory, if the strategy is not pure, we apply probability model
Static Model: In this model, time factor is not considered.
Eg. Inventory problem of determining EOQ
Heuristic Model : In this model, are used to explore alternative strategies which have
been over looked previously.

8
Linear programming (LPP)
Linear programming is one of the important operation research techniques used in business
and industry.
Linear programming is a mathematical technique which involves allocation of limited
resources in an optimal manner, on the basis of a given criterion of optimality
Linear programming deals with the optimization of a function of variables known as
objective function to a set of linear equalities/ inequalities. Optimization may be maximize or
minimize.

Uses of linear programming


1. Linear programming technique is used to achieve the best allocation of available
resource which includes man hours, money, machine hours, raw materials etc.
2. Linear programming is the powerful quantitative technique which can be used to
solve problems concerning with maximizing profit and minimizing cost by allocating
the resource in such a way that it optimize the objective.

Advantages of Linear programming


1. It provides an insight and perspective into problem environment related with a multi
dimension phenomenon
2. It makes a scientific and mathematical analysis of the problem situation
3. It gives an opportunity to the design maker to formulate his strategies consistent with
constraints and objectives
4. By using LP, the decision makes make sure that he is considering best solution
Limitation of LPP
1. LPP treats all relationship as linear which is not hold good in many real life situations.
Fractional values appear in LPP which is not accepted
2. All the parameters in the linear programming model are assumed to be known
constants, where as in reality they are frequently neither known nor constants
3. Many problem are complex since the number of variables and constraints are quite a
large number and cannot be used in all business problem

Basic assumptions of LPP

1. Proportionality : There must be proportionally between objectives and constraints

2. Additivity: Sum of resources used by different activities must be equal to the total
quantity of resources used by eachactivity for all the resources individually or
collectively

9
3. Divisibility : The solution need not be whole number

4. Certainity : Coefficients in the objective function and constraints are completely known
and do not change during the period under study.

5. Finiteness : Activity and constraints are of finite numbers

6. Optimality : The solution to a problem is to be optimum (maximum or minimum)

Formulation of LPP

Steps involved in LPP are as follows

1.Identify the objective as maximization or minimization


2. Mention the objective quantitatively and express it as a linear function of variables known
as objective function
3 .Identify the constraints which are conditions stipulated in the problem. Constraints are
expressed in the form of linear inequalities or equation
Standard form of a mathematical model of linear programming problem
Optimise Z= C1x1+ C2x2+ ----------+Cnxn
Subject to a11x1+ a12x2+-----------+ a1nxn <-(>-)b1
a21x1+ a22x2+ ----------+a2nxn<-(>-) b2
am1x1+am2x2+-----------+amnxn<-(>-)bn
where x1,x2------xn>0
The above conditions are referred as structural constraints
Non Negative constraints where x1,x2,------xn>0
Applications of linear programming in management

1.Determining product mix


2.Product smoothening
3.Media selection
4.Travelling sales man Problem
5.Transportation problem
6.Assignment problem
7. Communication industry
8.Staffing problem
Problem 1
A manufacturer of furniture markets two products, chairs and tables . Processing of these
products is done on two machines A & B. A chairs requires 2 hours on machine A and 6
time on machine B.
A table require 5 hours on machine A and no time on machine B.

10
16 hours of time per day available on machine A and 30 hours on machine B. Profit gained
by the manufactures from a chair isRs. 1 and from a table is Rs. 5 respectively. Formulate
the problem into a LPP in maximize the total profit.
Solution
Let x1 be the number of chairs and x2 be the no of tables produced.
Profit from chair = 1 x x1
Profit from table = 5 x x2
Total profit Z = x1 + 5x2
Constraints
Machine B
Time required for chairs = 2 x x1 = 2x1
Time required for tables = 5 x x2 = 5x2
Total time required = 2x1+ 5x2
Available time = 16 hours
2x1 + 5x2 ≤ 16
Machine B
Time required for chairs = 6 x x1 = 6x1
Time required for tables = 0 x x2 = 0
Total time required = 6x1 + 0
Available time = 30 hours
6x1 + 0 ≤ 30
LPP :Maximize z = x1 + 5x2
Subject to 2x1 + 5x2 ≤ 16
6x1 ≤ 30
x1>0 x2 ≥0
2. A manufacturer of medicines is preparing a production plan on medicines A & B. There
are sufficient ingredients available to make 20,000 bottles of A and 40,000 bottles of B. But
there are only 45000 bottles into which either of the medicines can be put. Further more, it
takes 3 hours to prepare enough material to fill 1000 bottles of A, it takes one hour to
prepare enough material to fill 1000 bottles of B and there are 66 hours available for the
operation. The profit is Rs. 8 per bottles for A and Rs. 17 per bottle for B.
Formulate the problem as LPP
Solution
Let x1 be the no of bottles of medicine A to be produced
Let x2 be the no of bottles of medicine B to be produced

Objective function
Maximize Z = 8x1 + 17x2
Constraints ( 3 types)
1. Availability of bottles
2. Availability of hours
3. Availability of ingredients

First constraints

11
X1 + x2 ≤ 45000 --------(1)

Second constraint

+ ≤ 66

Ie.3x1 + x2 ≤ 66,000 --------------(2)


Third constraint
X1≤ 20,000 x2 ≤ 40,000
Programme can be formulated as
Maximize z = 8x1 + 7x2
Subject to
3x1 + x2 ≤ 66000
X1 + x2 ≤ 45000
X1 ≤ 20,000
X2 ≤ 40000 x1 ≥0 x2 ≥ 0
Problem 3.
A person requires 10,12 and 12 units chemicals A, B respectively for his garden. A liquid
product contains 5,2, and 1 units of A, B, C respectively per jar. A dry product contain 1,2
and 4 units of A, B, C per carton. If the liquid product sells for Rs. 3/ per jar and the day
product sells for Rs. 2 /- per carton, how many of each should be purchased in order to
minimize the cost.

Solution
Decision variable
Let x1 and x2 be the number of units of liquids and dry products
Objective function
Minimise Z = 3x1 + 2x2
Constraints for 3 chemicals
5x1 + x2 ≥ 10
2x1 + 2x2 ≥ 12
X1 + 4x2 ≥ 12
X1 x2 ≥0
Problem 4

A company manufactures two products A and B. These products are processed in the same
machine. It takes 10 minutes to process one unit of product A and 2 minutes for each unit of
product B and the machine operates for maximum 35 hours in a week. Product A requires 1
kg and 0.5 kg of raw material per unit the supply of which is 600 kg per week. Market
constraint on product B is known to be 800 unit every week. Product A costs Rs. 5 per unit
sold at Rs. 10. Product B costs Rs. 6 per unit and can be sold in the market at a unit price of
Rs. 8.

12
Determine the number of units of A and B per week to maximise the profit

Solution

Let z1 and x2 be the number of products of A&B

Profit in one unit of product A = 10-5 = 5

Profit of one unit of product B = 8-6 = 2

Objective function

Max Z = 5 x1 + 2x2

Constraints 10x1+ 2x2 ≤ 35 x 60)

Raw materials constraint x1 + 0.5 x2 ≤ 600

Market demand on product B x2 >- 800

LPP

Max Z = 5x1+ 2x2

Subject to 10x1 + 2x2 ≤ 2100

X1 + 0.5 x2 ≤ 600

X2 >800 x1 x2 ≥ 0

Diet problem
Problem 5
Objective of diet problem is to ascertain the quantities of certain foods that meet the
nutritional requirement at minimum cost. The food articles considered are milk, beef
and eggs and the nutritional requirements are vitamins A, B & C. The number of
milligrams of each of these vitamins contained within a unit of each food is given
below.

Solution.

Minimum daily
Vitamin Gallon of milk Pound of beef Dozen of egg
required
A 1 1 10 I mg
B 100 10 10 50 mg
C 10 100 10 50mg
Cost Rs 10 1.10 0.50

13
Formulate a LPP for the problem

Solution

Let x1 be the gallons of milk x2 be the pound of beef and x3 be the dozen of egg to be
included in the daily diet

Minimise

Z = 1x1 + 1.10x2 + 0.5 x3

Subject to

x1 + x2 + 10x3 ≥ 1 (Vitamin A)

100x1 + 10x2 + 10x3 ≥ 50 (Vitamin B)

10x1 + 100x2 + 10x3 ≥ 10 (Vitamin C)

x1 x2 x3 ≥ 0

Activity Questions

6 A company manufactures two types of products P1 & P2. Each product uses lathe and
milling machine. The processing time per unit of P1 on the lathe is 3 hours and on the
milling machine is 4 hours. The processing time per unit of P2 on the lathe is 10 hours and
on the milling machine is 4 hours. The maximum number of hours available per week on
the lathe and milling machine are 60 hours and 40 hours respectively. Also the profit per
unit of selling p1 and p2 are Rs6 and Rs8 respectively.

Formulate a linear programming model to determine the production volume of each of the
products such that total profit is maximized.

(Maximise Z = 6x1 + 8x2


Subject to 3x1 + 10x2 ≤ 60
4x1 + 4x2 ≤40
x1 ≥ 0, x2 ≥ 0
Problem 7
A textile company can use any or all the time different processes for weaving standard
whitepolyster fabric. Each of these production process has a weaving machine set up cost
and per sq meter processing cost. Cost and capacities of each of the these production
processes are shown in table

Weaving machine
Process number Processing cost / sqm ® Max daily capacity (sq)
Set up cost (Rs)
1 150 15 2000
2 240 10 3000
3 300 8 3500

14
The daily demand for cost for its white polyester fabric is 4000 sqm. The company
production manager wants to make a decision concerning which combination of
production processes is to be utilized to meet the daily demand for cost and what
production level of each selected production process to be operated to minimize the total
product costs

Develop a LP model to assist the production manager

(minimize Z = 15x1+ 8x3 + 10x2 + 150 y1 + 240 y2 + 300 y2


Subject to constraints x1 + x2 + x3 = 400
X1 – 2000y1 ≤ 0
X2 -3000 y3 ≤ 0 X1,X2<X3>0, Y1,Y2,Y3>0
X3 – 3500y3 ≤ 0
Problem8A company manufactures two products A and B. Each unit of B takes twice as
long as one unit of A and if the company were to produce only A it would have time to
produce 2000 units per day. The availability of raw material is sufficient to produce 1500
units per day of both A and B combined. Product B requiring a special ingredient only
Rs. 600 units can be made per day. If A fetches a profit ofRs. 2 per unit and B profit of
Rs. 4 per unit. Find the optimum product mix
Ans :Max Z = 2x1+ 4x2
Subject to constraints
x1 + 2x2 ≤ 2000
X1+ x2 ≤ 1500
X2 <600 ,X1,X2>-0
Problem 9An electrical appliance company produces two products refrigerator and
ranges production takes place in two separate departments I and II. Refrigerators are
produced in Dept I and ranges in Dep II. The company two product are sold on weekly
basis. The weekly products cannot exceed 25 refrigerators and 35 ranges. The company
regularly employs a total of 60 workers in two departments. A refrigerator require 2 man
work labour while a range requires 1 man week labour. A refrigerator contributes a profit
of Rs. 60 and range contributes a profit of Rs. 40. How many units of refrigerator and
ranges should the company produces to realize the max profit. Formulate the above as
LPP.
Solution
Maximize Z=60x1 + 4x2
Subject to x1 ≤ 25 x2 ≤35
2x1 + x2 ≤ 60
x1, x2 >0
Problem 10

A company produces two types of cow boy hats. Each hat of the first type requires twice
as much alabour time as the second type. If all the hats are of the second type only, the
company can produces a total of 500 hats a day. The market limits daily sales of first type
and second types to 150&200 hats a day. Assuming that profit per hat areRs. 8 for type
I and Rs. 5 for type 2. Formulate the problem as a LP model in order to maximize profit.

15
Solution

Maximise z = 8x1 + 5x2

Subject to constraints 2x1 + x2 ≤ 500


X1 ≤ 150, x2 ≤ 250
X1 x2 ≥ 0
Method of solving LPP graphical method
A LPP can be solved by two methods

1. Graphical method – Geometrical method


2. Simplex method – Algebric function

A linear programming comprises of

1. A objective function
2. Set of constraints
3. Non negative restrictions

In a graphical method, the problem is solved by the method of graph drawn in a two
dimensional plane. Since only two variables can be represented in a two dimensional
plane, the graphical solution can be used only when the objective function has just two
variables x1 and x2

When the objective function has three or more variables, graphical method cannot be
used to solve the problem

Steps for solving LPP by graphical method

1. Formulate the problem into a LPP


2. Each inequality in the constraint may be written as equality
3. Draw straight lines corresponding to the equality to the equation as obtained in step 2.
So there will be as many straight line as there are equations.
4. Identify the feasible region. Feasible region is the area which satisfies all the
constraints simultaneously.
5. The vertices of the feasible region are to be located and these ordinates are to be
measured

Solution is the coordinate of vertex which optimizes the objective function and
corresponding values of the objective function

Problem 1Solve graphically the following LPP

Max z = 50x1 + 60x2


Subject to 2x1 + 3x2 ≤1500
3x1 + 2x2 ≤1500
X1 ≤ 400
X2 ≤400

16
X1 x2 ≥0
Solution
2x1 + 3x2 = 1500 Put x1 = 0 , x2 = 500
Put x2 = 0, x2 = 750
Points are (0,500)(750,0)
3x1 + 2x2 = 1500 Put x1 = 0 , x2 = 750
Put x2 = 0, x2 = 500
Points are (0,750), (500,0)
3x1 + 2x2 = 1500 Put x1 = 0 , x2 = 750
Put x2 = 0, x2 = 500
3x1+ 2x2 = 1500 Put x1 = 0 , x2 = 750
Put x2 = 0, x2 = 500
Points are (0,750) (500,0)

Feasible region lies to the left of three quadrant lines. The feasible region is marked as the
shaded region.Optimal
region solution lies at the corner points of the boundary of the feasible
solution
1. M1 (0, 400) z = 50x1 + 60x2
Z = (50 x 0) + 60 x 400 = 24000
2. M2 (150,400) Z = 50 x 1580 + 60 x 400 = 31500
3. M3 (300, 300) Z = 50 x 300 + 60 x 300 = 33000
4. M4 (400, 150) Z = 50 x 400 + 60 x 0 = 20000

Maximum value of the objective function is

Z = 33000 when x1 = 300, x2 = 300


Problem 2
Solve the following LPP
Maximise z = 3x1 + 4x2
Subject to 5x1 + 4x2 ≤200
3x1 + 5x2 ≤ 150
5x1 + 4x2 ≥ 100

17
8x1 + 4x2 ≥80
X1, x2 ≥ 0
Solution
5x1 +4x2 = 200put
200 x1=0 x2= 50
Put X2 = 0 x1= 40
Points are (0,50) (40,0)
3x1 + 5x2 = 150 Put x1 =0 x2 = 30
Put X2=0, x1 = 50
Points are (0, 30) (50,0)
5x1 + 4x2 = 100 put x1 = 0, x2 = 25
Put X2 = 0 x1 = 20
Points are (0, 25),
25 (20, 0)
8x1 + 4x2 = 80 put x1 = 0 x2 = 20
Put X2 = 0, x1 = 10
Points are (0,20) (10,0)

Critical area or feasible region shown as darked


0 (20, 0) Z = 3x 2 + 0 = 60
A (40, 0) Z = 3 x 40 + 0 = 120
A (308, 11.5) Z = 3 x 30.8 + 4 x 11.5

= 138.4

C (0, 30) Z = 3 x 0 + 4 x 30 = 120

D(0, 25) Z = 3 x 0 + 4 x 25 = 100

Maximum value of Z = 138.4 when x1 = 30.8, x2 = 11.5

Problem 3 Maximize Z = x – 3y subject to the constraints(UNI


(UNI FEB 2015)
5x + y ≤ 300

18
X – 2y ≤ 200
2x + y ≥ 100
Y ≥ 200 x, y > 0
Solution : The equation can be converted to inequalities
X + y = 300
X – 2y = 200
X + 2y = 200
Y = 200
First constraint x + y = 300
Put x =0, y= 300
Put y=0, x= 300
Points are (0, 300) (300, 0)
Second constraint x – 2y = 200
Put x = 0, y= -100
Put y = 0, x= 200
Points are (0,-100)(200,0)
Third constraint
2x+y=100
Put x=0 y=100
Put y=0, x=50
The points are (0,100)(50,0)

19
The critical points are M1(0,300)

2qZ= x-3y=
3y= 0-300x3= -900

M2 (0, 100)

Z= x- 3y, 0 – 3 x 100 = -300

M3 = (50,0)
Z = 50 – 3 x 0 = 50
M4 = (75,- 75)
Z = x- 3y 75 – 3 x- 75 = 300
M5 = (200, 0)
Z = x- 3y = 200-
200 3x 0 = 200
M6 = (265, 35)
Z = 265-33 x 35 = 265 – 105 = 160
Maximum value of z at the point M4 (75, 100)
Where z = 300
Maximum value of objective function = 300
Minimisation case problems

Problem 4
Solve graphically the LP problem
Minimize z = 1000 x1 + 800 x2
Subject to 6x1 + 2x2 ≥ 12
2x1 + 2x2 ≥ 8
4x1 + 12x2 ≥ 24

20
x1 ≥
≥0, x2 ≥ 0

Solution

First constraint : 6 x1 + 2x2 = 12

If x1 = 0, x2 = 6

If x2 = 0, x1= 2

The points are (0, 6) (2,0)

Second constraint 2x1 + 2x2 = 8

If x1 = 0, x2 = 4

If x2 = 0, x1 = 4

Points are (0,4) (4,0)

Third constraint 4 x1 + 12x2 = 24

If x1 = 0, x2 = 2

If x2 = 0, x1 = 6

Points are (0,2) (6,0)

Points are M1, M2 , M3 , M4

21
M1 (6,0) Z = 100x1 + 800 x2

= 100 x 6 + 0 = 6000

M2 (3,1) Z = 1000 x 3 + 800 x 1

= 3000 + 800 = 3800

M3 (1,3) Z = 1000 x 1 + 800 x 3

= 1000 + 2400 = 3400

M4 (0,6) Z = 1000 x 0 + 800 x 6 = 4800

Minimum value of Z at M3 where x1 = 1X2 = 3

, Minimum value of Z = 3400

Problem 5

A Small scale manufacturer has production facilities for producing two different products.
Each of the product requires three different operations. Grinding , assembling and testing .
product I requires 15, 20 and 10 minutes to grind, assemble and test respectively where as
product 2 requires 7.5 , 40 and 45 minutes for grinding, assembly and testing. The production
run calls for at least 7.5 houses of grinding time, at least 20 hours of assembly time and at
least 15 hours of testing time. If product I costs Rs. 60 and product II costs Rs. 90 to
manufacture, determines the number of each product the firm should produceinorder to
minimize the cost of operation?

Solution : let x1 and x2 be the number of units of products I and II produced

Minimize Z = 60x1 + 90 x2
Subject to 15x1 + 7.5x2 ≥ 7.5 x 60
20x1 + 40 x2 ≥ 20 x 60
10x1 + 45x2 ≥ 15 x 60
X1 > 0 x2 ≥ 0
Converting all the constraints as equations
15x1 + 7.5 x2 = 450
20x1 + 40 x2 = 1200
10x1 + 45x2 = 900
X1 = 0, x2 = 0
Ist constraint 15x1 + 7.5x2 = 450
X1= 0, x2 = 60
X2 = 0, x1= 30
Points are (0,60),(30,0)
2nd constraint
20x1 + 40x2 = 1200
Put X1 =0, x2 = 30

22
Put X2 = 0, x1= 60
Points are (0, 30) (60,0)
(60,
rd
3 constraint

10x1+ 45 x2 = 900
Put X1 = 0, x2 = 20
Put X2=0, X1=90
Points are (0,20),(90,0)

Feasible region is ABCDE


Points X1 X2 Z = 60x1 + 90x2
A 0 60 5400
B 20 20 3000
C 36 12 3240
D 90 0 5400
E - - -
Z is minimum at B

20 units of products I and 20 units of product II may be manufactured so that cost is


minimum of Rs. 3000/-
3000

Activity Questions

23
Problem6 Solve the LPP graphically

Max Z = 5x + 8 y

Subject to 3x + 2y ≤ 36
X+ 2y ≤ 20
3x + 4y ≤ 42
X,Y ≥0(AnsMaxZ=82 when X=2,Y=9)
Problem 7
Egg contains 6 units of vitamin A per gm and 7 units of vitamin B per gm and the cost 12
paise per gm. Milk contains 8 units of vitamin A per gm and 12 units of vitamin B per gm
and costs 20 paise per gm. The daily minimum requirement of Vitamin A and vitamin B are
100 units and 120 units respectively. Find the optimal product mix.
Solution
Min Z = 205 x1 = 15, x2 = 1.25
Problem 8
Minimize Z= 25x1 + 22x2
Subject to constraints X1 + x2 ≥ 10
300 x1 + 400 x2 ≥ 3400
80x1 + 50x2 ≥ 680
X1 , x2 ≥ 0

Problem 9
Solve graphically
Maz Z = x1- 2x2
Subject to contraints– x1 + x2 ≤ 1
3x1 + 2x2 ≥ 12
0≤ x1 ≤ 5
2 ≤ x2 ≤ 4
Ans z = 1 where x1 = 5 x2 = 2
Problem 10
Old hens can be bought for Rs. 2 each but young ones cost Rs. 5 each. The old hens lay 3
eggs per week and the young ones 5 eggs per week each egg being worth 30 paise. A hen
costs Rs. 1 per week to feed., If I have only Rs. 80 spend for hens, how many of each
kind should I buy assuming that I cannot house more than 20 hens.
Some special cases of solutions
1.Infeasible : When no solution is obtained for a LPP satisfying all the constraints, there
exists no feasible solution that is if the feasible region of LPP is empty the solution is
infeasible
2. Unbounded : If in an LPP the solution of a variable can be made infinitely large
without violating the constraints, the solution is unbounded. That is feasible region is
unbounded for such a LPP

24
3. Alternative: If there are more than one optimum solution for the decision variable ( the
value of the objective function remaining the same) we say there are alternative or
multiple solution to LPP.
Problem 11
SDE UNI Feb 2016(5 Marks)
A firm engaged in producing two models A, B performs three operations. Painting,
assembling and testing. The relevant data are as follows (5 marks)

Unit sale Hours required for each unit Testing


Model
Price Assembly Painting Hrs
A Rs. 50 1.0 0.2 0
B Rs. 80 1.5 0.2 0.1

Total number of hours available


Assembly 600 painting 100 testing 30.
Determine weekly production schedule to maximize the profit.
Solution
Let x1 be the no of unit of model Aproduced
X2 be the no of units of model B produced
Constraints : Assembly, painting and testing
Maximize Z = 50x1 + 80x2
Subject to constraints X1+ 1.5X2<-600
0.2 x1 + 0.2x2 ≤ 100
0.1 x2 ≤ 30
X1, X 2 > 0
Solution graphically
Constraint I : x1 + 1.5 x2 = 600
If x1 = 0, x2 = 400
If x2 = 0 , x1 = 600
Points are (0, 400) (600, 0)
Constraints II0.2x1 + 0.2x2 = 100
If x1 = 0 , x2 = 500
If x2=0, x1 = 500
Points are (0,500) (500,0)
Constraints III 0.1 x2 = 30
X2 = 300
Point is (0,300)

25
Required area is O A B C D
Required points are O, A, B, C, D

Point Values Z = 50x1+80x2


O (0, 0) Z=0
A (0, 300) Z = 80*300=2400

B (150, 300) Z = 50*150+300*50 = 31500

C (150, 200) Z = 50*300+80*200 = 31000

D (500,0) Z = 50*500 = 2500

Value of Z is maximum at B where x1= 150


Maximum value of objective function Z = 31500

26
Problem 12

Solve the following LPP graphically(10 marks)


SDE UNI Nov 2013,Dec 2008
Max Z = 5x1 + 7x2
Subject to constraints x1 + x2 ≤ 4
3x1 + 8x2 ≤ 24
10x1 + 7x2 ≤35
X1, X2 ≥ 0
Constraint I x1 + x2 = 4
If x1=0 x2 = 4
If x2 = 0 x1 = 4
Points are (0,4) (4,0)
Constraint II : 3x1 + 8x2 = 24
If x1 = 0 x2 = 3
If x2 = 0 x1= 8
Points are (0,3) (8,0)
Constraint III : 10x1 +7x2 = 35
If x1= 0 x2 = 5

If x2= 0 x1 = 3.5Points are (0,5) (3.5, 0)

Required area is O A B C D

27
Point Value of objective function
Values
Values Z = 5x1+7x2
O (0,0) 0
A (0, 3) Z = 7*3 =21
B (1.5, 2.5) Z = 5*1.5+7 = *2.5 = 25
C (2.1, 1.8) Z = 5*2.1+7*1.8 = 23.1
D (3.5,0) Z = 5*3.5 = 18.5
Maximum value of objective function Z=25, When x1 = 1.5, x2 = 2.5 at point B

Simplex method
Simplex method is a linear programming technique in which we start with a certain
solution which is feasible. We improve the solution in a number of consecutive stages
until we arrive at an optimal solution.
For arriving at the solution LPP by this method, the constraints and objectives function
are presented in a table known as simplex table.
Main steps involved in simplex algorithm
1. Find an initial basic feasible solution of LPP
2. Test whether it is an optimal solution or not
3. If not optimal; improve the first trial basic feasible solution by a set of rules
4. Repeat steps 2 and 3 till an optimal solution is obtained

Feasible solution

A feasible solution to a LPP is the set of values of variables which satisfies all the
constraints and non negativerestrictions of the problem

Optimal solution

A feasible solution to a LPP is said to be optimum if it optimizes the objective function Z

Basic feasible solution

A feasible solution to a LPP in which the vector associated to non zero variables are
linearly dependent is known as basic feasible solution.

Steps involved on simplex algorithm

1. Formulate the problem into a LPP


2. Convert the constraints into equation by introducing non negative slack variables
If the constraintshas a ≤ sign, variables added to LH side of the constraints to convert
them into inequalities are called slack variables. For ≥ sign, subtract to the LH side
of the constraints are called surplus variables
Coefficient of slack / surplus variables in the objective function is zero.
3. Construct a simplex table
4. Conduct the test of optimality
This is done by computing the net evaluation
∆ j = Zj – Cj

28
If all ∆js are positive solution is optimal.If any of the ∆j s are negative, continue from
steps 5 onwards.
5. Select the entering variables which has the highest negative value called key column
.A column which has highest negative value in a maximization problem and calculate
the key row which is the row correspond to minimum ratio column in the table. These
values are calculated by dividing the values in the Bj column by the corresponding
values in the key column
The values in the ratio column are known s exit ratios or limiting ratio. The variables
with the smallest non negative ratio is the exit variable
6. Calculate the new values for the new table value corresponds top values in the key
rows are obtained by dividing each value in the key row by the key number
7. New values can be calculated in remaining rows are calculated as follows.
New row = (old row values to be changed ) - (Corresponding new value for key row)
(value corresponding to values in row to be changed)
Repeat steps 4,5,6 till optimality is reached
I. Problem 1

Solve by simplex method

Maximize Z = 5x1+ 3x2 subject to

X1 + x2 ≤2

5x1 + 2x2 ≤ 10

3x1+ 8x2 ≤12 x1, x2 ≥0

Introduce stack variables we have

X1 + x2 + S1 = 2

5x1+ 2x2 + S2 = 10

3x1+ 8x2 + S3 = 12 X1,X2>o

Simplex table

Value of Value of 0 Ratio


5 3 0 0
Cj basic basic
x1 x2 S2 S3
variable variable S1
0 S1 2 1 1 1 0 0 2/1 = 1
0 S2 10 5 2 0 1 0 10/5=2
0 S3 12 3 8 0 0 1 12/3=4

29
Zj-Cj -5 -3 0 0 0
All Zj-Cj are negative the solution is not optimal

X1 enters (x1 is the incoming varoables and S1 is the outgoing variables

New table

2= =2 =1 =2 =2 =0

0/0 = 0 0/1 = 0

Pivottedoperation

New row = (old row value to be changed ) – (Corresponding new value for key rows)
– value corresponds to key value to be changed)

10 = 10- (2 x 5) = 0

5 = 5 – (1 x 5) = 0

2 = 2- ( 1 x 5) = -3

0= 0-(1 x 5) = -5

1 = 1- (0 x 5) = 0

0 = 0 – (0 x 5) = 0

New values
12 = 12- (2 x 3) = 6
3 = 3- (1x 3) = 0
8 = 8- (1 x 3) = 5
0 = 0- (1 x 3) = - 3
0 = 0- (0x 3) = 0
1= 1- (0x3) = 1
0
Value of Value of
5 3 0 0
Cj basic basic
x1 x2 S1 S2 S3
variable variable

5 X1 2 1 1 1 0 0
0 S2 0 0 -3 -5 1 0
0 S3 6 0 5 -3 0 1

30
Zj 10 5 5 5 0 0
Zj-Cj 0 2 5 0 0

Since Zj- Cj is positive the solution is optimal


Hence x1 =2 , x2 =0
Value of objective function Z = 5 x 2 = 10
This is a case of degeneracy (Two tie variables occurs) and the values of one of
the basic variables is zero.
Problem 2
SDE Uniquestion Nov 2012 (10 marks)
Maximize Z = 4x1 + 10x2
Subject to constraints
2x1 + 4x2 ≤ 50
2x1 + 5x2 ≤100
2x1 + 3x2 ≤ 90
X1,X 2>0
Solution
Introduce stack variable
Max Z = 4x1 + 10x2 + OS1 + OS2 + 0S3
Subject to constraints
2x1+ x2 + s1 + s2 + 0s3 = 50
2x1 + 5x2 + 0s1+ 0s3 = 100
2x1 + 3x2 + 0s2 + 0s2+s3 = 90
Initial basic feasible solution (x1=0, x2=0)
S1 = 50, S2 = 100, S3 = 90
Simplex table
Value of
Basic 4 10 0 0 0 Ratio
Cj basic
Variable X1` X2 S1 S2 S3
variable

0 S1 50 2 4 1 0 0 = 50

0 S2 100 2 5 0 1 0 = 20

0 S3 90 2 3 0 0 1 = 30

31
Zj 0 0 0 0 0 0

Zj-Cj -4 -10 0 0 0

Smallest non negative value corresponds to row S2. Hence x2 enters and S2 leaves.

Basic 4 10 0 0 0
variable X1 X2 S2 S2 S3

S1= 0 30 1.6 4 1 1.8 0

X2 =
20 0.4 1 0 0.2 0
10

X3 =0 30 0.8 2 0 0.6 1

Zj 4 10 0 2 0

Zj-Cj 0 0 0 2 0

Revised key row=100/5 =20, 2/5= 0.4, 5/5=1,0/5=0

1/5=0.20, 0/5=0

Pivottedoperations

1st row IInd row


50 = 50- (20 x 1) = 30 90=90-(20x3) = 30
2 = 2 (0.4 x 1) =1.6 2= 2- (0.4 x 3) = 0.8
4 = 4-(1x0) =4 1= 1- (1x3) =- -2
1= 1- (0x1) = 1 0= 0-(0x3) = 0
2= 2-(0.2 x 1) = 1.8 0=0-(0.2 x 3) = - 0.6
0=0-(0x1)=0 1=1-(0x3) = 1

Since all dijsare positive or zero, the solution is optimum

Hence the optimum solution is x1= 0, X2 = 20

Objective function Z = 4 x 0 +10 x 200=200

Problem 3 BBA, JAN 2016

32
Solve the following using simplex method

Minimise Z = x1-3x2 + 2x3

Subject to constraints

3x1 – x2 + 3x3 ≤ 7

-2x1+ 4x2 ≤ 12

-4x1+ 3x2+ 8x3 ≤ 10

X1, X2, X3 ≥0

Solution

The minimization problem can be converted to a maximization problem as

Solution

Maximize Z = -x1+ 3x2 – 2x3

Subject to constraints

3x1- x2 + x3 ≤ 7

-2x1+4x2 ≤ 12

-4x1+ 3x2 + 8x3 ≤ 10

Introduce stack variable S1, S2, S3

Max Z = -x1+3x2 – 2x3 + 0S1+ 0S2 + S3

Subject to constraints

3x1- x2 + x3 + s1 = 17

-2x1+ 4x2 + S2 = 12

-4x1+ 3x2+ 8x3 + S3 = 10

X1, x2, x3 ≥0 S1, S2, S3 ≥ 0

Construction of a simplex table

Cj -1 3 -2 0 0 0
Value
Basic
basic X1 X2 X3 S1 S2 S3 Ratio
variable
variable

33
7/1
0 S1 7 3 -1 1 1 0 0
=7
12/4
0 S2 12 -2 4 0 0 1 0
=3
10/3
0 S3 10 -4 3 8 0 0 1
=3.33
Zj 0 0 0 0 0 0 0 3.3
Zj – Cj 1 -3 2 0 0 0

Solution is not optimal sinceone of the Zj – Cj is negative

X2 entries S2 leaves

New table

=3 =-1/2 4/4=1,0/4=0, 0/4=0 -

= 0.25, =0

Pivoted operation

7 = 7-(-1x3) = 10 10 = 10-(3x3) = 1
3 = 3-(-1x )= -4 = -4-(- x 3) = 0
-1 = -1-(1x-1) =0 8 = 8- (0 x 3) = 8
1=1-(0x -1)=1 0=0-(0x3) = 0
1= 1-(0x-1)=1 0= 0 –( x 3) = - -
1=1-(0x-1) =1 1 = 1- (0 x3) = 1
0=0-( x -1) =
0=0-(0 x -1) = 0

Revised simplex table

Cj -1 3 -2 0 0 0

5 1
X1 X2 X3 S1 S2 S3 Ratio

2 4
0 S1 10 0 1 1 0 10/5/2=4

34
3
−1 1
−1/2
2 4
= −6
3 X2 3 1 0 0 0

5 3 2
− − −
2 4 5
0 S3 1 0 8 0 1
3 3

2 4
Zj 3 0 0 0
Zj-Cj -1/2 0 2 0 3/4 0

Solution is not optimal since Zj- Cj is negative

.Highest non negative value is S1. Hence x1 enters

Here x1 enters S1 leaves

New table is obtained as follows

=4 =1 =0 = 2/5

= =0

Pivotedoperations

3 = 3-(4x ½ = 5) 1 = 1-(4x -5/2) = 11


-1/2 = -1/2 – (1 x -1/2 ) = 0 -5/2 = -5/2 – (1x -5/2)=0
1= 1-(0 x -1/2) = 1 0 = 0-(0x -5/2) = 0
0 = 0-(6/5 x -1/2) = 3/5 8= 8-(2/5 x -5/2) = 9
0= 0-(2/5x-1/2)= 1/5 0=0-(2/5x-5/2) =1
-3/4 = -3/4 – (1/10 x -5/2) =
¼ = ¼ - (1/10 x -½) = 3/10
-1/2
0 = 0-(0 x ½ ) = 0 1= 1-(0 x -5/2) = 1
Revised simplex table as follows

Cj -1 3 -2 0 0 0
Value of
Basic
basic X1 X2 X3 S1 S2 S3
variable
variable
5 X1 4 1 0 2/5 2/5 1/10 0
3 X2 5 0 1 3/5 1/5 3/10 0

35
0 S3 11 0 0 11 1 -1/2 1
Zj 5 3 33/5 13/5 7/5 0
Zj- Cj 6 0 23/5 13/5 7/5 0

Since all Zj-Cj are positive, solution is optimal

X1 =4, x2 = 5 x3 = 0

Max value of objective function

Z = -4 +3 x 5 + 2 x 0 = 11

Max value of objective function Z = +11 when

X1= 4, X2 = 5 ,X3 = 0

Simplex problem (Minimization problem)

In the case of ≥ inequalities in maximization/ minimization problem, the inequalities may be


greater than RHS. There may be surplus on LHS. To equalize the LHSand RHS, we have to
introduce surplus variables to the LHS and the surplus variables to be deducted from the
inequalities.

A minimization problem can be converted to maximization problem by change the sign of the
coefficients in the objective function.

Artificial variables

Since the coefficient of the surplus variables are negative this will not provide an initial base
feasible solution. This difficulty can be overcome by introducing another set of variables as
artificial variables with positive coefficients. These artificial variables have no value.
Hence the artificial variables are assigned very large positive coefficients or cost
represented by Min the objective function.

This method of solving LP problems by introducing artificial variables and assigning to


them very large positive or negative coefficients represented by M is termed Big M
method.Big M method is applicable for both minimization as well as maximization problem.

Problem 4MaximizeZ = x1+2x2 +x3

Subject to constraints 2x1 + x2 – x3 ≤ 2

-2x1+ x2 – 5x3 ≥ -6

4x1+ x2 + x3 ≤6

36
Solution x1, x2, x3 >0

Introducing slack and surplus variable

Max Z = x1+ 2x2 + x3 + x3 + 0S1 + 0s2 + 0S3 –MA1

Subject to constraint

2x1 +x2 –x3 +S1 + 0S2 + 0S3 = 2

-2x1 + x2 -5x3 + 0s1 –S2 + A1 = -6

4x1 + x2 + x3 + 0S1+0S2+S3 = 6

x1, x2, x3 >0

Putting x1, x2, x3 = 0

S1 = 2, A1 = 6, S3 = 6

Ratio
1 2 1 0 0 0 -M
Value of
Basic
basic X1 X2 X3 S1 S2 S3 A1
variable
variable
2/1=2
0 S1 2 2 1 -1 1 0 0 0
6/1=6
-M A1 -6 -2 1 -5 0 -1 0 1
6/1=6
0 S3 6 4 1 1 0 0 1 0

Zj 2M -M 5M 0 0 0 0

Zj-Cj 2M-1 -M-2 5M-1 0 0 0 0

Largest negative value x2 enters and S1 leaves.key row is Ist row

Ratio
1 2 1 0 0 0 -M
Value
Basic
Cj of basic X1 X2 X3 S1 S2 S3 A1
variable
variable
2/0=
2 X2 2 2 +1 -1 1 0 0 2
infinite
4/1=4
-M A1 -8 -4 0 -4 0 -1 0 1

37
9/0=
0 S3 4 2 0 2 -1 0 1 0
Infinite
Zj 4+4m 2 4m-2 2 M 0 -M

Zj-Cj 4m+3 0 4m-3 2 M 0 0


Key row

2/1=2 ,1/1=1 ,-1/1=-1 ,0/1=0 ,0 /1=0

Pivotedoperations

-6 = -6(2 x 1) = -4 6 = 6-(2 x 1) = 4
-6 = -6-(1x 1) = -4 4 =4-(2 x 1) = 2
1 = 1- (1 x1) = 0 1= 1- (-1 x 1) = 0
-5 = -5(-1 x 1 = 4) 1 = 1-(-1 x 1) = 2
0 = 0 – (0 x 1) = 0 0 = 0-(1 x 1) = -1
-1 = -1 – (0 x 0) =- 1 0= 0-(ox1)=0
0=0-(0x1)=0 1=1-(ox1)=1
1=1-(0x1)=1 0=0-(ox0)=0
Since alldijs are positive ,the solution is optimum

Optimum solution x1 = 0 ,x2 = 2 , x3= 0

Z = 0 + 2 x 2 + 0=4

Problem 5

Minimize Z = 3x1 + 8x2

Subject to constraints

X1 + x2 = 200

X1 ≤ 80 , x2 ≥ 60

X1 ≥ 0 x2 ≥ 0

Solution

Maximize Z = - 3x1 – 8x2 +0S1+ Os2 – MA1 – MA2

Subject to x1 + x2 + A1 = 200

X1 + S1 = 80

X2 – S2 + A2 = 60

38
This is a minimization case. Convert this to maximization problem by changing the sign of
the objective function

The objective function is

Z= -3 X1-8X2+0S1+0S2 –MA1-MA2

1 2 1 0 0 -M
Value of rATIO
Basic
Cj basic X1 X2 S1 S1 A1 A2
variable
variable
200/1=200
A1 -M 200 1 1 0 0 1 0
80/O=
X1 0 80 1 0 1 0 0 0
60/1=60
A2 -M 60 0 1 0 -1 0 1

Zj -M -2M 0 M -M M

Cj -3 -8 0 0 -M -M
Zj-Cj --M+3 - 0 M 0 0
2M+8

Highest negative ∆j is -2M + 8 .X2 is the incoming vector and A2 is the outgoing vector. Key
element is 1.

Revised simplex table

1 2 1 0 0 -M
Value of
Basic
Cj basic X1 X2 X3 S1 S2 A1
variable
variable
A1 -M 140 1 0 0 1 1

S1 0 80 1 0 1 0 0

X2 -8 60 0 1 0 -1 0

Zj -M -8 0 -M+8 -M

Cj -3 -8 0 0 -M
Zj-Cj --M+3 0 0 -M+8 0

39
Highest negative ∆j is –M+3. So X1 is the incoming vector. Minimum ratio is 80. So
outgoing vector is S1. Key element is 1

B CB XB X1 X2 S1 S2 A1 Ratio

A1 -M 60 0 0 -1 1 1 60

X1 -3 80 1 0 1 0 0 α

X2 -8 60 0 1 0 -1 0 -ve

Zj -3 -8 M-3 -M+8 -M

Cj -3 -8 0 0 -M

∆j 0 0 M-3 -M+8 0
S2 is the incoming vector. A1 is the outgoing vector. Key element is 1.

B CB XB X1 X2 S1 S2

S2 0 60 0 0 -1 1

X1 -3 80 1 0 1 0

X2 -8 120 0 1 1 0

Zj -3 -8 5 0 0

Cj -3 -8 0 0 0

∆j 0 0 +5 0
Since all∆js are positive, the solution is optimum

X1 = 80 , x2 = 120

Z = (3 x 80) + (8 x 120) = Rs.1200/

Problem 6

Solve the following LPP using Big M method


Maximise Z= 2X1+X2+3x3
Subject to constraints x1 + x2 + 2x3 ≤5
2x1 + 3x2 + 4x3 = 12
X1, x2, x3 ≥ 0

40
Solution :Introduce Slack and Artificial variables
MaximiseZ = 2x1+x2+ 3x3+ 0S1 – MA1
Subject to constraintsX1 + x2 + 2x3 + S1 = 5
2x1+ 3x2 + 4x3+ A1 = 12
Put x1, x2, x3 = 0 S1=5, A1 = 12
0 2 1 3 0 -M Ratio

5 X1 X2 X3 S1 A1
5/2 =
0 S1 5 1 1 2 1 0
2.5
12/4 =
-M A1 12 2 3 4 0 1
3
Zj -2M -3M -4M 0 -M

Zj-Cj -2M-2 -3M-1 -4M-3 0 0

Maximum negativeis -4M-3 smallest non negative is 2.5

X3 enters and S1 leaves

Revised simplex table

2 1 3 0 -M
Coefficient
Basic Value of
of basic X1 X2 X3 S1 A1
variable basic value
variable
3 X3 2.5 0.5 0.5 1 0.5 0

- M A1 2 0 1 0 -2 1

Zj 1.5 1.5-M 3 1.5+2M -M

Zj-Cj -0.5 0.5-M 0 1.5+2M 0

Key row

5/2 = 2.5 ½ = 0.5 ½ = 0.5 2/2 = 1 0/2 = 0

IInd row

12 = 12 – (2.5 x 4) = 2

41
2 = 2-(0.5 x 4) = 0

3 = 3 – (0.5 x 4) = 1

4 = 4-(1 x 4) = 0

0= 0-(0.5 x 4) = -2

1= 1- (0 x 4) = 1

2 1 3 0 -M
Coefficient
Basic Value of
of basic X1 X2 X3 S1 A1
variable basic value
variable
3 X3 2.5 0.5 0.5 1 0.5 0

-M A 2 0 1.0 0 -2 1

Zj 1.5 1-5-M 3 1.5+2M -M

Zj-Cj -0.5 0.5-M 0 1.5+2M 0

Largest negative value is -1. Smallest positive ratio is 2. Hence A1 leaves and x2 enters

Revised simplex tableHence x1 enters and x3 leaves

2 1 3 0
Coefficient of Basic Value of
X1 X2 X3 S1
basic variable variable basic value
3 X3 1.5 0.5 0 1 1.5

1 X2 2 0 1 0 -2
Key ratio=2/1=2, 0/1=0,1/1=1,0/1=0,2/-1=-2

Revised row

2.5=2.5-(2x0.5)=1.5

0.5=0.5-(0X0.5)=0.50

1=1-(OX0.5)=1

0.50=0.50-(1X0.5)=0

0.5=0.5-(2X0.5)=1.50

42
2 1 3 0 -M
Coefficient Value of
Basic
of basic basic X1 X2 X3 S1 Ratio
variable
variable value
1.5/0.5 =
3 X3 1.5 0.5 0 1 1.5
3
1 X2 2 0 1 0 -2 2/0 = α

Zj 1.5 1 3 2.5
Zj-Cj -0.5 0 0 2.5

Here X1 enters and X3 leaves

2 1 3 0
Coefficient Value of
Basic
of basic basic X1 X2 X3 S1
variable
variable value
2 X1 3 1 0 2 3

1 X2 2 0 1 0 -2

Zj 2 1 4 4
Zj-Cj 0 0 1 4

Key row

1.5/0.5=3 ,0.5/0.5=1,0/0.5=0,1/0.5=2,1.5/0.5=3

Pivotted Operations

2 = 2 –(3 x 0) = 2

0 = 0-(1 x 0) = 0

1 = 1- (0 x 0) = 0

0 = 0 – (2 x 0) = 0

-2 = -2(3 x 0) = -2

Since all Zj-Cj is positive or zero the solution is optimal

Hence max value of objective function

Z = 2x1 + x2 + 3x3 = 2x3 + 2 + 0 = 8

43
Problem 7

Max Z = 3x1+ 2x2 + 5x3 subject to constraint

X1 + 2x2 + x3 ≤ 430

3x1+ 2x3 ≤ 460

X1+4x2 ≤ 420

X1 >0 x2 ≥ 0 x3 ≥ 0

Solution

Maximize Z = 3x1 + 2x2 + 5x3 + s1+ 0S2 + 0S3

Subject to constraints

X1 + 2x2+ x3 + S1 + 0s2+ 0S3 = 430

3x1 + 0x2+ 2x3 + s1+ S2 + 0S3 = 460

X1 + 4x2 + 0X3 + 0S1 + 0S2 + S3 = 420

Simplex table

3 2 5 0 0 0
Value Ratio
Coefficient
Basic of
of basic X1 X2 X3 S1 S2 S3
variable basic
variable
value
430/1=430
0 S1 430 1 2 1 1 0 0
460/2=230
0 S2 460 3 0 2 0 1 0
420/0=infinite
0 S3 420 1 4 0 0 0 1

Zj 0 0 0 0 0 0

Zj-Cj -3 -2 -5 0 0 0
Key row elements

460/2 = 230 , 3/2 = 1.5 , 0/2 = 0 , 2/2 = 1, 0/2 = 0

½ = 0.5 , 0/2 = 0

Pivotted operation

44
3 2 5 0 0 0
Coefficient Value
Basic Exit
of basic of basic X1 X2 X3 S1 S2 S3
variable ratio
variable value
200/2 =
0 S1 200 -0.5 2 0 1 -0.5 0
100
=230/0=
5 X3 230 1.5 0 1 0 0.5 0
α
420/4 =
0 S3 420 1 4 0 0 0 1
105
Zj 7.5 0 5 0 2.5 0

Zj-Cj 4.5 -2 0 0 2.5 0

Key row
430 = 430- (230 x 1) = 200
1=1-(1.5 x 1) = 2
2=2-(0 x 1) =2
1= 1- (1 x 1) = 0
0.5=0.5-(0.5x1)=-0.5
0=0-(0x1)=0
Pivotted operations
420 = 420 – (230 x 0) = 420
1= 1- (1.5 x 0) = 4
0=0-(1 x 0)=0
0=0-(1x0) =0
0=0-(0.5 x 0) = 0
1=1-(0x0) = 1
All dijs are not positive. Hence the solution is not optimal
Largest negative value is -2

Here x2 enters a S1 leaves

45
3 2 5 0 0 0
Coefficient Value of
Basic
of basic basic X1 X2 X3 S1 S2 S3
variable
variable value
2 X2 100 -0.25 1 0 0.5 -0.25 0

5 X3 230 1.5 0 1 0 0.5 0

0 S3 20 2 0 0 -2 1 1
Key row

200/2 = 100 -0.5/2 = -0.25 2/2 = 1, 0/2 = 0

½ = 0.5 -0.5/2 = -0.25 0/2 = 0

Pivoted row operations

2nd row

230 = 230 – (100 x 0) = 230

1.5 = 1.5 –(-0.25 x 0) = 1.5

0= 0- (1 x 0) = 0

1 = 1- (0 x 0) = 1

0= 0- (0.5 x 0) = 0

0.5 = 0.5 – (-0.25 x 0) = 0.5

0 = 0-(0 x 0) = 0

IIIrd row

420 = 420 – (100 x4) = 20

1 = 1-(-0.25 x 4) = 2

4 = 4- (1 x 4) = 0

0 = 0-(0 x 4) = 0

0=0-(0.5 x 4) = -2

0=0-(-0.25 x 4) = 1

1=1-(0x4) =1

Since all dijsare positive the solution is optimal


46
Optimal value x2 = 100

X3 = 230

X1 = 0

Hence the value of objective function

Z = 3x1 + 2x2+ 5x3

= 2 x 100 + 5 x 230 = 1350

Special type of LPP problems

1. Infeasible solution

An infeasible problem is a problem in LPP which is not capable of mathematical


solutions due to contradictory nature of constraints.

Here one or more artificial variables remain in the final solution with non zero values

2. Unbounded problem
An LPP is said to be unbounded solution, if the objective function can be increased or
decreased arbitrarily, then is no finite optimum of the objective function. Here no exit
variable exists since the simplex table contain only negative ratios.
3. Problems with multiple optimum
There can be more than one optimal solution for certain problem. In such cases, the
problem does not have a single or unique solution. In the final simplex table, if the
index row indicates the value for any non basic variable to zero it means that alternate
optimal solution exists for the problem. Thus the problem has an alternate solution
which is optimal.

Degeneracy in simplex problem.


Degeneracy is a condition that may occur during computational process of LPP

A basic feasible solution of LPP is said to be degenerate, if at least one of the basic variable is
zero. In an LPP, degeneracy occurs at the following stages.

1. Degeneracy may appear at the very firstiteration if one of the basic variable is zero.
2. Degeneracy if may occur at some subsequent iterations This happens when there is a
tie for selecting exit variable

When the simplest algorithm is applied to a degenerate feasible solution to get a new BFs,
the value of the objective function may remain unchanged.

In some cases, the problem of cycling may occurs, the same sequence of simplex table are
repeated for ever without reaching optimum solution. In this case

47
ofcycling,variableswhich have left the basic solution in the previous iteration reenters and
no optimal solution will be reached.

Duality in simplex algorithm

Every linear programming problem is associated with another linear programming


problem called Dual.Original problem is called Primal problem

Duality theorem sates that for every maximization (minimization) problem in linear
programming there is a unique similar problem of minimization (or maximization)
involving same data which describe the original problem

Procedure to find the dual of a given primal

1. If the primal is maximization, dual is minimization and vice versa


2. Constraints in the right hand side of the constraints of primal becomes the coefficients
in the objectives function of the dual and vice versa
3. Dual of the dual programme will be the original problem itself
4. In the constraints, first coefficients of different constraints in the primal becomes the
first constraints in the dual and son

Applications of dual

1. If the primal problem contains a large number of rows (constraints) and smaller
number of columns (variables) computational procedures can be considerably reduced
by converting it into dual
2. Calculation of dual checks the accuracy of the primal solution
3. Disability in linear programme has certain far reaching consequence of economic
nature
4. It gives additional information as to how the optimal solution changes as a result of
changes in the coefficient and formulation of the problem. This is termed post
optimality or sensitivity analysis

Sensitivity analysis

After formulating mathematical model to a linear programming problem and then


attaining optimum solution of the problem, it may be required to study the effects of
changes (discrete or continuous) in the different parameter of problem on optimum
solution

Analysis of such problems assessing due to the slight change made in the parameter or
structure of a given LPP after attaining its optimum solution known as

post optimality analysis or sensitivity analysis.

The sensitivity analysis is concerned with the extent of optimal solution of LPP for
changes in one or more

48
1. Coefficient in the objective function
2. Coefficient of variables in the constraints
3. RHS constraints in the constraints

The sensitivity analysis makes the LPP solution to a dynamic tool that evaluates change
conditions.

The following post optimally problems can be answered with the help of sensitivity
analysis

1. When the profit contribution increases or decreases


2. When the resources availability is curtailed or increased
3. If a new activity is added or existing and is deleted

Advantages of LPP

1. Optimum use of productive factors within the firm


2. Improved quality of decision
3. Use of sensitivity analysis helps the management to modify the solution and obtain
modified solution easily
problem 8

Write the dual of the problem

Maximize Z = 3x1+ x2 + 2x3

Subject to constraints

x1 + x2+ x3 ≤ 5

2x1 + x3 ≤ 10

X2 + 3x3 ≤ 15

X1, x2 , x3 > 0

Solution

Dual

Minimize Z = 5y1+ 10y2 + 15 y3

Subject to constraints y1+ 2y2 ≥ 3

Y1 + y3 ≥ 1

Y1 + y2+ 3y3 ≥ 2

Y1, y2, y3 >0

49
Problem 9

Find the dual of the problem

Maximize Z = 30x1 + 40x2

Subject to constraints 6x1+ 12x2 ≤ 120

8x1+ 5x2 ≥ 60

3x1+ 4x2 = 50

X1, x2 ≥ 0

Solution

Equality to be changed as ≤ and ≥

Minimize

Z = 120 W1 – 60 W2 + 50W3 – 50W4

Subject to constraints

6W1- 8W2 + 3W3 – 3W4≥ 30

12W1 – 5W2 + 4W3 – 4W4 ≥ 40

W1, W2, W3, W4 >0

This is a mixed constraint

Second constraints to be multiplied by -1

-8X1-5X2 ≤ 60

(Third constraints equation has to be converted unit two inequalities in opposite direction)

3x1+ 4x2 ≤ 50

3x1+ 4x2 ≥ 50

Second inequality to be multiplied by -1

-3x1-4x2 ≤ -50

Problem can be now converted to dual

Problem 10

Minimise Z= 2x2+5X3

Subject to

50
X1+X2>2

2X1+X2+6X3<6

X1-X2+3X3=4

X1,X2,X3>0

Solution : This is a problem with mixed constraints .Since it is a minimisation Problem,


all the constraints should have sign>_

Multiply the 2nd constraint by -1

-2x1-x2-6x3> -6

Third constraint is equality

X1-x2+3x3<4

X1-x2+3x3>4

Multiply the first inequality by -1

-x1+ x2 – 3x3 ≥ -4

Dual of the problem

Maximize Z = 2w1-6W2 -4W3 + 4W4

Subject to

W1-2W2-W3 + W4 ≤ 0

W1 –W2+ W3 – W4 ≤ 2

0W1-6W2 -3W3+ 3W4 ≤ 5

W1,w2,w3>0

Activity Question

11. Write the dual of problem

Minimise Z =4x1+ 2x2+ 5x3

Subject toconstraints x1 + x2 <10

3x1 + x2 + x3 > 23

7X1 – x2 = 4

X1, x2, x3 ≥ 0

51
Problem 12

Write the dual of the following primal LPP

Minimize Z = 4x1+ 5x2 -3x3

Subject to x1 +x2 + x3 = 22

3x1 + 5x2 – 2x3 ≤ 65

X1 + 7x2 + 4x3 ≥ 120

X1 + x2 ≥ 0 x1, x2, x3 ≥ 0

Problem 13

UNI Feb 2015 10 marks

Use simplex dual method to solve the LPP

Minimise Z = 4x1+ 2x2 + 3x3

Subject to constraints 2x1+ 4x3 ≥ 5

2x1+3X2 +x3 ≥ 4

X1, x2, x3 ≥ 0

The dual of the problem

Maximize Z = 5w1+ 4w2

Subject to constraints

2w1+ 2w2 ≤ 4

W1+ 3w2 ≤ 2

4w1+ w2 ≤3

W1,w2 ≥ 0

Introducing stack variable

2w1+ 2w2 + S1+ 0s2 + 0s3 = 4

0w1 + 3 w2 + 0s1+ 0s2 + s3 = 2

4w1+w2+0s1+ 0s2 +s3=3

Put w1, w2 = 0, s1 =4, s2 = 2, s3 = 3

52
5 4 0 0 0
Coefficient Value of Ratio
Basic
of basic basic w1 w2 S1 S2 S3
variable
variable value
4/2=2
0 S1 4 2 2 1 0 0
2/0
0 S2 2 0 3 0 1 0
¾=0.75
0 S3 3 4 1 0 0 1

Zj 0 0 0 0 0

Zj-Cj -5 -4 0 0 0 0

S3 leaves w1 enters

5 4 0 0 0
Coefficien Value of
Basic
t of basic basic w1 w2 S1 S2 S3
variable
variable value
2.5/1.5 = 2
0 S1 2.5 0 1.5 1 0 0
2/3=0.67
0 S2 2 0 3 0 1 0
0.75/0.25 = 3
5 W1 0.75 1 0.25 0 0 0.25

Zj +5 +1.25 0 0 1.25

Zj-Cj 0 -2.75 0 0 1.25

New value for key row

¾ = 0.75, 4/4 = 1, ¼ = 0.25 0/4 = 0 0/4 = 0 ¼ = 0.25

Pivotted operation

Ist row 4 = 4- (0.75 x 2 ) = 2.5

2 = 2 – (1 x2) = 0

2 = 2- (0 .25 x 2 ) = 1.5

1= 1- (0x 2) = 1

53
0= 0-(0x2) = 0

0= 0-(0x 2) = 0

2nd row

2 = 2- (0.75 x 0) = 2

0= 0- (1x 0)= 0

3= 3-(0.25 x 0) = 0

0=0-(oxo)=0

1 = 1-(0 x 0) = 1

0 = 0-(0 x0) = 0

Largest negative value corresponds to W2

Hence w2 enters and S2 leaves

5 4 0 0 0
Coefficient Value of
Basic
of basic basic w1 w2 S1 S2 S3
variable
variable value
0 S1 1.5 0 0 1 -0.405 -0.5

4 W2 0.67 0 1 0 0.33 0

5 W1 0.5825 1 0 0 -0.0825 0

Pivotted operation (2nd row)

Key row 2/3 = 0.67 , 0/3 = 0, 3/3 = 1 0/3 = 0

1/3 = 0.33 0/3 = 0

1st row

2.5 = 2.5 - (1.5 x 0.67) = 1.5


2.6 0= 0-(1.5 x 0) = 0

1.5 = 1.5 –(1.5 x 1) = 0

1= 1 – (1.5 x 0) = 1

0 = 0-(1.5 x 0.33) = -0.495

54
0 = 0- (1.5 x 0) = 0

111rd row

0.75 = 0.75 – (0.025 x 0.67) = 0.5825

1= 1-(0.25 x 0) = 1

0.25 = 0.25 – (0.25 x 1) = 0

0 = 0- (0.25 x 0) = 0

0 = 0- (0.25 x 0.33) = -0.0825

0.25 = (0.25 x 0) = 0.25

5 4 0 0 0
Coefficient Value of
Basic
of basic basic w1 w2 S1 S2 S3
variable
variable value
0 S1 1.5 0 0 1 -0.495 -0.5

4 W2 0.67 0 1 0 0.33 0

5 W1 0.5825 1 0 0 -0.0585 0.25

Zj 5 4 0 0.9075 1.25

Zj-cj 0 0 0 0.9075 1.25

Since all dijs are positive and zero, the solution is optimal

Hence the optimal solution is

W1 = 0.5825 w2 = 0.67

Maximum value of objective function

Z = 5 x 0.5825 + 4 x 0.67 + 0 = 5.68

Activity Question 14

Solve the problem by simplex method

Minimise Z = 20x1+ 24x2+ 18x3

Subject to constraints

55
2x1 + x2 + x3 ≥30

X1 + x2 + x3 ≥ 20

X1+ 2x2 + x3 ≥ 20

X1 x2 x3 ≥ 0 (Ans Z=404 when X1=10,X2=4,X3=6)

Problem 15

Maximise Z = 5x1+ 3x2

Subject to 2x1 + x2 ≤ 1

X1 + 4x2 ≥ 6, x1, x2 ≥ 0

Problem 16

Minimize Z = 12x1 + 20x2

Subject to 6x1 + 8x2 ≥ 100

7x1 + 12x2 ≥ 120

X1 x2 >0 (Ans X1=15,X2=5/4 Min Z= 205)

Problem 17

Use simplex method to

Min Z = 3x1+ 5x2 + x3

subject to constraints

3x1+ 4x2 – 5x3 ≤8

2x1+ 6x2 + x3 ≥ 7

X1- 2x2 + x3 ≤ 5

X1 x2, x3 ≥ 0

Solution

The minimization problem can be converted to maximization problem as

Max Z = -3x1 – 5x2 – x3

Subject to 3x1+ 4x2 – 5x3 ≤ 8

2x1 + 6x2 + x3 ≥ 7

X1 – 2x2+ x3 ≤ 5

56
Introduce stack artificial variables

Max z = -3x1-5x2 –x3 +0 s1 + 0s2+ 0s3-MA1

3x1 + 4x2 – 5x3 + S1 = 8

2x1+ 6x2+ x3 + 0S1 – S2 + A1 = 7

X1-2x2+ x3 +S3 = 5

Putting x1, x2, x3 = 0

S1 = 8, A1 = 7, S3 = 5

5 4 0 0 0
Coefficient Value
Basic
of basic of basic X1 X2 X3 S1 S2 S3 A1 Ratio
variable
variable value
S1=0 S1 8 3 4 -5 1 0 0 0 8/4=2

-M A1 7 2 6 1 0 -1 0 1 7/6=1.17
5/-2=
S3=0 S3 5 1 -2 1 0 0 1 0
-2.5
Zj -2M -6M -M 0 M 0 -M

Zj-Cj -2M+3 -6M+5 -M+ 0 M 0 0


Largest negative value is -6M + 5

Here x2 enters and A1 leaves

Key row

7/6 = 1.17 2/6 = 0.33 6/6 = 1 , 1/6 = 0.17 , 0/6 = 0,

-1/6 = -0.17 0/6 = 0 1/6 = 0.17

Pivotted row

1st row 2nd row


8 = 8-(1.17 x 4) = 3.32 5 = 5- (1.17 x -2) = 7.34
3= 3 – (0.33 x 4) = 1.68 1 = 1- (0.33 x -2) = 1.66
4 = 4- (1 x 4) = 0 -2 = -2 –(1x -2) = 0
-5 = -5 –(0.17 x 4) = -5.68 1 =1-(0.17 x -2) = 1.34
1 = 1- (0 x 4) = 1 0 = 0- (0 x -2) = 0
0 = 0-(-0.17 x 4) = 0.68 0 = 0-(0.17x -2) = -0.34
0 = 0-(0x 4) = 0 1=1-(0x-2)= 1

57
0=0-(0.17x4)= - 0.68

0=0-(0.17x2)= 0.34

Revised simplex table

5 4 0 0 0 0 -M
Coefficie
Basic
nt of Value of
variabl X1 X2 X3 S1 S2 S3 A1 R
basic basic value
e
variable
0 S1 3.32 1.68 0 -5.68 1 0.68 0 -0.68

-5 X2 1.17 0.33 1 0.17 0 -0.17 0 0.17

0 S3 7.34 1.66 0 1.34 0 -0.34 1 0.34

Zj -1.65- -5 -0.85 0 0.85 0 -0.85


M+
Zj – Cj 1.35 0 0.15 0 0.85 0
0.85

All dijs are positive. Hence solution is optimal. Optimum value of objective function
where x2 = 1.17 ,X1 = 0 x3 = 0

Minimum value of objective function =

3 x 0 + 5 x 1.7 + 0

= 5.85

58
Unit II

Assignment Problem

Assignment problem is a special case of transportation problem in which objective is to


assign a number of origins (or persons) to the equal number of destination (or tasks) at
minimum cost.

Assignment problems deals with the allocation of jobs to person or allocation of tasks to
facilities, the assignment is to be done on the basics of one job to one person or one task
to one facility

Cost matrix

The assignment problem can be stated in the form of

n x nmatrix called cost matrix where Cij is the cost of assigning ith source to jth
destination

Source 1 2 3 ….. n

1 C11 C12 C13 …. C1n

2 C21 C22……C2n

n Cn1 Cn2 … Cnn

Mathematically an assignment can be stated as follows

Minimise the total sost

Z=∑ ∑ Cijxij

Subject to condition ∑!" = 1 for j = 1, 2 ….. n for which one job is to be done
by ith person where i=1 ….. n

∑!" = 1fori = 1,2 … .n which means one person to be assigned the jth job

MethodofsolvingAssignment problem
(Hungarian Alogarithm or Reduced Matrix Method)

1. Subtract the smallest element of each row in the cost matrix from every element of
that row
2. Subtract the smallest element of each column in the reduced matrix from every
element of that column
3. a.Starting with rows of the matrix obtained, examine all the rows having exactly one
zero element. Enclose this zero within showing that assignment is made there.

59
Cross and all other zeros in the column to show that they cannot be sued to make
other assignments. Proceed in this way until last row is examined
b.Examine all the columns with one unmarked zero. Marked at the zero and cross
all the zeros of the row in which is marked. Proceed in this way till the last column
is examined
c.Continue these operations (a) and(b) successively until we reach any of the
following solution
d. If all the zeros are enclosed by in this case we have maximal assignment and
assignment in every row or column or got solution.
2. Remaining unmarked zeros lie at least two rows or column. It does not contain
assignment in all rows and all columns
In this case the following procedure is followed
4. Draw the minimum number of horizontal and vertical lines necessary to cover all
zeros at least once.
5. Select the smallest of the elements which is not covered by lines.subtract it from all
the elements that do not have a line through them, add it to the every element that lies
at the interaction of two lines and leave the remaining elements of the matrix
unchanged.
6. Now reapply steps 3 to 5 to the modified matrix

Unbalanced Assignment Problem


An assignment problem is said to be unbalanced ,whenever the number of tasks (job)
is not equal to the number of facilities (person). Thus the cost matrix of an unbalanced
assignment problem is not a square matrix. For the solution of such problems we add
dummy rows or columns to the given matrix to make square. The cost on dummy
rows or columns are taken to zero. Now the problem reduces to balanced assignment
problem and can be solved by assignment algorithm
Maximization problem
If the objective of the allocation is to minimize some variable, such as cost, time etc it
is known as minimization problem. If on the contrary the objective is to maximize
some variable such as sales ,revenue and profit it is known as maximization problem.
In order to solve a maximization problem, it has to be converted into a minimization
problem. This can be done by multiplying each element of the related matrix by (-1) .
After converting a maximization problem into a minimization problem, it can be
solved by Hungarian problem or minimization problem can be converted to
maximization by subtracting highest element from all the elements. The problem now
converted to minimization problem.
Problem 1(Nov 2012 University SDE)
Find the optimal assignment of jobs to machines which will minimize the total set up
time
machines
Job
1 2 3 4 5
1 10 11 4 2 8

60
2 7 11 10 14 12
3 5 6 9 12 14
4 13 15 11 10 7

This is an unbalanced assignment problem. Convert this to balanced assignment


problem by adding dummy row
machines
Job
1 2 3 4 5
1 10 11 4 2 8
2 7 11 10 14 12
3 5 6 9 12 14
4 0 0 0 0 0

Row least numbersubtraction


8 9 2 0 6
0 4 3 7 5
0 1 4 7 9
6 8 4 3 0
0 0 0 0 0

Column least number subtraction


8 9 2 0 6
0 4 3 7 5
0 1 4 7 9
6 8 4 3 0
0 0 0 0 0

Allocation
8 9 2 [0] 6
[0] 4 3 7 5
0 1 4 7 9
6 8 4 3 [0]
0 [0] 0 0 0

8 9 2 0 6
0 4 3 7 5
0 1 4 7 9
6 8 4 3 0
0 0 0 0 0

61
8 8 1 0 7
0 3 2 6 5
0 0 3 6 9
6 7 3 2 0
1 0 0 0 1

8 8 1 [0] 7
[0] 3 2 6 5
0 [0] 3 6 9
6 7 3 2 [0]
1 0 [0] 0 1
Optimum solution
Job 1 – Machine 4-2
Job II – machine 1 – 7
Job III – Machine 2-6
Job IV – Machine 5-7
Hence total optimal cost = 2 + 7 + 6 + 7 = Rs. 22
Problem 2 ( UNI Feb 2016)Marks 20 Marks
A company has four machines to do three jobs. Each job can be assigned to one and
only one machine. The cost of each job on each machine is given in the following
table.
What are the job assignments which will minimize the cost?
Solutions
Machines
1 2 3 4
A 18 24 28 32
B 8 13 17 19
C 10 15 19 22

This is an unbalanced assignment problem


18 24 28 32
8 13 17 19
10 15 19 22
0 0 0 0

62
Now it becomes a balanced assignment problem
0 6 10 14
0 5 9 11
0 5 9 12
0 0 0 0

[0] 6 10 14
0 5 9 11
0 5 9 12
0 [0] 0 0

Draw minimum number of vertical and horizontal lines to cover all zeros
0 6 10 14
0 5 9 11
0 5 9 12
0 0 0 0

Subtract 5 from elements and add 5 to the intersection points


0 1 5 9
0 0 4 6
0 0 4 7
5 0 0 0

[0] 1 5 9
0 0 4 6
0 0 4 7
5 0 [0] 0

3 rdrow allocation not made. Draw minimum number of horizontal and vertical
lines to cover all zeros.

63
0 1 5 9
0 0 4 6
0 0 4 7
5 0 0 0
Least element uncovered is 4. Subtract 4 with respective elements and add 4 at
intersection points.
0 1 1 5
0 0 0 2
0 0 0 3
9 4 0 0

[0] 1 1 5
0 [0] 0 5
0 0 [0] 3
9 4 0 [0]

0 1 1 5
0 0 0 2
0 0 0 3
9 4 0 0

Allocation is Minimum cost

A- 1 = 18 + 13+ 19
B- 2 = Rs. 50

C- 3

D- 4

Problem 3 (UNI Feb 2015 10 Marks )

Solve the assignment problem

64
A B C D
I 1 4 6 3
II 9 7 10 9
III 4 5 11 7
IV 8 7 8 5
Solution

Subtracting lowest element from each row

0 3 5 2

2 0 3 2

0 1 7 3

3 2 3 0

Subtracting lowest element from each column

0 3 2 2
2 0 0 2
0 1 4 3
3 2 0 0

[0] 3 2 2
2 [0] 0 2
0 1 4 3
3 2 [0] [0]

Minimum no of vertical / horizontal lines to cover all zeros

0 3 2 2
2 0 0 2
0 1 4 3
3 2 0 0
Subtracting 1 from elements not marked and add 1to intersection points

65
0 2 1 1

2 0 0 3

0 0 3 3

4 2 0 1

[0] 2 1 1

2 0 [0] 3

0 [0] 3 3

4 2 0 1
I–1

II- 3

III- 2

IV – 4

Optimum cost

= 1+ 10 + 5 + 5 = 21

Problem 4

Five different machines can do any of the five required jobs with different profits
resulting from each assignment as given below

A B C D E
1 30 37 40 28 40
2 40 24 27 21 36
3 40 32 33 30 35
4 25 38 40 36 36
5 29 62 41 34 39
Find out the maximum profit possible through optimal assignment

Solution

66
This is a maximization problem. So convert into minimization problem. Subtract all
elements from highest of elements (ie 62). After this problem gets converted to
minimization problem.

32 35 22 34 22

22 38 35 41 26

22 30 29 32 27

37 24 22 26 26

33 0 21 28 23
Subtracting smallest element from each row from every element of row and smallest
element from each column from every element.

10 3 0 12 [0]
[0] 16 13 10 4
0 8 7 10 5
15 2 0 4 4
33 [0] 21 24 23

10 3 0 8 0
0 16 13 15 4
0 8 7 6 5
15 2 0 0 4
33 0 21 24 23
Smallest element uncovered is 4 subtract 4 from elements and add 4to intersection
points

14 3 [0] 8 0
0 12 9 11 [0]
[0] 4 3 2 1
19 2 0 [0] 4
37 [0] 21 24 23
Assignment is complete

I- C – 40
II- E – 36
III- 1 – 40

67
IV- D – 36
V- B – 62

Total profit maximum

= 40+ 36 + 40 + 36 +62 = 214

Prohibited assignment (Restricted assignment)

In some assignment problems, it may not be possible to assign a particular task to a


particular facility due to space or size of the task or other restrictions. In such
situations, we can assign a very cost (sayα ) to the corresponding cells. So that it will
be automatically excluded in the minimizing process of assignment.

Travelling sales man problem (routing problem)

Travelling sales man problem is a special type of routing problem. The routing
problems are those where we have to select a route, from an origin to destination,
which yields minimum cost.

Suppose a salesman has to visit in cites .He wishes to start from a particular city visits
each city once and then return to his starting time. Objective is to select the sequence
in which cities are visited in such a way that his total travelling time is minimized. If
there are 4 cities A,B, C, D then once solution can to A to C, C to D, D to B, B to A

Formulation of a travelling sales man problem as an assignment problem

Travelling sales man problem is very similar to assignment problem except that in the
former there is an additionalrestriction . The addition restriction is choosing a
sequence which can minimize cost. This is the route condition.

Problem 5

A sales man has to visit fivecities A, B, C, D, E. The between cities are given in the
table below. If the sales man streets from city A and has to comeback to city A which
route should be select so that the distance travelled by him is minimized.

A B C D E
A - 4 7 3 4
B 4 + 6 3 4
C 7 6 - 7 5
D 3 3 6 - 7
E 4 4 5 7 -
Solution

Assign α to the cells having no values

68
A B C D E
A α 4 7 3 4
B 4 Α 6 3 4
C 7 6 α 7 5
D 3 3 7 α 7
E 4 4 5 7 α
Making row and column subtraction and making zero assignment we have

A B C D E
A α 1 4 [0] 1
B 1 α 3 0 1
C 2 1 α 2 [0]
D [0] 0 4 α 4
E 0 [0] 1 3 α

Draw minimum number ofvertical and horizontal lines to cover zeros

Α 1 4 0 1
1 Α 2 0 1
2 1 α 2 0
0 0 4 α 4
0 0 0 2 α
Least element uncovered by lines is 1. Subtracting or adding 1 to the respective elements and
making zero assignment

Α 0 3 0 0
0 Α 1 0 0
2 1 α 3 0
0 0 3 α 4
0 0 0 3 α
Draw minimum number of vertical lines and horizontal lines to cover all zeroes
Α 0 2 0 0
0 Α 1 0 0
2 1 Α 3 0
0 0 3 α 4
0 0 0 3 α

69
Draw minimum no. of horizontal vertical lines

α 0 2 0 0
0 Α 1 0 0
2 1 α 3 0
0 0 3 α 4
0 0 0 3 α
Draw

Α [0] 2 [0] 0
0 Α [1] [0] α
2 1 α 3 [0]
[0] [0] 3 α α
[0] 0 0 3 α
Minimum distance travelled

A – D, B – C, C- E, D – B, E- A

Total distance leveled = 3 + 6 + 5 + 3 + 4 = 21

A–D–B–C–E–A

Activity Questions

Problem 6

Solve the following travelling sales man problem

From city 1 2 3 4 5

1 - 10 25 25 10
2 1 - 10 15 2
3 8 9 - 20 10
4 14 10 24 - 15

5 10 8 25 27 -
Problem 7

Given below is a matrix showing profit for different jobs done through differentmachines .
Find an assignment programme which will maximize the total profit

70
M1 M2 M3 M4
J1 51 53 54 50
J2 47 50 48 50
J3 49 50 60 61
J4 63 64 60 61
Problem 8

Solve the following assignment problem for minimizing cost

I II III IV
A 32 26 35 38
B 27 24 26 32
C 28 22 25 34
D 10 10 16 16

Problem 9

A company is faced with the problem of assigning six different machines to five different
jobs. The costs are estimated as follows (in hundreds of rupees)

jobs

machines A B C D E
1 5 10 2 12 2
2 4 10 3 14 6
3 6 13 5 16 9
4 7 14 4 18 12
5 8 14 6 18 12
6 12 18 10 20 12
Solve the problem assuming that objective is to maximize total cost

(Ans Total Cost Z= 4000)

Problem 10

The owner of a small machine shop has four machines available to assign job for the day.
Five jobs are affected with expected profit fore each machine on each job which are as
follows.

jobs

Machine A B C D E
1 62 78 50 111 82
2 71 84 61 73 59
3 87 92 111 71 81
4 48 64 87 77 80

71
Find by using assignment method, the assignment of machines to job that will result in a
maximum profit which job should be declined

(Ans376 ,5th job to be declined since 5 the mechanic is dummy)

Problem 11 UNI SDE Dec 2019(20 marks)

Find the optimal solution for the given assignment problem

Machines

M4 M5
Job M1 M2 M3

3 9
A 7 5 2

4 5 10
B 6 5

6 8
C 5 4 5

2 6
D 8 3 3

Solution

This problem is an unbalanced Assignment problem.Convert this to balanced


assignment problem

7 5 2 3 9

6 5 4 5 10

5 4 5 6 8

8 3 3 2 6

0 0 0 0 0

Apply Hungarian Principle

72
Subtract the lowest element from each row

1 7
5 3 0
0 1 6
2 1
2 4
1 0 1
0 4
6 1 1
0 0
0 0 0

Subtract the lowest element from each column

5 3 0 1 7

2 1 0 1 6

1 0 1 2 4

6 1 1 0 4

0 0 0 0 0

Allocation

5 3 [0] 1 7
2 1 0 1 6
1 [0] 1 2 4
6 1 1 [0] 4
[0] 0 0 0 0
All the rows does not have assignment

Draw Minimum no of horizontal and vertical lines to cover zeroes

5 3 0 1 7
2 1 0 1 6
1 0 1 2 4
6 1 1 0 4
0 0 0 0 0

73
Lowest element not covered by lines is 1 .Subtract the lowest element from other elements
and add 1 to the intersection points

4 3 0 0 6
1 3 0 0 5
0 0 1 2 3
5 1 1 0 3
0 1 1 1 0
Allocation

4 3 0 0 6
1 3 0 0 5
0 [0] 1 2 3
5 1 1 [0] 3
0 1 1 1 [0]
Draw Minimum no of horizontal and vertical lines to cover zeroes

4 3 0 0 6
1 3 0 0 5
0 0 1 2 3
5 1 1 0 3
0 1 1 1 0

3 2 0 0 6
0 2 1 0 6
1 0 2 1 2
6 1 1 0 4
0 1 1 1 1

3 2 [0] 0 6
0 2 1 0 6
1 [0] 2 1 2
6 1 1 [0] 4
[0] 1 1 1 1
Allocation

I- M3 – 2
II- M5 – 10
III- M2 – 4

74
IV- M4 – 2
V- M1 – 0

Optimum Allocation = 2+ 10 + 4 + 2 +0 = 18

TRANSPORTATION PROBLEM

Transportation problem is a special kind of linear programming problems in which goods


are transported from a set of sources to a set of destinations subject to supply and demand of
the source or destination on respectively such that the total transportation cost is minimum.

The objective is to transport various amounts of s single homogenous

commodity that are stored at several origins is a number of destination.

Transportation problem is applicable to other problems also example allocation problem,


product mixetc

Transportation techniques can be applied not only to cost minimizing problem but also to
time minimizing problem, profit maximizing problem.

Uses of transportation problem

1. To minimize the transportation costs from factories to ware house or ware houses to
markets.
2. To determine the lowest cost allocation for a new factory
3. To determine minimum cost production schedule.

Transportation table

D1 D2 ………Dn Availability

O1 c11 cij cin a1

02 c21 c22 ….. c2n a2

0m cm1 cm2 cmn an

Requirement b1 b2 ……….. bn

The matrix is known as transportation table. Origin is denoted as 01, 02….. On and
destination is noted as D1, D2,….Dn and Cj for the cost associated with transportation
problem.

LPP for transportation problem

Minimise Z = ∑!# ∑!" , Cijxij

Subject to ∑!" = ai for 1, 2…… n xij ≥ 0

75
Basic assumptions in transportation techniques

1. ∑ $ = ∑ $ is total qty available for destination is equal to total requirement in


different destinations together.
2. Unit transportation cost from one origin to a destination is certain
3. Unit cost is independent of the quantity transported
4. Objective is to minimize the transportation cost

Definitions

Feasible solution :Feasible solution to a transportation problem is a set of non negative


individual allocation which satisfy the row and column restriction.

Therefore for feasibility the sum of allocation in the rows must be equal to availability in
that row.

Similarly sum of allocations in the column must be equal to demand in that column

Basic feasible solution : A feasible solution to a maximum transportation problem is said


to be basic feasible solution, if the total number of allocation is equal to (m+n-1) if m is
the no of rows and n is the no of coloumns

Optimal solution : A feasible solution (basic or not) is said to be optimal, if it minimizes


the total transportation cost.

Non degenerate basic feasible solution : A feasible solution of a maximum


transportation problem is said to be degenerate basic feasible solution if the

1. No of allocations must be equal to m+n -1


2. The allocations are in independent positionSteps for solving a transportation problem
1. Set up a transportation table with m rowsrepresenting the origin and n columns
representing destination
2. Develop an initial feasible solution to the problem
3. Test whether the solution is optimal or not
4. If the solution is not optimal, modify the allocation
5. . Repeat steps 4 and 5 until an optimal solution is obtained.

Initial basic feasible solution

Initial feasible solution are those which satisfy the rim requirement. That is the allocation
made in every row taken together is equal to availability shown in that row similarly for each
column, the total allocation should be equal to the requirement in that column

The initial basic solution is determined using

1. North west corner rule


2. Lowest cost entry method

76
3. Vogel approximation method is more preferred since the initial basic feasible solution
obtained either optimal or very close to the optimal solution.

North West corner rule

Alogarithm steps

Step 1 : Allocate to the cell(1,1) maximum possible amount which is the minimum of row
total and column total. So either arow or column total gets exhausted. So cross of total
row or column as the case may be

Step 2 : Consider the reduced matrix. In that matrix, allocate to the cell (1,1) maximum
possible amount (Which is the minimum of present row total or column total)

Step 3 : Repeat the above steps until all the available quantities are exhausted.

Problem 1

Find the initial feasible solution to the transportation problem by northwest corner rule

Origin D1 D2 D3 Supply

01 2 7 4 5

02 3 3 1 8

03 5 4 7 7

04 1 6 2 14

Demand 7 9 18
Solution

5 2 7 4 5
3 3 1 8
5 4 7 7
1 6 2 14
7 9 18

2 3 3 1 8
5 4 7 7
1 6 2 14
2 9 18

77
6 3 1 6
4 7 7
6 2 14
9 18

3
4 7 7

6 2 14
3 18

4 7 4
14 14
2
18

Various allocation are as follows


5
2 7 4 5
3 6
3 3 1 8
3 4
5 4 7 7
14
1 6 2 14
7 9 18

Total transportation cost

(5x2) + (2x3) + (3x6) + (4x3) +(4x7) + (2x14) = 102 Rs.

Lowest Costentry method

Choose the cell having lowest cost in the matrix allocate there, as much possible which is
the minimum of row total and column total. This either a row total or column total is
exhausted.

Cross off the corresponding row or column from the reduced matrix, locate the cell
having lowest cost. Allocate to that cell maximum possible thus leading to a feasible
reduced matrix.

Continue this process until all requirements are exhausted

78
Problem 2

Find the initial basic feasible solution to the following transportation problem by lowest
cost entry method
W1 W2 W3
F1 2 7 4 5
F2 3 3 1 8
F3 5 4 7 7
F4 1 6 2 14
7 9 18
Solution

Lowest cost on cells (2,3) & (4,1) select one of these say (2,3). Allocate minimum of
8and 18 ie.8. Thus F2 is exhausted

2 7 4 5
8
3 3 1 1
5 4 7 7
1 6 2 14
7 9 10

Next allocation & F4 (1st column)

2 7 4 5
5 4 7 7
7
1 6 2 7
9 10

7 4 5
4 7 7
6 7 2 7
9 10

Minimum cost
3 4
7 5

4 7 7

9 3

79
2
7 2 7 2
7
4 7

9 9

2 3
2 7 4 5
8
3 3 1 8
7 4
5 7 7
7 7
1 6 2
7 9 18
Total transportation cost

= (2 x7) + (3x4) + (8x1) + (7x4) + (7 x1) + (7x2) =Rs. 83

VOGELS APPROXIMATION METHOD (VAM)

Step 1

Since this method, we write the difference between smallest and second smallest costs in
each column below the corresponding columns, within brackets.similarly write similar
difference in each row to the weight of corresponding row. These differences are known
as penalty

Step 2

Select the row or column having the largest penalty and allocate the maximum possible
amount to the cell with the lowest cost in that row or column as the case may be. This
either the row total is column total is completely exhausted (cross of that row or column).
Construct thereduced matrix with the remaining rows and column

Step 3 : For the reduced matrix obtained apply step 1 & 2 until all rows and column totals
are exhausted. The initial solution obtained by Vogelsmethod is more close to the optimal
solution than the solution obtained by other two method

Problem 3

Find the initial solution for transportation problem by Vogels method.

80
W1 W2` W3 Supply
F1 2 7 4 5
F2 3 3 1 8
F3 5 4 7 7
F4 1 6 2 14

W1 W2` W3 Supply
5
F1 2 7 4 5(2)
F2 3 3 1 8(2)
F3 5 4 7 7(1)
F4 1 6 2 14(1)
Demand
7 9 18
(1) (1) (1)

Penalties
For rows = 4- 2 = 2, 3-1 = 2, 5-4 = 1, 2-1 = 1
For columns 2-1 = 1, 2-1 = 1
Max penalty is 2 (1st row)
8
3 3 1 8(2)
5 4 7 7(1)
1 6 2 14(1)
2 9 18
(2) (1) (1)
Max penalty is in row 2

5 4 7 7(1)

1 6 2 14(1)
2 (4) 9(2) 10(5)

Maximum penalty is 5

5 4 7 7(1)
10
1 6 2 14(1)
2 9 10

81
5
4 7(1)
2
1 6 4(5)
2(4) 9(2)

7
4 7
6 2
9(2)

2
6 2
9

Max penalty is 5
Total cost = (5 x2) + (8x1) +(7 x4) +(2 x ) (2 x6) + (10x2) = Rs. 80

Problem 4
Solve the transportation problem(UNI Feb 2015)
Distribution centre

Origin 1 2 3 4 Availability
I 21 16 25 13 11
II 17 18 14 23 13
III 32 27 18 41 19
Req 6 10 12 15 43
Solution

Availability = 43
Requirement = 43
We will find the initial basic feasible solution using VAM
1 2 3 4
21 16 25 13 11(3)
17 18 14 23 13(3)
32 27 18 41 19(9)
6 10 12 15
(4) (2) (4) (10)

82
Max penalty is 10 (4th column)
11
21 16 25 13 11

17 18 14 23 13

32 27 18 41 19

6 10 12 15

17 18 14 23 13(3)
32 27 18 41 19(9)
6 10 12 4
(15) (9) (4) (18)
Max penalty is 4th column
4
23
17 18 14 13(3)

32 27 18 41 19(9)

6 10 12 4
Availability

17 18 14 9

32 27 18 19
6 10 12

6
17 18 14 9(3)
32 27 18 19(9)
6 10 12
(15) (9) (4)
Maximum penalty is 1st column
3
18 14 3(4)
27 18 19(9)
10(9) 12(4)

7 12 18
27 19(9)

7 12

83
Initial basic feasible solution
(11x 13) + (6 x 7) + (3 x 18) + (4x23) + (7 x 27) + (12 x 8) = Rs. 796

Problem 5 (Feb 2016) UNI

Solve the transportation problem

Origin Distribution centre

Origin A B C D Availability
P 11 13 17 14 250
Q 16 18 14 10 300
R 21 24 13 10 400
Req 200 225 275 250

Solution

Availability = 250 + 300 + 400 = 950

Requirement = 200 + 225 + 275 + 250 = 950Solve by VAM method

11 13 17 14 250(2)
16 18 14 10 300(4)
21 24 13 10 400(3)
200 225 275 250
(5) (5) (1) (4)

Max penalty 5 (1st row)


200
11 13 17 14 250
16 18 14 10 300
21 24 13 10 400
200 225 275 250

13 17 14 50
18 14 10 300
24 13 10 400
225 275 250

13 17 14 50(1)
18 14 10 300(4)
24 13 10 400(3)

84
225 275 250
(5) (1) (4)

50
13 17 14 50(1)
18 14 10 300(4)
18 14 10 300(4)
24 13 10 400(2)
225 275 250
(5) (1) (4)

17518
14 10 300(4)
24 13 10 400(3)
175 275 250
(6) (1) (0)

125 125(4)
14 10
13 10 400(3)
275 250
(1) (0)

150 250 400(3)


13 10

275 125

150
13

200 50
11 13 17 14 250
175 125
16 18 14 10 300
275
21 24 13 10 400
200 225 275 250
Total transportation cost = (1 x 200) + (13 x 50) + (18 x 175) + (10x 125) + (13 x 275) +
(10 x 125)
200 + 650 + 3150 + 1250 + 3575 + 1250 = Rs. 7240
Optimal Solution

85
By applying Vogelsmethod or lowest cost entry method or North west corner rule, an initial
basic feasible solution can be obtained. Next step is to examine whether the solution can be
improved. For the we have to conduct the test of optimality by modified distribution method
(MODI’s method)

MODIs Method

Step 1 : When the initial basic feasible solution obtained, some cells are occupied and other
unoccupied. No of occupied cells is (m+n-1) let cij be the cost of cell (i,j)
Then we determine (m+n) number called Ui&Vj values by framing (m+n) numbers called
Ui&Vj values by framing (m+n-1) equations of the form Ui+Vj = Cij correspond to each of
the occupied cell.
For solving the equations, we take one of Ui and Vj values as zero (since no of unknown is
one more than the number of equation)
Step 2 : Then we calculate cell evaluation known as dijvalues for unoccupied cells by the
formula dj = Cj –(Ui+Vj)
Step 3 : If all the dj values are positive, the solution is optimal and unique. If at least one of
them is zero, and other positive, the solution is optimal but alternative solution exists. If at
least dij is negative, the solution is not optimal.
Step 4 : If the solution is not optimal, make reallocation. Give maximum allocation to the cell
for which dij is negative, making one of the occupied cells empty
Then steps 1 to 4 will be repeated until solution becomes optimal.
Problem 6 (UNI NOV 2012- 5 Marks)

Solve the transportation model by MODIS method

Supply
21 16 25 13
11
17 18 14 23 13
32 27 18 41 19
6 10 12 15
Solution
First we have to find the initial basic feasible solution by vogels approximation method
Initial basic feasible solution

W1 V1 V2 V3 V4 Supply
11
W2 21 16 25 13 11
6 3 4
W3 17 18 14 23 13
7 12
W4 32 27 18 41 19

6 10 12 15

86
Minimum transportation cost

= (13x 11) + (17 x6) + (18 x3) + (23 x3) + (23 x 4) + (27 x 7) + (18 x12) = Rs. 796

To find the optimality using MODIs Method

Total no of allocation = 6

(m + n-1) = (4 + 3-1) = 6

Since total no of allocation = (m +n-1) the allocation are independent,non degenerate feasible
solution is established.

For the occupied cells

U1+ v4 = 13

U2 + V1 = 17

U2 + V2 = 18

U2 + V4 = 23

U3 + V2 = 27

U3 + V3 = 18

Put U2 =0 V1= 17 U2 = 18

U3 + 18 = 27 U3 = 9

9 + V3 = 18 V3 = 18-9 = 9

U1 + 23 = 13

U1 = -10

For unoccupied cells,

dij= Cij – (u1+vj)

Cij

21 16 25 X
X X 14 X
32 X X 41
Ui+VJ)

87
7 8 -1 X
X X 9 X
26 X X 32

Dij = Cij – (u1 + Vj)

14 8 26 x
X X 5 X
6 X X 9

Since all dijs are positive, the solution is optimal

Hence the optimal solution is the initial basic feasible solution

Problem 7

Solve the following transportation problem by MODIS method

2 7 4 Supply 5
3 3 1 8
5 4 7 7
Demand 1 6 2 14

Solution

Solve by vogels approximation method

Initial basic feasible solution obtained

2 7 4 Supply 5
3 3 1 8
5 4 7 7
1 6 2 14
Demand 7 9 18
m+n-1= 4=3-1=6

Total no of allocation =6

To test the optimality dij = (Cij – (Ui + Vj)

For occupied cells

U1 + V1 = 2

U2 + V3 = 1

88
U3 + V2 = 4
U4 + V2 = 6
U4 + V3 = 2
U4 occurs max no of times u4 = 0
V1 = 1, V2 = 6, V3 = 2
U2 + 2 = 1 U2 = -1
U3 = 4-6 = -2
U3 + V2 = 4
U3 + 6 = 4
U1+ 1 = 2, u1 =1
For unoccupied cells Cy – (Uc+ Vj)
Cij
X 7 4
3 3 X
5 X 7
X X x
Ui+Vj

X 1+6=7 1+2 = 3
-1+1=0 -1+6=5 X
-2+1 = -1 X -2+2=0
Dij= Cij – (ui1+ Vj)

X 0 4-3=1
3-0=3 3-5=-2 X
5+1-6 X 7-0=7
X X x
Since one of the dij values is negative the solution is not optimal. Make reallocation. Give
max allocation to the cell with negative dij occurs (2,2)

Transfer 2 from 4throw ,2ndcoloumnto 2nd row 2nd column

5 2 7 4 5
2 6
3 3 1 8
7 4
5 7 7
2 12
1 6 2 14
7 9 18

To test optimality again


U1 + V1 = 2
U2+ V2 = 3
U2 + V3 = 1
U3 + V2 = 4

89
U4 + V1 = 1
U4+ V3 = 2
Put U4 = 0 V3 = 2 V1 = 1
Put U1 = 1 U2 = -1 V2 = 4 U3 = 0

Cij Ui + Vj
X 7 4 X 4+1=5 2+1=3
3 X X 1-1=0 X X
5 X 7 0+1=1 X 0+2 =2
X 6 X X 0+4 = 4 x

dij= Cij – Ui + Vj

X 2 1
3 X X
4 X 5
X 2 x
Now all dijsare positive. Solution is optimal

Total transportation = (5 x 2) + (2 x3) + (6 x 1) + (7x 4) = 76

Maximisation in transportation problem

A transportation problem in which objective is to maximize can be solved by converting the


given maximization problem to a minimization problem .For this, select the highest value
and subtract all other values from this highest values. Then the given problem becomes
minimization problem.

Problem 8 (UNI NOV 2013)

Solve the following transportation problem to maximize the profit

Source A B C D Supply
1 15 1 42 33 23
2 80 42 26 81 44
3 90 40 66 60 33
Demand 23 31 16 30 100
Solution

This is a maximization case problem. Highest value is 90. Subtracting all the elements by 90
to convert problem into minimization problem

1 75 89 48 57 23
2 10 48 64 9 44
3 0 50 24 30 33

90
Demand 23 31 16 30 100

Finding IBF by VAM

75 89 48 57 23 (9)
10 48 64 9 44 (1)
0 50 24 30 33 (24)
23 31 16 30 100
(10) (2) (24) (21)

89 48 57 Supply 23
48 64 9 44
50 24 30 10
Demand 31 16 30

89 48 57 23(9)
48 64 30 9 44(39)
50 24 30 10(6)
31 16 30
(2) (24) (31)
Supply

89 48 23
48 64 14
50 24 10
Demand

supply
89 16 48 23(41)

48 64 14(16)
50 24 10(26)
31 16
(2) (24)

89 7
48 14
50 10
31

91
Supply

7 16
75 89 48 57 23
14 30
10 48 64 9 44
23 10
0 50 24 30 33
Demand 23 31 16 30 100

Maximum Profit = (89 x 7) + (48 + 14) + (10 x 50) + (23 x 0) + (48 x 1) + (9 x 30) = Rs.
2833

Unbalanced transportation problem

available amounts is not equal to sum of all requirements in destination together ∑ $ ≠ ∑ '
A transportation problem is said to be unbalanced transportation problem if the sum of all

Transportation problems are unbalanced when

1. Supply is less than demand : Lost sales &loosing customers. Introduce dummy origin
2. Supply is greater than demand
Profit is reduced by the loss caused by unsold quantity. Dummy destinations are
introduced to absorb the excess capacity of origin
An unbalanced transportation problem can be converted to a balanced transportation
problem by introducing a fictious source of destination which will provide the surplus
or demand. The cost of transporting unit from the fictiouss (source or destination) is
taken to be zero
After converting the unbalanced transport problem into a balanced transportation
problem by a dummy source or destination, it can be solved by Northwest corner
rule/ least cost method / VAM
Degeneracy in transportation problem
In a transportation problem, degeneracy occurs ,whenever the number of individual
allocation less than (m+n-1) where m & n are respectively number of rows and
columns of the transportationmatrix.
Degeneracy in the transportation problem are developed in 2 ways
1.The basic feasible solution may be degenerated from the initial stage onwards.
2. They may become degenerate at any immediate stage
In such cases, we allocate a small number ∆ which is equal to zero to one or more
empty cells so that the total no of allocations is (m+n-1)
Problem 9
Determine the optimal transportation plan from the following table giving the plant to
make shipping costs and quantities required at each market and available at each plant
Plant W1 W2 W3 W4 Available
F1 11 20 7 8 50

92
F2 21 16 10 12 40
F3 8 12 18 9 70
30 25 35 40
Solution
Here the total requirement of the market = 30+ 25+ 35 + 40 = 130
The total availability at the plants = 50 + 40 + 70 = 160 .
This is an unbalanced assignment problem since the total availability is 30 more than
the total requirements
So we convert this problem to a balance one by introducing a fictions market W5 with
requirement 30 such that the cost of transportation from plants to the market W5 are
zero.
Balanced transportation problem is
Plant W1 W2 W3 W4 W5 Available
F1 11 20 7 8 0 40
F2 21 16 10 12 0 70
F3 8 12 18 9 0
30 25 35 40 30 160

93
7 8 50 (1)

10 12 10 (2)

18 15
9 15 (9)

35 40
(3) (1)

7 25 8 50(1)

10 12 10(2)

35 25
(3) (4)

25 7 25
10 10

35

Plant W1 W2 W3 W4 W5 Available

25
F1 11 20 25 7 8 0 50

10 30
F2 21 16 10 12 0 40

30 25 15
F3 8 12 18 9 0 70

30 25 25 40 30 160

Total transportation cost


= (25 x 7) + (25 x 8) + (10 x 10) + (30 x 0) + (30 x 8) + (25x 12) + (15 x 9) = Rs.
1150
Problem 10
A manufacturer wants to ship 8 loads of his product as shown below. The matrix
gives the mileage from origin O to Destination D

A B C Available
X 50 30 220 1

94
90 45 170 3
Z 250 200 50 4
Requirement 4 2 2

Shipping costs Rs. 10 per load mile what shipping should be used?
Solution
By vogel method, we get the following initial basic feasible solution
A B C
X 1 50 30 220 1
y 3 90 45 170 3
Z 250 2
200 2
50 4
4 2 2

Since the total allocations is 4 which is one less than the (m+n-1)=5,the solution is
degenerate
Now to resolve this degeneracy,y we allow a very small amount ∆ to the cell (2,2)
getting 5 allocations at independent position.
New solution is
A B C
X 1 50 30 220 1
y 3 90 ∆
45 170 3
Z 250 2
200 2
50 4
4 2 2
Testing for optimality
For occupied cells
U1+ V1 = 50
U2 + V1 = 90
U2 + V2 = 45
U3 + V2 = 200
U3 + V3 = 500
U3 = 0 U1 = -195
U2 = -155
V1 = 245 v2 = 200
V3 = 50
Cj

X 30 220
X X 170
250 X x
U1+ Vj

95
X 5 -145
X X -105
245 X x
Dij=Ui+Vj

X 25 365
X X 225
X X x

Since no dij is negative, the solution is optimal

Optimal solution is

X-A 1 unit

Z – B- 2 units

Y to A – 3 units

Z – C (2 units)

Cost of shipping the minimum cost

∆ x 45) + (200 x 2) + (2 x 50) =Rs. 8200


Rs 10(1 x 50) + (3x 90) + (∆

Activity Questions

Problem 11Solve the following Transportation Problem by MODIs method

W1 W2 W3 W4 Capacity
F1 19 30 50 10 7
F2 70 30 40 60 9
F3 40 8 70 20 18
Req. 5 8 7 14

(Ans:Total cost = 743 Rs)

Problem 12

Solve the following transportation problem to maximize profit by MODIs method

A B C D Availability
1 40 25 22 33 100
2 44 35 30 30 30
3 35 35 28 30 70
Req. 40 20 60 30

96
(Max profit = 5130 Rs)

13.Acompany has 3 plants p1, p2 p3 each producing 50, 100, 150 units of a similar
product. There are 5 ware houses w1, w2, w3, w4& w5 having demand of 100, 70, 50, 40
& 40 units respectively. Cost of selling one unit from various plants to ware houses differ
as given by cost matrix below

Determine the transportation schedule so that cost is minimized

W1 W2 W3 W4 W5 ai
P1 20 28 32 55 70 50
P2 48 36 40 44 25 100
P3 35 55 22 45 48 150
dj 100 70 50 40 40
(Ans = 9240)

Problem 14

Solve the following TP

A B C D Supply
1 11 20 7 8 50
2 21 16 20 12 40
3 8 12 8 9 70
Demand 30 12 8 9 70
(AnsRs. 1130)

Difference between transportation problem and assignment problem

Transportation Problem Assignment Problem

1 Not of sources or number of Since assignment is done on one no


destination need not be equal Hence basis,the number of source and destination
the cost matrix is not necessary a a equal. Hence the cost matrix must be a
square matrix square matrix
2 The problem is balanced if the total The problem is unbalanced if the cost
supply and demand are nor equal matrix is not a square matrix
3 Capacity and requirement value is Capacity and requirement value is exactly
equal to a1 & b1 for pth source and jth one
destination

97
Unit III

NETWORK MODELS

Project : A project consist of interrelated activities which are to be executed in a certain


order before the entire task is completed. The activities are inter related in a logical sequence
known as Precedence relationship

The work on a project cannot be started until all its immediate preceding activities that
involve planning, procuring processing the inputs etc are completed.

Eg. Excavation for foundation should be done only after the site preparation; white washing
should be done after plastering

Some of the typical project are as follows.

• Construction of a house
• Commissioning of factory
• Construction of ship
• New product launching
• Research to develop a new technology

Project management is generally applied for constructing items of public utilities, large
industrial projects, organizing mega events etc. Project management is considered to be
an important area in production scheduling mainly because many of the industrial
activities can be viewed as project management problem.

Eg. Fabrication of boilers , construction of railway coaches

Network : Network is a combination of activities and events of a project.

The problem of networks to find a certain course of action which minimize time, cost or
distance in performances of various activities.

Network problems may occur in many areas such as production, distribution,


transportation, communication and activities of commercial undertaking

Objectives of network analysis

1. Minimization of total cost of a project


2. Minimization of total time of a project
3. Minimization of cost of project for a given time
4. Minimization of idle resource
5. Planning scheduling and controlling projects

Network techniques

98
Network techniques is a major advancement in management science. This technique is
based on the basic characteristic of all projects.

Different network techniques are PERT, CPM etc. The two network technique help
manages to plan, schedule, monitor and control large and complex projects.

Uses of network techniques

1. It helps the management in planning the completed projects controlling work & plan
and updation of plan
2. It helps the management in reaching the goal with minimum time and least cost and
also in forecasting probable project duration & associated cost
3. Network techniques have resulted in better management control, better utilization of
resource and better decision making .
4. Network techniques have resulted in saving of time or early completion of project.

Application of network techniques

1. Construction of buildings and factories


2. Administration
3. Manufacturing
4. R&D
5. Marketing

Networking diagram
The project can be broken into a number of distinct and well defined jobs called activities.
The beginning or end of each activities constitute an event of project
A graph drawn connecting various activities and events of a project is network diagram.
Each event is represented on it by a circle and each activity by arrow.
The arrow denotes the sequence of activities.

Two types of network diagram

1. Event oriented diagram : Emphasis given on the events of project also referred as
PERT. Events are first selected and events in such network falls in a logical sequence.

2. Activity oriented diagram : Emphasis is given an activities of the project. Activities


are arranged in logical order

Eg.CPM

Phases of project management

Project management has 3 phases planning, scheduling and controlling

The planning phase has following steps

99
1. Dividing the project into distinct activities
2. Estimating the time requirement for each activity
3. Establishing the precedence relationship among activities
4. Construction of arrow diagram (network)
Scheduling phase determined the start and end time of each and every activities which

can be summarized in the form of a time chart/ Gantt chart

The control phase uses the arrow diagram and time chart for continuous monitoring and

progress reporting.

Activity : An activity is a tasks associated with project

Eg. Lay pipe line

Here A is an activity A
Number 2 initial node 2 3
Number 3 is terminal node

Start and terminal actvities

Activities which have no predecessors are called start activities

Dummy activities : Certain activities which neither consumers time non resources but are
used simply to represent a connection or a link between the events are known as dummy
activities . It is shown in the network by a dotted line.

The purpose of introducing dummy activity is

1. To maintain the uniqueness in the numbering system as every activity may have
distinct set of events which activity can be identified.
2. To maintain proper logic in the network

B D
A Dummy

Rules for network construction

1. The Starting event and ending event of an activity are called tail event and head event
respectively
2. The network should have a unique starting mode (tail event)
3. The network should have a unique completion node (Head event)
4. No activity should be represented by more than one arc in the network

100
5. No two activities should have the same starting node and same ending
ending node
6. Dummy activity is an imaginary activity indicating precedence relation shop only.
Duration of dummy activity is zero.
Problem 1 Draw a network diagram to the following activities
Activity Time duration
1-2 2
1-3 4
1-4 3
2-5 1
3-5 6
4-6 5
5-6 7
Network diagram

Problem 2 Draw a network diagram compressing of activities A,B,C …. and K such


that the following relation are satisfied.
1. A, B, C are the first activities of project and can start simultaneously
2. A precedes D
3. B precedes E & F
4. E precedes H & I
5. D & I precedes J
6. J precedes K
7. H., G, K are the terminal activities of the project.

101
Construct the network diagram

1. A is the first operation


2. B & C can be performed in parallel and immediately successor t o A
3. D, E & F follows B
4. G follows E
5. H follows D, but it cannot start until E is complete
6. I & J succeeds G
7. F & J precedes K
8. H & I precedes L
9. M succeeds L & K
10. Last operation N succeeds M & C

Earliest event time T(E)

The earliest occurrence time or earliest event time (TE) is the earliest at which an event can
occur. Earliest occurrence of event 2 is denoted as E2.
E2

Latest event time (TL) : The latest allowable occurrence time or latest time by which can
event must occur
occur to keep the project on schedule. Latest occurrence of an event 2 is
denoted by L2.

1. Earliest start time (EST) : it is the earliest time which an activity can commence.
EST of activity 2-3
2 isE3
2. Earliest finish time (EFT) Earliest finish time of a activity is defined as the earliest
time by which an activity can be finished.
EFT = EST + Activity duration
3. Latest finish time (LFT)
Latest finish time for an activity is the latest time by which an activity can be
finished without delaying the completion of project.
pro
LFT of an activity =Latest expected time of head event
For activity 2-3
2 , LFT = L2
4. Latest start time (LST)

102
Latest start time of an activity is the latest time by which an activity can be started
without delaying the completion of project

LST = LFT – Activity deduction

Slack and float

Slack is a term associated with events . It denotes is flexibility range within which an
event can occurs.

Slack of event 2 = L2 – E2 – LFT – EST

Float ; Float is associated with activity time Float denotes the range within which the
activity starts time or finish time may fluctuate without affecting the completion of the
project

Types of float

1. Total float
2. Free float
3. Independent float
4. Interferring float

Total float

1. Total float is the time spent by which starting or finishing an activity can be delayed
without delaying the completion of the project

Total float = LFT – EFT or LST – EST

2.Free float : Free float is that portion of positive total float that can be used by an
activity without delaying any succeeding activity . The concept of free float is based on
the probability that all activities start as early as possible.

Free float = EST of successor – EFT of present activity

3 Independent float

It is defined as the excess of minimum available time over the required activity duration

Independent float is the amount of an activity could be delayed if preceding activity finish
at their latest and the subsequent activity start at their earliest

Independent float = EST of an subsequent activity – LFT of preceeding activity –


Duration

103
4 Interfering float =Total float – Free of float

Uses of float

1. To solve resource leveling and resource allocation problem


2. Floats gives flexibility in representing some activity to smoothen the level of
resources or allocate the limited resources
3. Floats reduces the cost of project.

Critical path

Critical path is the path having longest duration . The activities on the critical path are
shown by dark of double line arrows.

Critical path in the network diagram is a path on which all the activities have zero float.

All activities on the critical path are zero float activities.

Critical activity : critical activity is the activity lying in the critical path.

Critical path method (CPM)

CPM is a network technique consist of

1. Planning the sequence of activities to be performed in a network


2. Scheduling the time and resources to various operation
3. Controlling the performance so that they are not deviating from the plan

CPM is generally used for repetitive jobs

Features of CPM

1. CPM is a deterministic problem


2. It is used when duration of each activity or time is known exactly
3. It is used for repetitive projects
4. CPM can be effectively in production planning, road systems, traffic schedule and
communication network
5. CPM emphasis the relationship between applying more resources to shorten the
duration of given job in a project and increased cost of these addition resources.

Steps involved in CPM

1. List out the activities and draw a network


2. Find the earliest event time (Tg) and latest event time EL of each event shown in the
network diagram
3. Calculate EST, EFT, LST & LFT for each activity
104
4. Determine the float
flo for each activity
5. Determine the critical activities having zero floats
6. Draw the double lines in network diagram passing through critical activities. The
double line shows the critical path
7. Calculate the total project duration which is the sum of duration
duration of critical activities.

Problem 1
A project schedule has the following characteristics

Activity Time Activity Time


1-2 4 5-6 4
1-3 4 6-7 8
2-4 1 6-8 1
3-4 1 7-8 2
3-5 6 8-10 5
4-9 5 9-10 7

a. Construct the network diagram


b. Compute TE, TL for each event
c. Find EST, LST, EFT, LFT values for all activities
d. Find the critical path and project duration

Calculate TE and TL values


E1 = 0
E2 = E1 + project duration 1-2
1 2 = 0+4 = 4
E3 = E1 + Duration 1-3
1 = 0+1 = 1
E4 = max (E2 + duration 2-4
2 4 and E3 + duration 3-4)
E4 = max (4+1, 1+1) max (5,2) = 5
E5 = E3 + Duration (3-5)
(3 1+6 = 7
E6 = E5 + 4 = 7+4 = 11
E7 = E3 + 8 = 7+ 8 = 15
E8 = Max (E7 + duration 7-8,
7 8, E6 + Duration 6-8)
6
= max (15 + 2, 11+1) = 17
E9 = E4 + 5 = 5+5 = 10

105
E10 = Max [E8+5, E9+7] = max (23,
(23, 17) = 22

Computation of TL values
L10 = L10 = 22
L9 = L10 - Duration (9,10) = 22-77 = 15
L8 = L10 – Duration (2,8) = 22-5 = 17
L7 = L8 – Duration (7,8) = 17-2 = 15
L6 = L8 – 1 = 17-1 = 16
L5 = Min (L8-4,
(L8 L7 – 8) = Min (12,7) = 7
L4 = L9 – 5 = 15-5 = 10
L3 = Min (L4-1,
(L4 L5-6) 6) = Min (9,1) = 1
L2 = L2 -1 = 10-1 = 9
L1 = Min (L2 – 4, L3-1)
1) = Min (5,0)=0
Finish Float
Activity Finish
Activity Start EST EFT = Start LST LST –
time LFT
EST EST
1-2 4 0 4 5 9 5
1-3 1 0 1 0 1 0
2-4 1 4 5 9 10 5
3-4 1 1 2 9 10 8
3-5 6 1 7 1 7 0
4-9 5 5 10 10 15 5
5-6 4 7 11 12 16 5
5-7 8 7 15 7 15 0
6-8 1 11 12 16 17 5
7-8 2 15 17 15 17 0
8-10 5 17 22 17 22 0
9-10 7 10 17 15 22 5

(11,3) (3,5) (5,7) (8,100 are critical activities with float zero

Critical path = 1-3-5-7-8-10

Project duration = 1+6+8+2+5 = 22

106
Problem 2 SDE KU UNI April 2014

The utility data for a network is given below. Determine the total float, independent

float and identify the critical path

Activity 0-1 1-2 1-3 2-4 2-5 3-4 3-6 4-7 5-7 6
Duration 2 8 10 6 3 3 7 5 2 8

Solution

NETWORK DIAGRAM

Float Independent
Activity Start Finish Start Finish
Activity LST - float
time EST EFT = LST LFT
EST
0-1 2 0 2 0 2 0 -
1-2 8 2 10 8 16 6 -
1-3 10 2 12 2 12 0 -
2-4 6 10 16 17 23 7 0
2-5 3 10 13 22 25 12 0
3-4 3 12 15 20 23 8 0
3-6 7 12 19 12 19 0 0
4-7 5 16 21 22 27 6 0
5-7 2 13 15 25 27 12 0
6-7 8 19 27 19 27 0 0

E0 = 0
E1 = E0 + 2 = 0+2 = 2
E2 = E1+ 8 = 2+8 = 10
E3 = E1 +10 = 2+10 = 12
E4 = max (E2+6, E3 + 3) = max (16, 15) = 16
E5 = E2+ 3 = 10+3 = 13
E6 = E3 + 7 = 12+ 7 = 19
E7 = max (13+2, 16+5, 19+8) = 27
L7 = 27
107
L6 = L7 – 8 = 27-8 = 19
L5 = L7 – 2 = 27-2 = 25
L4 = L7 – 5 = 27-5 = 22
L3 = Min (19-7,
(19 22-3) = 12
L2 = Min (25-3,
(25 22-6) = 16
L1 = Min (16-8,
(16 12-10) = 2
L0 = L1 – 2 = 0
Critical
tical activity = 0-1
0 -3-6-7

Project competition time = 2+10+ 7 + 8 = 27 days

CPM is represented in the network diagram

Independent float = EST of subsequent activity – LST for preceeding activity –


Duration
Problem 3 KU SDE UNI 2013 20 Marks
A small maintenance
maintenance projects consist of the following jobs whose precedence
relationship are given below.
Jobs 1-2 1-3 2-3 2-5 3-4 3-6 4-5 4-6 5-6 6-7
Duration days 15 15 3 5 8 12 1 14 3 14

a. Draw arrow diagram representing the project


b. Find the total float of each activity
c. Find the critical path and total project duration

108
Solution

Project finish Project


Project Float
Project start time Finish
Activity Duration Start time LST –
time EFT LST=LFT- Time
EST EST
Duration LFT
1-2 15 0 15 0 15 0
1-3 15 0 15 3 18 3
2-3 3 15 18 15 18 0
2-5 5 15 20 32 37 17
3-4 8 18 26 18 26 0
3-6 12 18 30 28 40 10
4-5 1 26 27 36 37 10
4-6 14 26 40 26 40 0
5-6 13 27 40 37 40 0
6-7 14 40 54 40 54 0

Calculating El values and TL values

E2 = E1+ 15 = 0+15 = 15

E3 = Max (0+15, 15+3 )= 18

E4 = E3 + 8 = 18+8 = 26

E5 = max (15+5, 26+1) = 27

E6 = Max (18+12, 26+14, 27+3) = 40

E7 = 40+14 = 54

L7 = 54

109
L6 = L7 – 14 = 54-14 = 40

L5 = L6 – 3 = 37

L4 = Min (40-4,
(40 37-1) = 26

L3== Min (26-8,


(26 40-12) = 26

L2 = Min (18-3,
(18 37-5) = 15

L1 = Min (18-15,
( 15-15) = 0

Critical path = 1-2-3-4-6-7


1

Total duration = 15+3+8+14+14 = 54 days

Problem 4 SDE KU NOV 2012-10


2012 10 Marks

Complete the earliest start, earliest finish, latest start and latest finish of each activity
of the project given below
Activity 1-2 1-3 2-4 2-5 3-4 4-5
Duration days 8 6 10 2 5 3

110
Network diagram

To find TE and TL values

E1 = 0

E2 = E1 + 8 = 0+8 = 8

E3 = E1 + 6 = 8+6 = 14

E4 = Max (E2 + 10, E3 + 5) = 19

E5 = max (E2 +2, E4 +3) = 22

To find TL values

L5 = 22

L4 = L5 – 3 = 19

L3 = L4 -3 = 16

L2 = Min (E5 -3, L4 – 10) = Min (22-3,


3, 19-10)
19 =9

L1 = Min (E2-8,
(E2 E3 – 6) = 1
Finish EFT Start Finish Float LST
Activity Duration Start EST
= EST LST LFT – EST
1-2 8 0 8 1 9 1
1-3 6 0 6 10 16 10
2-4 10 18 9 9 19 1
2-5 2 8 10 20 22 12
3-4 5 14 19 14 19 0
3-5 3 19 22 19 22 0

EFT = EST + duration Float = LST – EST


LST = LFT – Duration

111
Critical activity = activity having zero float = 3-4-5
3 = 5+3 = 8
Critical path is represented in

Question No 5

A project has five activities. Draw the network diagram

Activity Preceeding activity


A -
B -
C A
D A
E B,C

Network diagram for the project is shown

Problem No 6

KU SDE NOV 2011 A project has the following activities and duration

Activity Time days Preceeding activity


A 1 -
B 2 -
C 2 A,B
D 4 B,C
E 1 C
F 1 C
G 4 D
H 8 G,E,F

112
a. What is the minimum completion period of work

b. Draw the project network and indicate the critical path

Net work diagram is shown below

Calculation of EL &TL values

E1= 0

E2 = 0+1 = 1

E3 = E2 +2 = 3

E4 = Max (E2+2, E3+ 2) = 5

E5 = Max (E3 + 4, E4 + 1) = 7

E6 = max (E5 +8, E4 +4) = 15

TL values are shown below

L6 = 15

L5 = L6 -8= 15-8 = 7

L4 = Min (L6 – 4, L5-1) = 6

L3 = Min (L5-4, L4-1) = 3

L2= Min (L4-2 ,L3-2)=1

L1 = L2 -1 = 0

113
Critical Path Diagram

Critical Path-1-2-3-5-6

Maximum duration =1=2+4+8=15

Start Finish EFT Float LST


Activity Duration Start LST Finish LFT
EST = EST - EST
1-2 1 0 1 0 1 0
2-3 2 1 3 1 3 0
3-4 2 3 5 4 6 1
2-4 2 1 3 4 6 3
3-5 4 3 7 3 7 0
4-5 1 5 6 6 7 1
4-6 4 5 9 11 15 6
5-6 8 7 15 7 15 0

Question No 7 KU SDE Dec 20018

An architect has been awarded a contract to prepare plan for an urban renewal project. The

job consists of the following activities and their estimated time.

Activity Description Immediate Predecessor Time in Hours


A Prepare preliminary sketch 2
B Outline specification 1
C Prepare drawing A 3
D Write specification A,B 2
E Run off prints C,D 1
F Have specification B, D 3
G Assemble bid packing E, F 1

a. Draw an arrow diagram for the project

114
b. Determine the critical path and calculate the total float and free float of each activity

Calculation of EL &TL Values


E1=0
E2= El + 2 = 2
E3 = E1 + 1 = 3
E4 = max (E1 +2, E2 +3, E3 + 1) = 5
E5 = max (E1+ 3, E4 + 1) = 6
L5 = 6
L4 = L5 -1 = 5
L2 = L4 -3 = 2
L1 = Min (L3-1,
1, L2 – 2, L4-2, L5 -3) 0
= Min (3,0,3,3) = 0 Earliest time Latest time
Float Free
Start Finish Start Finish
Activity Duration LST - float
EST EFT = LST LFT
EST
1-2 2 0 2 0 0 0 -2(0)
1-3 1 0 1 3 3 3 1
2-4 3 2 5 2 0 0 -5(0)
1-4 2 0 2 3 3 3 1
3-4 1 3 4 4 1 1 -4(0)
1-5 3 0 3 3 3 3 2
4-5 1 5 6 5 0 0

For negative free float, values can be taken as zero

115
Total float = LST – EST or LFT – LST

Free float = EST of successor – EFT of present activity

Critical path 1-2-4-5


1 = 2+3+1 = 6 days

Critical activities

1. Prepare preliminary sketches


2. Prepare drawings
3. Assemble bid package

Question No.8 SDE UNI FEB 2016

Draw the network and determine the critical path for the given data

Jobs 1-2 1-3 2-4 3-4 3-5 4-5 4-6 5-6


Duration days 6 5 10 3 4 6 2 9

Find the total float, free float and independent float of each activity

116
Calculation

E1= 0

E2 = E1+ 6 = 0+6 =6

E3 = E1+ 5 = 0+5

E5 = max (E3 + 4, E4 + 6)

= Max (9,22) = 22

E6 = max (E4 + 2, E5 + 9)

= Max (18, 31) = 31

L6 = 31

L5 = L4-9 = 31-9 = 22

L4 = (L5-6, L6-9)

= Min (16, 22) = 16

L3 = Min (L5- 4, L4-3)

+= Min (18,13)= 13

L2 = L4 – 10 = 16-10 = 6

L1 = Min (L2-6), L3-5)= = Min (0,8) = 0


Float Free float
Start Finish Start Finish
Activity Duration LST -
EST EFT = LST LFT
EST
1-2 6 0 6 0 6 0 -6(0)
1-3 5 0 5 8 13 8 1
2-4 10 6 16 6 16 0 5-6-11(0)
3-4 3 5 8 13 16 8 5-8=-3(0)
3-5 4 5 9 18 22 13 7
4-5 6 16 22 16 22 1 -6(0)
4-6 2 16 18 29 31 13 41
5-6 9 22 31 22 31 0 0

Critical activity 1-2, 2-4, 4-5, 5-6

Total project duration 6 + 10+ 6+9 = 31

Free float = EST of successor – EFT of present activity

117
Independent float = EST of subsequent activity –LFT of preceding activity – Duration

Activity Questions

9.The following refers a project network


Activity A B C D E F G H I J
Precedor A A A A E D G,F C, H B
Duration in days 1 4 2 3 2 3 2 1 3 2

a.Draw the network

b. Determine the critical path of project completion time

Question No 10 A project has the following schedule


Activity 1-2 1-3 1-4 2-5 3-6 3-7 4-6 5-8 6-9 7-8 8-9
Duration in days 2 2 1 4 8 5 3 1 5 4 3

Construct network and compute

a.EST, LST , EFT & LFT of the activities

b. Total float for each activity

c. critical path & duration

Critical path = 1-3-6 -9

Duration of path 2+8+5 = 15

Free float of (2-3) = E3 – E2 – Duration (2-3) = 15

Independent float of (2,3) = (E3- L2 – Duration 2,3)

Project Evaluation and review technique(PERT)

PERT was developed in 1950s

PERT is a management technique proposed for those project or operations which are of non
repetitive nature for these projects in which precise time determination for various activities
cannot be made

Assumption in PERT

1. Activity duration are independent ie time required for one activity has no relation with
another activities
2. PERT is used in those places where a project cannot be easily defined in terms of time
and resources.
3. Time duration in each activity is not a constant and hence un certain
118
Time estimates in PERT

1. Optimistic time estimate (to)

This is the shortest possible time in which an activity can be completed under ideal
conditions. This particular time estimate represents the time in which the activity or
job can be completed if everything goes well with no problem or adverse condition

2. Pessimistic time estimate (tp)

It is the maximum time that would be required to complete the activity. The particular
time represents the time it takes to complete a particular activity if everything went wrong
abnormal situation exists.

3. Most likely time estimate (tm) It is the time which the activity will take most
frequently performed a number of times. This time estimate a reflects a situation

()*+,-*,.
where condition are normal expected time based on 3 estimates.

/
Average time Te =

Steps involved in PERT calculation

1. Identify the events and activities and prepare a suitable network for the given problem
2. Events are numbered in ascending order from left to right
3. Obtain the various time estimate for each activity . They are most likely tm,
pessimistic tp and optimistic to estimate

,)*+,-*,.
/
Calculate expected time te =

4. Compute the float associated with each activity

The activity with zero float are critical activities . Determine critical path through the
critical activities. Calculate the TE & TL for each event

5. Find the total expected duration time te by adding the time estimates for various
activities on critical path

Application / Uses of PERT

1. Research & development


2. Prototype production
3. Irrigation projects
4. Launching of space craft
5. Project planning & control

CPM & PERT are both network analysis techniques used in planning and control of
projects. Both the techniques make use of network diagram for management of projects.

119
Difference between PERT & CPM
CPM PERT
1. CPM uses only single time estimate PERT uses multitime estimate for activity
for activity and does not consider duration (consider 3 time estimate
uncertainty. It is a determines to a uncertainity in time duration stochastic
model model)
2. CPM is used for repetitive jobs PERT is used for non repetitive jobs
3. CPM makes use of cost duration PERT analysis does not consider cost
relationship to arrive optimum
schedule
4. CPM is an activity oriented PERT is an event oriented
5. CPM is used for construction and PERT is used for planning scheduling
business problem research programme

Probability of completion of project by a specified date

1. Find the variance of time estimate of all determines variance of each activity =
012 134
5
σ = variance of project duration – sum of variances of time estimates of all critical
activities.
2. Find the probability of finishing the project on some fixed target by using the table of
normal distribution
Value of z = Due date- Expected date of completion /standard deviation of
critical path σ
σ = 678- )9 :;< ; =>7 )9 =< , =;? ;=, : , >7
Problem 11
For a project given below,Find1. The expected time for each activity
2.TG&TL values for all events
3. EST, EFT, LST,LFT values for all activities 4. Critical path
The time estimates are
Task A B C D E F G H I J K
Least
4 5 8 2 4 7 8 4 3 5 6
time
Greatest
6 9 12 6 10 15 16 8 7 11 12
time
Most
likely 5 7 10 4 7 8 12 6 5 8 9
time

120
Solution

Most
Optimistic time Pessimistic time Expected time
Task probably
to tp Te=1/6(to+4tm+tp)
timetm
A 4 6 5 5
B 5 9 7 7
C 8 12 10 10
D 2 6 4 4
E 4 10 4 4
F 7 15 7 7
G 8 16 8 9
H 4 8 12 12
I 3 7 6 6
J 5 11 8 8
K 6 12 9 9

Using expected time as duration of activities


E1 = 0, E2 = E1+5 = 0+5 = 5
E3 = Max (0+7, 5+10) = 15
E4 = 0+4 = 4
E5 = max (15+7, 4+12) = 22
E6 = Max (22+5, 4+12 ) = 27
E7 = 22 + 6 = 28
E8 = Max (28+9, 27+8) = 37
L8 = 37
L7 = 37--9 = 28
L6 = 37--8 = 29
L5 = Min (28-6,
(28 29-5) = 22
L4 = Min (29-12,
(29 22-19) = 13
L3 = Min (28-6,
(28 29-5)
L1 = 22--7 = 15 L2 = 15-10 = 5 L1 = 0

121
Task Te EST EFT LST= LFT – duration LFT Float LST - EST
A(1-2)
2) 5 0 5 0 5 0
B(1-3)
3) 7 0 7 8 15 8
C(2-3)
3) 10 5 15 5 15 0
D(1-4)
4) 4 0 4 9 13 9
E(3-5)
5) 7 15 22 15 22 0
F(4-5)
5) 9 4 13 13 22 0
G(4-6)
6) 12 4 6 22 29 9
H(5-7)
7) 6 22 28 22 28 13
I(5-6) 5 22 27 24 29 0
J(6-8) 8 27 35 29 37 2
K(7-8)
8) 9 28 37 28 37 0
Critical Activities are 1-2,2-3,3-5,5-7,7-8
1
Critical Path A-CC-E-H-K
1-2-3-5-7-8

Problem 12

The following table lists the jobs of a network along with three time estimates

Jobs Optimistic Most likely Permissible


1-2 3 6 15
1-6 2 5 14
2-3 6 12 30
2-4 2 5 8
3-5 5 11 17
4-5 3 6 15
6-7 3 9 27
5-8 1 4 7
7-8 4 19 28

122
22
1.Draw the project network

2. Calculate the length and variance of critical path

3.What is the approximate probability that jobs on critical path can be completed in

a. 41 days b. 35 days

4.What is the probability that the probability that the project will not be completed within
45 days.

5. Find the due date which has 95% chance to meet


Availability to tm tp te = 1/6 (t00 + 4tm + tp)
1-2 3 6 15 7
1-6 2 5 14 6
2-3 6 12 30 14
2-4 2 5 8 5
3-5 5 11 17 11
4-5 3 6 15 7
6-7 3 9 27 11
5-8 1 4 7 4
7-8 4 19 28 18

Tg and TL values are as follows

E1= 0

E2 = 0+7 = 7

E3 = 7+14 = 21

E4 = 7+5 = 12

E5 = Max (21 +11, 12+7) = 32

E6 = 0+6 = 6

E7 = 6+11 = 17

123
E8 = Max (32+4, 17+18) = 36

L8 = E8 = 36

L7 = 36-18 = 18

L6 = 18-11 = 7

L5 = 36-4 = 32

L4 = 32-7 = 25

L3 = 32- 11 = 21

L2 = Min (21-14, 25-15) = 7

L1 = Min (7,7, 7-6) = 0

Availability Float Variance =1/6 (tp – to) 2


1-2 7-0-7=0 4
1-6 7-0-6=1 4
2-3 21-7-14=0 16
2-4 25-7-5=13 1
3-5 32-21-11=0 4
4-5 32-12-7=13 4
6-7 18-6-11=0 16
5-8 36-32-4=0 1
7-8 36-17-18=1 16

Critical activities are (1,2) (2,3) (3,5) (5,8)

variance of critical activities are 4,16,4,1

Critical Path=1-2-3-5-8

Optimum length of critical path = 7+14+11+4 = 36 days

124
Variance of critical path = 4+ 16+4+1 = 25

σ=5

3a. Scheduled time of completion of job is 41 days t = 41

Z = t-t0/σ = 41-36/5 When z is the area under standard normal curve is 0.34

Since Z is positive,Probability= 0.5+ 0.34= 0.84

Probability that the job will be completed in 41 days=0.84

b. .t = 35 , z = 35-36/5 = -0.2

Area corresponds to z = -0.2 is 0.0793

Since z is negative, probability = 0.5-0.0793 = 0.4207

Probability that the job will be completed within 35 days= 0.42

3. Probability that the job will not be completed with 45 days = 1-probability job
completed with 45 days when t = 45

Z = 45-36/5 = 1.8

P(z<1.8) = 0.5+0.4641 = 0.9641

P(Job will not be completed within 45 days)

= 1-0.9641 = 0.0359 = 3.597%

5. Given probability = 0.95=0.5+0.45

When z = 1.64, area = 0.4495

t-te/σ = z

ie. 1.64 = t-36/5

t= 36+8.2 = 44.2 day

Due date for which 95% chance – 44.2 days

Advantages of PERT & CPM

1. It helps the management to concentrate their attention on critical activities and


completion on time
2. It provides the best way of planning and scheduling of projects
3. Gives complete information about duration slackiness of activities

125
4. Suggests area for increasing efficiency and reduction of cost.

5. Helps to formulate new schedule when existing schedule cannot be met


6. Minimize the delay in competition of project

126
127
Critical activity = 1-3-5
Optimum length of critical path = 12+16 = 28
Variance of critical path
Activity Variance

015 − 94
=6
6
1-3

0 A4
5
3-5 = 144/6 = 24

SD = √30 = 5.47
1 1
. E
2. Z = σ
= = 2/5.47 = 0.3656

Where z is the area under normal curve

Z <0.3656

Since z is positive, probability 0.5+ 0.3656

= 0.8656

Probability that the job will be completed within 30 days = 0.8656

Question 13

A small project is composed of seven activities where estimates are given in the following
table.
Activity Name of activity optimistic Most likely pessimistic
1-2 A 9 9 36
1-3 B 9 18 27
1-4 C 18 18 45
2-5 D 9 9 9
3-5 E 18 45 72
4-6 F 18 45 63
5-6 G 27 45 81
a.Find the expected duration of each activity
b. What is the expected project duration
Solution

te = to + 4 tm + tp/6

128
Calculation te = to + 4 tm + tp/6

9 F 04 G 944 F 36
Activity Expected time te

6
9 F 04 G 1844 F 27
A 13.5

6
18 F 04 G 1844 F 45
B 18.0

6
9 F 04 G 944 F 9
C 22.5

6
18 F 04 G 4544 F 72
D 9.0

6
18 F 04 G 4544 F 63
E 45.0

6
27 F 04 G 4544 F 81
F 43.5

6
G 48.0

Hence to determine the expected project duration we should draw a network diagram for
project, determine the expected lengths of various paths in the network using expected
times

Various path in the network

A – D- G B E–G
B-

C–F

Expectedd length of each path

A – D- G = 13.5+ 9+ 48 = 70.5 days

B- E- G = 18+45+48 = 111 days

C-F
F = 22.5 + 43.5 = 66 day

129
Critical path is B- E – F with expected length of 111 days

project duration 111 days

Activity Questions

Problem 14

In a PERT network the critical path comprise of 6 activities. Its estimated duration of
days is given below.
Activity optimistic Most likely permistic
1 6 12 24
2 16 18 20
3 12 16 20
4 4 10 16
5 6 18 24
6 6 8 10

If the projects scheduled for completion within 84 days what is the probability of
achieving schedule(Ans: 0.6554)

Problem 15

For a small project, the critical path comprises of 4 activities whose estimates duration in
days are given below.
Activity optimistic Most likely pessimistic
1-2 3 5 7
2-4 7 9 11
4-5 - 0 -
5-6 6 8 10

Compute the probability for achieving project duration of

a.20 days(Ans.0.0405)

23.5 day (Ans 0.9032)

Crashing of projects

The normal time of a project duration is the to time duration of all critical activities
together (ie critical path duration). The reduction in the project duration can be achieved
by reducing the time required for selected activities, from their normal time
The maximum reduction in the time possible for any activities is limited to its crash time
(ie beyond this time, reduction is not possible)
The progress of reducing the total project duration by reducing activity timings is known
as crashing of project

Procedure of crashing

130
1. Find the critical path and identify the critical activities

JKLMN O3M1 P3KQLR O3M1


2. Calculate the cost slope for different activities using the formula
cost slope = P3KQLR 1#QS JKLMN 1#QS
3. Rank the activities in the critical path as per the ranking
4. Activity having lower cost slope should be crashed first to the maximum extent
possible

Calculate the new direct cost by adding the cost of crashing to the normal cost

5. As the critical path duration is reduced by crashing as per step 3, other path become
critical as we get parallel critical paths

This means that project duration can be reduced by simultaneous crashing of activities in
the parallel critical paths

6. By crashing as per step 3 and 4, one reaches a point when further crashing is either
not possible or does not result in the reduction of crashing of project duration (This
can happen when activities which can be crashed lie in the non critical path)

For the different project duration total cost is found out

Total cost is got by adding corresponding over head cost to the direct cost and the
duration cost is got by adding the crashing cost cumulatively to the normal cost.

Problem 16

The following table gives duration in days and cost in Rs. Of the activities for a project
Activity Normal time Crash time Normal cost Crash cost
1-2 4 3 600 800
1-3 2 2 400 400
1-4 5 4 750 900
2-3 7 5 400 600
2-5 7 6 800 1000
3-5 2 1 500 650
4-5 5 4 600 850
4050

Indirect cost per day is Rs. 200

a.Draw the network of the project

b.Find the normal duration and cost of project

c. Find the optimum duration and cost of project

131
Solution

a.critical path is 1-2-3-5


1

Normal duration of project = 4+7+2 = 13 days

Cost of project = sum of normal costs + Indirect cost for 13 days

= 4050 + (13 x 200) = Rs. 6650

c. Optimum duration is the duration for which the total cost is minimum
minimu

To find the optimum duration, we crash the activities calculate the cost slope for each
activity using

JKLMN O3M1 P3KQLR 1#QS


Cost slope = P3KQLR 1#QS
JKLMN 1#QS

800 600
Activity Normal time Crash time Normal Crash Cost Cost slope
200
4 3
1-2 4 3 600 800

900 750
1-3 2 2 400 400 0
150
5 4
600 400
1-4 5 4 750 900

200
7 5
1000 800
2-3 7 5 400 600

200
7 6
650 300
2-5 7 6 800 1000

150
2 1
850 600
3-5 2 1 500 650

250
5 4
4-5 5 4 600 850

In the critical path, 1-2-3-5


1 cost slope is least for activity 2-3
2 3 and can be crashed for 2
days

132
Available time for crashing

Activity Normal time


1-2 4-3=1
1-3 2-2=0
1-4 5-4=1
2-3 5-4=1
2-5 7-5=2
3-5 7-6=1
4-5 2-1=1
4-5 5-4=1

First stage crashing

Activity Possible crashdays Cost slope


1-2 1 200
2-3 2 100
3-5 1 150

Cost slope least for activity 2 and 3 and can be crashed for 12 days

so the project duration reduces to 11 days

Extra cost incurred = 100 x 2 = 200 Rs

Now there are 2 critical paths

1-2-3-5 1-2-5

Second stage crashing

Activities in critical path Possible crash Cost slope


1-2 1 200
2-3 0 -
2-5 1 200
3-5 1 150

To reduce the project duration to one more day (to reduce the days of duration to 10) we
have two choice

a. crash 1-2 by one day criteria cost being 200

Crash 2 & 5 & 3 & 5 by one day (extra cost being Rs. 150+200 = 350)

So it is economical to select choice a

So now the project duration is 10 days

Now there are three critical paths

133
1-2-3-5

1-2-5

1-4-5 Third stage crashing


Activies in critical path Possible crash Cost slope
1-2 0 -
2-3 0 -
2-5 1 200
3-5 1 150
4-5 1 250
1-4 1 150

For reducing the number of says by one, the choice are

a. Crash one day from 1-4, 2-5, 3-5

Cost being 150+200+150 = 500

b. Crash one day from 4-5, 2-5 & 3-5

Cost being 250+200+150 =600

It is economical to select choicea. Thus the duration of critical path is 9 days

Now We have same critical paths

No further crashing is possible as all possible crashing has been done.

Table showing various stages of crashing


Indirect cost Direct cost =normal cost + crash cost Total
Stage Duration
@(Rs 200
0 13 2600 4050 6650
1 11 2200 4050+200 6450
2 10 2000 4050+400 6450
3 9 1800 4050+900 6750

Total cost is least for the duration of 10 days

So optimum duration is 10 days with the total cost of Rs. 6450.

Problem 17

The following table gives the activities and other relevant data of a project
Activity Normal Crash Normal Crash
1-2 4 3 120 100
1-4 6 4 300 500
1-3 2 1 60 120
2-4 5 3 300 500

134
3-4 2 2 200 200
2-5 7 5 230 350
4-5 4 2 300 480

Indirect cost per day for the project is rs. 100

a. Draw the network diagram


b. Find the normal duration and cost of the project
c. Crash the number of days to the maximum possible
d. Find the optimum duration and cost?

Normal duration of the project =4+5+4=13 days

Normal cost of project = 120 + 300 + 60 + 300+ 200 +230 + 200 (13 x 100) = 2710

JKLMN TUVW P3KQLR O3M1


P3KQLR XYKL1#3! OKLMN XYKL1#3!
Cost slope =
Activity 1-2 1-4 1-3 2-4 3-4 2-5 4-5
Cost slope 60 100 60 100 0 60 140

Ist stage crashing

Activity in critical path Possible crash days Cost slope


1-2 4-3=1 60
2-4 5-3=2 100
4-5 4-2=2 140
Lowest cost slope is for 1-2 . We can crash 1 day from 1-2
Extra cost is Rs. 60. Project duration reduces to 12 days
2nd stage crashing
Activity in critical path Possible crash days Cost slope
1-2 0 60
2-4 2 100
4-5 2 140
Least cost slope for 2-4 (for 1-2 there is no possible crash days)

135
Lowest cost slope pertaining to activity 2-4 crash 2 days from 2-4. Extra cost is Rs. 200.
Hence project duration is 10 days
Now there are 3 critical paths. That is path having duration 10 days
They are 1-2-5
1-2-4-5
1-4-5
Therefore we have to crash simultaneously from all these three paths
IIIrd stage crashing
Activity in critical path Possible crash days Cost slope
1-2 0 60
2-5 2 60
1-4 2 100
4-5 2 140
2-4 0 100

Now we have only one choice. That is crash 2 days from 2-5 and 4-5 simultaneously

Extra cost isRs. (140 + 60) x 2 = 400

Then the project duration is 8 days

Now we have 4 critical paths. All have 8 days duration

They are 1-2-5

1-2-4-5

1-4-5

1-3-4-5

IV stage crashing

No further crashing is possible because we have no crash days simultaneously to be


applied on the critical path

Table showing various stage of crashing


Stage Duration Independent cost Normal cost + Crash cost Total
Before crashing 13 1300 1400+0 2710
After crashing
1 day 12 1200 1410+60 2670
2 days 10 1000 1400+(200+60) 2670
1 day 8 800 1410 + (400 + 200) 2870

136
After crashing maximum possible number of days project duration in 8 days with a total
cost of RS. 2870. But the total cost is minimum is RS. 2670 which the project duration is
10 day

Optimum duration = 10 days and cost = Rs. 2670.

Activity Question 18

A project has five activities and its is requested to prepare least cost schedule for all
possible duration from normal time to crash time
Activity Preceeding activity Normal Crash Normal Crash
A - 4 3 260 320
B - 8 5 300 510
C A 5 3 170 270
D A 9 7 220 300
E B,C 5 3 200 60

Ans (Project crash duration = 140 days Critical Path-A-C-E 14 days

-Project crash cost = Rs. 1570)

Least cost scheduling there wise called crashing

Features of crashing

1. It is an important aspect in project scheduling


2. It tries to find the cheapest method of reducing the overall project completion time
thereby reducing the time of activities in critical path

Resource allocation
A resource is a physical variable such as man, material , machine, space or money that is
required for completing various activities or jobs of a project.

The network analysis PERT/ CPM can be successfully carried out if the availability of
resources is liberal or unlimited. But usually all necessary resources will not be available
in unlimited quantities. Availability of the some if the resource such as manpower and
material may be restricted. Availability funds, capital investment and heavy equipment
are the most important resources that need be allocated carefully

For a given network, the requirements of various resources are determined using the early
start schedule of each activity. There may be activities which are to be performed
simultaneously and may require common resources. The requirement of resources to
execute these simultaneous activities may exceed the available resources. Hence the
requirement of a particular type of resources may not be uniform all along the project
duration. The planning should be done on such a manner that resource are utilized in a

137
manner that resources are utilized in a more or less uniform manner. Large fluctuations in
these demand may cause problem in the project execution

The resource allocation procedure consists of two main activities

1. Resource smoothing
2. Resource leveling

Resource smoothing (Load smoothing)

If the constraint is the total project duration, then the resource allocation only smoothens
the demand on resources. Hence the period for maximum demand is located and the
activities according to their float are shifted so that there is balance between availability
and requirement of resources. Proper utilization of floats can smoothen the demand of
resources to the maximum allocation is called resources smoothening

Resource leveling (load levelling)

There are various activities in a project demanding varying levels of resources should not
go beyond the prescribed level. The operation of resource allocation is called resource
leveling.

Difference between resource smoothening and resource level

In resources smoothening methods, the total project duration is not changed. But some of the
activities stage times are shifted by these available floats so that a uniform demand for the
resources is generated. However the resources are considered to be unlimited.

In resource leveling, the activity start times are so rescheduled that the peak demand for a
particular resource are considered to be limited. In resheduling the activities, the floats are
used. But its does not give the desirable results , the total project duration may be changed.

Given the following information suggest some appropriate allocation schedule


Activity 1-2 2-3 2-5 3-4 4-7 5-6 6-7 7-8
Duration
6 4 1 3 4 3 3 5
days
Men 8 4 2 6 4 3 4 4

1.Maximum men requested at any time

2.No of days in which work will be completed

138
Maximum duration = 22 days

Work can be got done in the following manner


1-2 6 days 8
2-33 & 2-5,.
2 2-3 & 5-6 1 day 3 day 3+1= 4 days 4+2 = 6 man
3-4 3 days 6 man
6-77 & 4-7
4 3 days 4+4 = 8 man
4-7 1 day 4 man
7-8 5 days 4 men
22days Max 8 men at a time

Consolidation

Day 1-6 7 88-10 17-13 14-16


16 17
Men 8 6 7 6 8 4

Resource Histogram

139
Maximum number of men required = 8

No.of works will be completed = 22 days

Problem 20

Manpower required for each activity of a project is given below.


Activity 0-1 1-2 1-3 2-4 3-5 3-6 4-7 5-7 6-8 7-9 8-9
Duration 2 3 4 2 4 3 6 6 5 4 4
Crew size men 4 3 3 5 3 4 3 6 2 2 9

Find the maximum man power required for the project with same duration after rescheduling
source allocation by resource smoothing technique

Solution
TE values for all events
E0 = 0 E1 = 0+2 =2, E2 = 2+3 = 5
E3 = 2+4 = 6
E4 = 5+2 = 7 E5 = 6+4 = 10 E6 = 6+3 = 9
E7 = Max (10+6, 7+ ) = 16
E8 = 9+5 = 14 E9 = max (16+4, 14+4) = 20
TL value
L9 = 20 L8 = 20-4 = 16 L7 = 20-4 = 16 L6 = 16-5 = 11
L5 = 16-6 = 10 L4 = 16-6 = 10 L3 = Min (11-3, 10-4) = 6
L2=10-2=8 L1=Min( 6-4,8-3) =2 L0= 0

Critical path is 0-1-3-5-7-9

Project duration = 20 day

140
141
142
143
144
University Exam previous Questions in Unit 3 (Module 3)

1. What are the different types of floats associated with an activity in CPM Model(5
Marks)

2 .What are the uses of floats? 5 marks

3 .Write short note on critical path method (5 Marks)

4. How are net works useful to the management-(10 Marks)

5 .Explain the terms Optimistic, Pessimistic and most likely time estimates(5 Marks)

6. Distinguish between PERT &CPM(5 Marks)

7. .Distinguish Between PERT &CPM(5 Marks)

8 .Explain the different types of floats associated with an activity in CPM Model (5
Marks)

9 Write short note on PERT (5 marks)

145
Unit IV

Queuing Theory

A queue is formed when the demand for a service exceeds the capacity to provide that
service.

Queing theory is a quantitative technique which consists in constructing mathematical


models of various types of queuing system. Queuing theory deals with the analysis of queries
and queuingbehaviors.

Objectives of Queing theory

1. To achieve the economic behavior between waiting time costs and service costs
2. Queuing theory can be used to estimate the different characteristics of the waiting line
such as average arrival rate, average service rate, average waiting time and average
time spent on the system
3. Queuing theory helps in taking decision about the optimum capacity so that the cost
of investment is minimum keeping the extent of queue within tolerancelimits.

Application of queuing theory

1. Scheduling of aircraft
2. Scheduling of works and jobs in Production control
3. Minimization of congestion due to traffic delay in Toll booths
4. Scheduling of components to assembling line
5. Scheduling and routing of salesman

Role of Queuing theory in management

Queuing theory plays a very important role in management ultimate good of queuing
theory is to achieve an economic balance between cost of service and cost associated with
waiting, based on probability theory it attempts to minimize the extent and duration of
investment costs.

Queuing theory is able to provide with the estimated average time and intervals under
sampling method and helps in taking decision about the optimum capacity. So that cost of
investment is minimum keeping the extent of queue within tolerance limits.

Queing system

Arrival – Queue – service – outlet

Types of queuing system

1. Single queue – single service point – There is only one queue and one service
station. This system is also known as single channel facility. In this system, the

146
customer waits in the only queue that is available till the service point is ready to take
him for servicing
2. Single queue multiple service points

In this system, there is only one queue there are severalservice points. A customer waits
in single queue until one of the services station is ready take him for servicing.

3. Multiple queue single service point


In this system, there are several queues, the customer can join any one of these several
queuesbut there is only one service station.
4. Multiple queue multiple service points

In this system, there are several queues and also several service station also known as
multi channel facility

Terminologies

1. Customer – Customer are these who waiting in the queue or receiving service
customers may be people, machine, ships, letters etc.
2. Queue – A group of item such as people, machine etc waiting for service in a service
station
3. Server – A server is a person or machine by whose service is rendered
4. Service station – Point where service is provided
5. System – Queue + service
6. Queue length – No of customers waiting in the queue
7. Time spent by customer – Time spent for waiting in the queue plus service time
8. Queuing system – Compress of arrival of customers waiting in queue, picked up for
service being served and departure of customers.
9. Average length of queue – Number of customer in the queue per unit time
10. Waiting time : Time upto which customer has to wait in the queue before taken to
service
Customersbehavior in queue.
1. Jockeying – Moving from one queue to another queue, when there are several
queues in the hope of reducing waiting time
2. Reneging – Joining the queue and leaving it after ward.
3. Balking – Customer have priority over others. Customers are served before others
regarding of their order of arrival

Limitations of queuing theory

1. Most of the queuing models are complex cannot be easily understood. There is
always an element of uncertainty
2. Queue discipline also impose some limitation
3. In multiple channel queuing several times to practice from one queue forms
arrival for another make analysis more complex.

147
Characteristics of queing model

1. Arrival distribution

It is the input process. It represents the in which no of customer arrive at the system
called random arrival.

Arrival rate is defined as the rate at customers arrive to be service or the number of
customer arriving per unit of time

Arrival time is randomly distributed according to Poisson son distribution

Men value of arrival, rate is represented by λ

2. Service (departure) distribution


It represents the pattern in which the no. of customers leave the system
No of customers served per unit of time is called Service rate
Service facilities can be fixed or distributed in accordance with some probability
distribution
Average no of customers being served in unit of time u
Average time taken to service a customer=1/u=Service time =

3. Service discipline or queue discipline


If the service facilities are free, the incoming customer is taken into service
immediately
If the service facilities are busy the customers in the queue may be handled in
either of the following ways services become free
1. FCFS (FIFO)
2. LIFO
3. Random service
4. Priorities

4.Queue output

In a single channel facilities the output of the queue does not possess any problem
for the customer who leaves after getting service. When the system is multi stage
channel facilities, because break down of a service station, can have repurcuison
on the queries.

Simple queue – single channel system characteristics

1. Single queue and single service point

148
2. Queue discipline is first come first served
3. Queue has infinite capacity
4. Arrivals are random and follow a poison distribution
5. No simultaneous arrivals
6. Service times are random and follow exponential distribution
7. No simultaneous arrivals
8. Traffic intensity is less than 1
Traffic Intensity δ

Traffic intensityδ is defined as the rate of mean arrival λ to mean service rate µ

ZSL! LKK#[LR KL1S


Traffic intensityδ = ZSL! MSK[#OS KL1S = µ
λ

Simple queue δ<1

δ>1, queue will grow without end

δ= 1, No change in queue system

Kendal’s notation for queuing system

Characteristics of queing system has been universally standardized in the format a/b/c

Where a = arrival distribution

b= service time distribution

c= no. of servers in the system

This is referred as kendels equation

Eg. Simple queue which has poisson arrival distribution, exponential service time
distribution and single server represented as M/M/1

Formulaes in queing system

1. Expected no. of customers or unit in the waiting line or serviced or


Expected numbers in the system E(n) = 0µ λ4
λ

2. Expected number of units (or customers ) in the queue

µ0µ λ4
λ
Queue length = E(nq) =

µ0µ λ4
λ
3. Average waiting time in the queue of an arrival E(w) =
4. Average time an arrival spend in the time system

149
E(v) = µ λ
5. Probability that the no an the queue and being served is greater than k is
λ
P (n>k) =(µ )k+1

Problem1

1. The following information is available with regarding to a queue system


a. Customer arrives at the rate of 12 hours
b. Arrival and service follow Poisson distribution
c. Service rate is 15 customers per hour
calculate
1. Average no. of customer waiting for service
2. Average time a customer must wait in the queue
3. Average time that a customer must spend is the system
Mean arrival rate of customer λ = 12 per hour
Mean service rateµ = 15 per hour
Solution
1. Average no. of customers waiting for service
= µ0µ λ4 0 4
λ λ
= = 3.2 customer

2. Average waiting time of customer = 0µ λ4


λ
=
= 0.27 hrs = 16 mts
3. Average time a customer spend in the system
= 0µ λ4
=0
4
= 0.33 hours = 20 mts
Problem 2( SDE Feb 2015 -10 Marks)

In a supermarket the average arrival rate of customer is 10 in every 30 minutes following


Poisson process. The average time taken by the cashier to list and calculate the customer
purchase is 2.5 minutes following exponential distribution

1. What is the probability that queue length exceeds 6


2. What is the expected time spend by the customer on the system?
λ= minute = x 60 = 20 customer/ hour

.
µ= x 60 = 24 customer/ hour
λ
Pn>6 (µ)k+1 = ( ) 6+1= 0.279
3. Expected time spent by the customer in the system
= (µ λ
= = 0.25 hours
= 0.25 x 60 = 15 minutes
Problem 3 Dec 20108- 10 Marks

150
Arrivals in a queue are considered to be poisson with an average time of 6 minutes
between one arrival and the next, length of queue service is an exponential variable
with mean time of 2 minutes
a. What is the probability that a person average at the queue will have to wait
b. What is the average length of queue which forms from time to time?
Solution

5 5
= 6 ,λ = person / minute = x 60 person = 10 person/ hour
=µ = 2, µ = ( ) person/ minute = 60 = 30 \]^_`a/hour
λ

λ
a. Probability that a person arriving at the queen will have to wait pn>k = ψ

=Pn>0= Traffic intensity = = 0.33

b. Average length of queue = µ0µ λ4 0 4


λ
= = 0.16
4. Arrive at an telephone booth are considered to be poisson with an average time of 10
minutes between one arrival and the next. Duration of a phone call assumed to be
distributed exponentially with mean 3 minutes
1. What is the probability that a person arriving a the booth will have to wait
2. Fraction of time the phone will be in use
3. Telephone Company would expect to have to wait atleast three minutes for the
phone. By how much increase inorder to justify a second booth
4. Find the average number of units in the system

Solution

λ= x 60 = 6 per hour

µ = x 60 = 20 per hour

1. Probability a person have to wait (an arrival have to wait) k= 0,


pn>k=( b )k+1
5
b
λ
= = = 0.3

2. Fraction of time the phone will be in use = = b = 0.3


λ

b0b λ4
λ
3. Average waiting time in the queue =

b0b λ4
λ
= 3 minutes = 0.05 hours µ = 20

0 λ4
λ
= 0.05 λ = 10 per hour

Increase in the mean arrival rate = 10-6 = 4customers per hour.


second booth is justified when the increase in the arrival rate of customer is 4 hours
4. Average number of units in the system = b
λ
λ

151
5
5
= = 3/7
Problem 5
5. In a bank every 15 minutes, one customer arrives for cashing the cheque. The
staff in only payment counter takes 10 minutes for serving the customer on an
average.
state suitable assumption and find
1. Average queue length
2. Increase in arrival rate inorder to justify a second counter ( when the waiting
time of a customer is atleast 15 minutes, the management will increase one
more counter)

Solution

Assumption : Arrival pattern follows Poisson distribution

Service time follows exponential distribution

λ = 1/15 per minute = 4 per hour

µ = 1/10 per minute = 6 per hour

1. Average queue length = b0b λ4


= 505 4
λ
= 1.33
2. Second counter is justified when the average waiting time of arrival in queue
should be at least 15 minutes
Let λ1 be the new arrival

b0b λ4
=5
λ

60 λ1= 15 x 6 (6-λ1) λ1= 3.6

For second counter to install, required arrival rate is 3.6 per hour

But present arrival rate λ = 4 per hour

So even now a second counter is justified.

Problem 6

UNI SDE Nov 2011(10 Marks) A single man post office has 5 consumer for which the
single postman perform services. Assume that the service item is exponential andarrival rate
is poisson, find

1. Average waiting time of an arrival


2. Average length of waiting line
3. Average time an arrival spends in the system if the average rate is 4 per house with an
average service time of 10 minutes

152
Solution

λ = 4 per hour

µ= man power per minute = 6 man per hour

1. Average waiting time = b0b λ4


= 505 4
λ
= hr = 20 minutes

2. Average length of waiting time = b0b λ4


= 505 4
λ
= 1.33

b λ
3. Average time an arrival spends in system = =5 = 0.5 hr
= 30 minutes

Problem 7
KU Feb 2016 (5 Marks)

In a public telephone booth, the arrival on the average 15 per hour. A call on average takes 3
minutes. If there is just one phone, find the expected number of callers in the booth at any
time

Solution

λ = 15 per hour

µ = x 60 = 20 per hour

Expected no. of callers in the booth = b


λ
λ

= = = 4 callers

Problem 8

At a certain petrol pump customers arrive according to poisson distribution with an average
time of 10 minutes between arrival. The time intervals between services at the petrol pump
follow exponential distribution and the mean time taken to service a customer is 4 minutes.

calculate

1. Expected average queue length


2. Average number of customers in queuing system
3. Average waiting time of customers
4. Average time spent by a customers in system

153
By how much should the flow of customer be increased to justify the opening of a second
service point if the management is willing to open the same provided the customer has to
wait for 10 minute for the service?

Solution

Mean arrival rate = 1/10 = 0.1 x 60 = 6 customers

b
= 4 µ = = 0.25 per minute = 0.25 x 60 = 15 customers

5
b0b λ4 0 54
λ
1. Average queue length = = = 0.267 customers
5
2. Average no.of customers in the queuing system = b =0 54
λ
λ
=0.667customers

b0b λ4
λ
3. Average waiting time of customer in the queue = =
16/15(15-6)=2.67 minutes
4. Average time spend by the customer in the sytem=b λ
=1/(15-16)=
6.67 minutes

Average waiting time of a customer at present are calculated above is 6.67


minutes.Management is willing to open a second service point only if a customer has to
wait for 10 minutes E(w) = 10 minutes or (10/60) hr

Mean arrival rate λ to determine which will result in an average waiting time of 1/6

Mean arrival rate λ

5
= b0b λ 4 0
λ λ
= λ

λ1 = 10.71

Hence the mean arrival rate of a customer should go from the present rate of 6 customers per
hour to 10.71 customer per hour inorder to justify the opening of a second service point

Activity Questions

Problem 9

People arrive at a theatre ticket booth in a poison distribution arrival rate of 25 per hour
average time is exponentially distributed with an average time of two minutes

Calculate
1. Mean number in the waiting line
2. Mean waiting time
3. Utilization factor

154
Ans :(4.12, 10, 83%)

Problem 10

A postal clerk can service a customer in 5 minutes, the service time being
exponentially distributed with an average of 10 minutes during the early morning
slack period and an average of 6 minutes during the afternoon peak period. Calculate

a. Average queue length


b. Expected waiting time in the queue during the two periods
Problem 11

A ticket window of a cinema theatre is manned by a single individual. Customers


arrive to purchase tickets in a Poisson fashion of mean rate 30 per hour. The time
requested to serve a customer has an exponential distribution with mean of 90 sec.

Find (1) expected queue length (2) expected waiting time

Ans :3,6 minutes

Problem 12

Car arrives at a toll rate on a frequency according to poisson distribution with mean
90 per hour. Average time for poising through gate is 35 seconds. Drivers complaint
of long waiting time. Authorities are willing to decrease the passing time through the
gate to 30 seconds by introducing new automatic device. This can be justified only
ifunder the old system, the number of waiting cars exceeds 5. In addition the
percentage of gates idle time under the new system should not exceed 10. Can the
new device be justified

Ansλ = 1.5 µ = 1.71

Traffic intensity =b Expected queue length = b0b λ4


λ λ
= 6.27

.
Traffic intensity in new system = b=
λ
= 0.75

% of idle in new system 1-0.75 = 0.25 = 25%

Since the % of idle time in new system exceeds 10%, the new device is not justified.

Multichannel system

Queuingcost: costs associated with waiting in queue are known as waiting time costs
or queuing cost

Examples of queuing costs

155
1. Cost of idle time due to waiting
2. Loss of production when the equipment has to wait sometime to be repaired
3. Loss of sales due to customers going elsewhere upon seeing large queues.
4. Service cost : cost associated with service are known as service costs.

Examples of service costs

1. Wages payable to employees to improve service facilities


2. Cost of better equipment installed to provide improve facilities
3. Expenditure incurred for providing additional cost

Total Cost= Queuing cost+ Service cost

Problem 13

A company’s quality control department is manned by single clerk,Who takes an


average of 6 minutes in checking parts of each of machine coming for inspection. The
machine arrive once in every 10 minutes on the average. One hour of the machine is
values at Rs, 15 and the clerk’s time is valued at Rs. 6 per hour what is the average
hourly queuing system costs associated with quality control dept.

Total cost = Queuing cost + service cost

Queuing cost = machine awaiting for Inspection

Queuing cost is the idle time of machine

It depends on the time spent by the system

Queuing cost = Waiting time cost + Service time cost

Mean inter arrival time λ = per minute

= x 60 = 6 machines / hr

5
Mean service rate = = hr

Average time spent by the customer in the system=1/10-6=1/4 hour

Cost of one hour of the machine = Rs. 15

Average queuing cost per machine = Rs. 15 x = 3

Hourly average hourly queuing cost = 6 x 3.75 = Rs. 22.50

Service cost

156
There is only one server in the system
Hence the service cost of system consisting of cost of service. That is the average
hourly cost of clerk is Rs. 6
Total hourly cost of quality control dept

Hourly queuing cost = Rs. 22.5

Hourly service cost = Rs. 6

Total hourly cost = 22.5 + 6 = Rs. 28.50

Problem 14

In a large factory fitter draw parts from the store, which is at present staffed by one
store keeper .The factory manager is concerned about the time spent by the fitter
getting parts from the store. He wants to know if the employment of storesAssistant
would be worthwhile. On investigation it is found

a. A simple queuing situation exists


b. Fitter costs Rs. 5 per hour
c. Store keeper costs Rs. 4/- per hour and can deal average with 10 fitters / hour
d. A stores assistant could be employed at Rs. 3.50 per hour and would increase the
capacity of stores to 12 per hour
e. On average, 8 fitter visit the store each hourCalculate the total cost before and
after introduction of the Stores Assistant ,Whether it is viable to employ stores
Assistant in the factory

SolutionWe have to calculate the average number of fitters in the system before and
after introduction of stores assistant and cost comparison

Current system (1 store keeper)


Mean arrival rate λ = 8 fitters / hour
Average number of fitter in the system = b
λ

A
λ

A
= = 4 fitters
Fitter cost per hour = Rs. 5
Hourly cost of queuing = Rs. 5 x 4 =Rs. 20/-
Hourly service cost = Cost of store keeper per hour = Rs 4/
Total Cost =20+4=Rs 24

New system ( 1 store keeper 1 store assistant)

Mean arrival rate = 12 fitter / hours

Average number of fitters in the system = b


λ
λ

157
A
A
= = 2 fitters

Fitter cost per hour = Rs. 5

Hourly cost of queing = Rs. 2 x 5 = Rs. 10

Hourly service cost

Cost of store keeper per hour = Rs. 4

Cost of stores assistant per hour = Rs. 3.50

Total hourly service cost = 4+3.50 = Rs. 7.50

Hourly queing cost is reduced from Rs. 20 to Rs10

Hence savings in queing cost = Rs. 20 – 10 = Rs. 10

Hourly service cost increased from Rs. 4 toRs 7.50.

Hence increase in service cost = 10-7.50 = Rs. 3.5

Conclusion

There is a net savings from the new system which is Rs. (10-3.50) = Rs. 6.50

Hence employment of stores assistant is recommended.

Question 15A certain type of machine breaks down at an rate of Rs. 5 per hour. The
break downs are in accordance with the poisson distribution .Cost of idle machines
hours comes to Rs. 15/- per hour. Two repairman x and have been interviewed. X
changes Rs. 12 per hour and services the machine at the rate of 7 per hour where
asYchargesRs. 15 per hour and services the machine at an average rate of 9 per hour
which repairman services should be hired. (Assume that the workshift is of 8 hours)

Solution

Here the queue consists of machines waiting for service. To determine which
repairmen’s services should be used, We should calculate the total cost of queing
when x is employed.

Break down rate of machines (Mean arrival rate of λ)

Mean service rate (µ) = 7 per hour

Expected number of machines in the system E(n)

158
E.
λ
=µ = = 2.5machines per hour
λ

This means that 2.5 machines hours will be lost in an hour due to waiting
In a work shift of 8 hours, machine hours lost will be 2.5 x 8 = 20 machine hours
cost of idle machine hours – Rs. 15 per hour
Hence queing cost of one workshift = 20 x Rs. 15 = Rs. 300
Hiring charges of X = Rs. 12 per hour
Hence service cost of one workshift = Rs. 8 x Rs. 12 = Rs 96
Total cost of queing and service = 300+ 96 = Rs. 39
When Y is employed
Mean arrival rate λ = 5 per hour
Mean service rate µ = 9 per hour
Expected number of machines in the system
λ
E(n) = = = 1.25 per hour
µ λ
This means that 1.25 machine hours will be lost in an hour due to waiting
In a work shift of 8 hours, machine hour lost will be 1.25 x 8 = 10 machine hours
Cost of idle machine = Rs. 15 per hour
Hence queing cost of one workshift = 10 x Rs. 15 = Rs. 150
Hiring charges of y = Rs. 15 per hour
Hence service cost of one workshift = 8 x Rs. 15 = Rs.120
Total cost of queuing& service = 150+ 120 = Rs. 270

Since the total cost of queuing& service is less when repairman Y is employed it is
more economical to employ Y
Parallel multi channel system
Multi channelsystemarequeuing systems where there are more than one service points
and one or more queues. Service points may be series or parallel.
M/M/K : In this case there is only one waiting line and several service points. Arrival
follows Poisson distribution and service time follows exponential distribution
whereKendells Notation as M/M >K where k stands for parallel service points
No. of unitsor customer in the system is represented by n
If n< k, there is no queue because all the arrival are being serviced.
If n > k queue is formed.
Traffic intensity of multi channel system δ=λ/ku

Classification of queing models


Model I : M/M/J : (α/FCFS)
First M standard for poison arrival
Second M stands frompoisson departure
FCFS – First cum first served

Model II : M/M/I : α/FCFS for long cheque


Model III : M/M/I : N/FCFS
Model I (M/M/I) : α/FCFS

159
This is the queue model with
1. Arrival and departure following poisson distribution service rate following
exponential distribution
2. Single channel with infinite capacity
3. Service discipline :First come first served. There are single channel problems.
They are simple queuing problem. This is also called Birth and death model
4. Problems in previous pages belonging to model 1
Model II (M/M/I) : α/FCFS

General singlestationqueing model or General Erlang model

This is same as Model I except that mean arrival rate and service rate are not constant.
Both are dependent on n in the number of units in the system
Arrival rate = λn service rate = µn
Consider the cases (1) λn = !* and µn = µ
λ

(2) λn = λ (α constant ) and µn = nµ


In both cases, expected no of units in system
λ
E(LS) = δ=µ
E(LS) – Average no. of units in the system
E(Lq) – Average queue length
E(ws) – Average waiting time in system
E(wqs) – Average waiting time in system

λ
1. Average queue length E(lq) = ELs - µ
2. Average waiting time in queue E9wq) = λ (Elq)
δc
dLO13K#LR !
3. Probability for n units in the system δn = e-δ
4. Probability that the unit is idling δo = e-δ
When λn = λ and µn = µ the model coincides with model 1

Question No 18A shipping company has a simple unloading built with ships arriving
in a poisson fashion at an average rate of 3 per day. The unloading time distribution
for a ship with an unloading crews is found to be exponential with mean unloading

!
time days. The company has a large laboursupply without regular working hours
and to avoid long waiting lines the company has a policy of using as many unloaded
crews as there are ships waiting in line or being unloading Under these conditions,
find
a.The average number of unloading crews working at any time
b .probability that more than 4 crews will be needed
Solution
Mean arrival time λn = λ = 3 ships per day (constant)

160
Mean service rate is not constant. It depends on waiting line µn = = 2n
e

As there is one unloading crew, n = 1, µ = 2


λ
Average number of unloading crew working at any time E(LS) = δ = µ = = 1.5
S δ
Probability for n units = pn = dLO13K#LR ! Sn
P (ship entering service wall need more than 4 crews
= p(n≥5) = 1- (po + p1 + p2+ p3 + p4)

F F F k
S gh δ S gh δ S gh δj S gh δ
! ! ! !
1 -f

=1-] l1 + 1.5 + + + m
. 0 . 4 0 . 4 j 0 . 4
5

=0.019
Model III (M/M/I) = N /FCS

This is a queuing model with (a) Poisson service (b) single channel (c) finite capacity
(d) first come first served
In this system, λn = λ when n<N
λn = 0 when n ≥ N
Where N is a fixed quantity such that arrivals will not exceed N
λ
µn = µδ = µ

Pn= 0
Probability forn units in the system

δ P*
δ
)δn for 0 ≤ n ≤ N and δ ≠ 1

δ P*
δ
Po =

E(LS) = Po ∑!! aδa


Average number of units in the system

δ P*
δ
= [s + 2s2 + …… Nsn]

Problem no.13
In a railway marshaling yard goods train arrive at the rate of 30 teams per day.
Assuming that interval arrival time follows an exponential distribution and the service
time (time taken to hump atrain) distribution is also exponential with an average of 36
minutes. The line capacity of the yard is to admit 9 trains only (these 10 lines, one of
which one is ear marked for the shunting engine to reserve itself from thecrust of the
hump to the rear of train

Calculate the following on assumption that 30 trains on an average are received in the
yard

a. Probability that the yard is empty


b. Expected no .of trains in the yard
c. Average queue length

161
d. Average waiting time in the system
e. Average waiting time in the queue

Solution
N = 90
λ = 30 trains per day

5 5
µ= trains per machine = x 60 x 24 = 40 per day


λ
δ=µ= = 0.75

Probability that the yard is empty


.E
δn * 0 .E 4 o
δ
Po = = = 0.2649

Expended no of trains in the yard

2 δn *
δ
E(LS) = (δ+ 2δ2 + … 9δn)

= 0.2649 (0.75 + 2(0.75)2 + ….. 9 x (0.75)


= 2.4 trains
λ
Average queue length El-q1 = ELs - = 2.4 – 0.75 = 1.65 trains
µ

= λE(LS) = (2.4) x 24 hours = 1.92 hours

Average waiting time in the queue Ewq = λ Elq = x 1.65 x 24 =

1.32 hours

Problem 18
A petrol station has a single pump and space for no more than 3 cars ( 2 waiting and
one being served) A car arriving when the space is filled to capacity goes elsewhere
for petrol. Car arrive according to Poisson distribution at a mean rate of one every 8
minutes. Their service time has an exponential distribution with a mean of 4
minutes.
The proprietor has the opportunity of renting an adjacent piece of land, which would
provide space for an additional car to wait ( he cannot build another pump). The rent
would be Rs. 0.50 and the station is open 10 hours every day. Would it be profitable
to rent the additional space?
λ = 1 per 8 minute = A x 60 = 7.5 per hours
µ = 1 per 4 minute = x 60 = 15 per hour
λ E.
δ=µ = = 0.5

np
δ
Pn = Pn
δ

162
Maximum number in system = 3

Ien≤ 3 or N = 3

Consider when N= 3

.
0 . 4 0 . 4j
δ
P3 =
δ
x δ2= =0.067

Consider when N = 4

.
δ 0 . 4 0 . 4
δ
P4 = = = 0.032
δ

Increase in cases served per hour =λ (0.067 – 0.032) = 0.262

Increase in carsserved per week= 0.262 x 10 x 7 = 18.34

Increase in profit per week = 18.34 x 0.50 = Rs. 9.17

But rent for additional space = Rs. 10 per week which is more than 9.17

It is not economical to increase the existing space

163
Unit 4

Simulation
University previous Questions Part A& Part B)

1. Write a short note on Monte Carlo simulation June 2014 (5 Marks)


2. Explain simulation modeling – June 2014 (10 Marks )
3. What is simulation? Explain Monte carlo simulation(Nov 2013)-5 Marks
4. What are the uses of simulation –Nov 2012(5 Marks)
5. Define simulation. Give two application areas where the technique is suitable
(Dec 2008)-10 Marks
Simulation
Simulation model is a simplified representation of real life situation which helps to
understand a problem and helps to find the solution by trial and error approach.
Simulation technique
Simulation technique is used in situations where it is not possible to construct
mathematical tools like linear programming
Simulation is the process of designing a model of real system and conducting
experiment withthis model for the purpose of understanding the behavior for the
operation of the system.
Simulation is a problem solving technique which uses the computer aided
experimental approach to study problems that cannot be analyzed during formal
analytical methods.
Slope involved in Simulation

1. Identify the problem


2. Identify decision variable associated with the problem
3. Construct a numerical model
4. Validate the model
5. Design the experiments
6. Run the simulation method
7. Examine the results and select the best course of action

Identify problem

Identify decision variable,


performance criteria and decision
rule
Construct simulation model

Validate model

Design experiments (specify value


of decision variable to be tested)
164
NO
Run or conduct simulation Modify the model by
changing input ie
Is simulation process completed
YES values of decision
variables
Examine results & select best course
of action

Advantages of simulation
1.Simulation techniques can be used to solve the problem where the values of variables
are not known or partly known
2. Simulation methods are easier than analytical model
3. From the simulation model, data for further analysis can be generated
4. Effect of using the simulation model can be studied without using it in real situation
5. Simulation models are flexible
Disadvantages of simulation

1. Optimal results cannot be guaranteed by simulation


2. In many situation it is not possible to quantity the variables which affect the behavior
of system
3. In a number of situations, simulation is comparative costlier and time consuming
4. It is very difficult to construct simulation models if the number of variables are large
and their interaction ship is complex
5. Simulation technique provides only statistical estimates rather than exact results

Application of simulation techniques

1. Shop floor management

Analysing where to locate factors or plants inorder to be able to distribute goods and
service at lowest rate

Job scheduling problem

Implementation of new manufacturing technical such as just in time, flexible


manufacturing lean manufacturing, poke yoke, kanban system, kaizen etc

Level of machinery maintenance to minimize the service and break down cost

2. Design of queing system and Inventory system

Examining the ship arrivals and unloading in harbour point to avoid forming of queues

165
Testing a series of inventory order policies to find the least cost order point

3. Financial planning : Evaluation of investment opportunities


4. New product introduction in the market
5. Designing and planning of future production of automobile components
6. Demand forecasting
7. Capital budgeting problem
8. Replacement problem

Montecarlo simulation

Monte Carlo simulation is a simulation technique which is used to solve the problems
by the application of random number. They are selected in such a way that every number
has an equal chance of probabilityof selection.

Steps involved in Monte Simulation

Monte Carlo method is asubstitutionofmathematical evaluation of a model

The basics of Monte carlo techniques is the random sampling of variables possible value.
For this technique some random number are required which may be converted into
random variates whose behavior is known from past experience. Monte carlo technique is
concerned with experiments on random number and provide solution to complicated OR
problem

General procedure of Monte Carlo Methods

This method uses random number for originating some data by which a problem can be
solved. The random numbers are used in creating a new set of hypothetical data of a
problem from past experience. If no pattern can be assumed for the data, then randomness
can be assumed. When past information is not available, it can be obtained by conducting
a preliminary survey. The data collected are plotted on a graph from which a cumulative
probability function is obtained.

In Monte Carlo method, the sequence of s random number are selected from the random
number table. The random number obtained as taken a decimal numbers and also the
probability obtained at random from the parent population. These probabilities are
plotted on the cumulative frequency curve of the given data. The value of x correspond
to each probability given by the random number is the desired random sample values.

Steps involved in Monte carlo simulation

1. Construct the model which represents the relevant characteristics of the real world
system
2. Determine the probability distribution for the value of interest.
3. Determine the cumulative probability

166
4. Select the random numbers using random number table
5. From the sequence of random numbe,r determine the corresponding variables of
interest.

Advantages of Monte carlo method

1. Helpful in finding solution of complicated mathematical expression which is not


possible otherwise
2. By these methods, difficulties of trial and experimentation are provided.
3. Simulation techniques are easy to train staff and workers.

Disadvantages of Monte carlo method

1. These are costly way of getting a solution of any problem


2. These methods do not provide optimal answer to the problem. The answers are good
only when the size of sample is sufficiently large

Applications of Monte carlo simulation

1. Queing problem
2. Inventory problem
3. Risk analysis concerning major capital investment
4. Flexible budgeting
Problem 1(SDE Feb 2015)
1. Suppose the demand for a particular item is normally distributed with a mean of 175
units and standard deviation of 25 units / day. Simulate the demand for the next 20
days.

µ = 175 units σ = 25 units

Solution

Determine the cumulative probability using the normal distribution table. Assign random
numbers corresponding to probability

Determination of cumulative probability and assigning random numbers


Demand Z = x-µ/σ Cumulative q \ 100 Random
probability Ep number
100 -3 0.0013 0.13 0.01
125 -2 0.0228 2.25 0.14-22
150 -1 0.1587 15.87 2.29-15
175 0 0.50 50 15-818.50
200 +1 0.8413 84.13 50.01-84.7
225 +2 0.9772 97.72 84.14-97.7
250 +3 0.9987 99.87 97.73-10

167
Simulating demand for 20 days using random number given below
Day 1 Random number Demand x
1 59 180
2 65 190
3 29 159
4 13 150
5 56 178
6 12 145
7 63 206
8 61 205
9 70 192
10 75 195
11 54 177
12 90 210
13 96 222
14 26 156
15 43 173
16 07 128
17 08 129
18 40 172
19 79 208
20 45 174

Problem 2

A tourist car operator finds that during the past 100 days, the demand for the car had been
carried as shown below.
Trips per
0 1 2 3 4 5
week
No. of day 8 12 15 30 20 15

(Total 100 days)

Use random number simulate the demand for a 10 week period

(Use the random number 9,54,42,01,80,06,26,57,79,52)


Random number
Trip Probability Cum probability
interest
0 8/100 = 0.08 0.08 0-0.7(upto 8)
1 12/100= 0.12 0.20 08-19 (upto 21)
2 15/100=0.15 0.35 20-34(upto 35)
3 30/100=0.3 0.65 35-64 (upto 65)
4 20/100 = 0.20 0.85 65-84 (upto 85)
5 15/100 = 0.15 1.0 85-99(upto 100)

168
Simulation for 10 weeks
Demand(No.of
Weeks Random number
Trips)
1 9 1
2 54 3
3 42 3
4 01 0
5 80 4
6 06 0
7 26 2
8 57 3
9 79 4
10 52 3

Average demand = 23/10 = 2.3

The simulated demand for the cars for the new 10 weeks = 2.3

Explanation

To find the demand ,locate interval in which the random number of a week. The interval
shows the demand

Activity Question3

Suppose that the sales for a particular per day is Poisson with mean 5, then general 20
days of sales by Monte carlo method

169
Unit V

Game theory

Game Theory is a theory of conflict and it is mathematical theory which deals with
competitive situations

It is a type of decision theory which is concerned with the decision making in situations
where two or more rational opponents are involved under conditions of competition and
conflicting interests

Definitions

1.Game

Game is an activity carried out by two or more persons having conflicting interest who
agreed to play abiding by certain predetermined rules and limitations

2. Competitive games

This involves two or more intelligent persons parties actively participate to dominate over
the components strategies and make attempts to win over.

Here each participant has a set of finite or infinite choice of actions available to him and
anyone of these choices can be used freely at a given time

3.Strategy :

Strategy of a player is determined rule by which a player decide his course of action
duringthe game. That is a strategy for a given player is a set of rules or programmes that
specify which of the available courses of section, he should select at each play

4 .Player : Each participant in the game is called player

5.Pure strategy : A pure strategy is a decision in advance of all plays to choose a


particularpre determined course of action.

6.Mixed strategy : A player is said to adopt mixed strategy when he does not adopt a
single strategy all the time but would play different strategies each a certain time. The
player would choose a particular course of action on the basis of probability.

7. Pay off matrix

The outcome of a game in the form of gains or losses to the completing player for
choosing different courses of action is known as pay off. In the game, the gain and losses
resulting from different moves and counter moves when represented in the form of a
matrix is known as pay off matrix

170
B1 B2

Pay off matrix A1

2 −3
r2 l m
0 1

Here A is the maximizing player and B is the minimizing player. Each element in the
matrix is the gain for A when he chooses a course of action against which B chooses
another course of action

Here each rows of the pay offmatrixindicates gain to A for his particular strategy. A is
called maximizing player and B is minimizing player.

Value of game

Value of game is the maximum guaranteed gain to the maximizing player. (A) if both the
player use the best strategies. It is the expected pay off of a play when all the player of the
game follow there optimum strategies.

A game is said to be fair, if the value of game is zero.

Characteristics of competitive game (Scope of game theory)

1. There exists finite number of competitors called players


2. Each player has a list of finite number of possible courses of action
3. Each of the player chooses a single courses of action from the list of courses available
to him
4. Each play is associated with an outcome known as pay off. It determines a set of
payment
5. Possible gain or loss of each player depends not only on the choice made by him but
also choice made by his opponent

Different types of game

1. Two person and n person games

Game involving two players is called two persons game and n person is called n person
game

2.Zero Sum &Non Zero Sum Game

A zero sum game is one in which the sum of payments to all the competitors is zero fo
every possible outcome of the game. Here the sum of the points won equals the sum of
points lost.

A non zero sum game in which the sum of the pay offs from any play of the game may
be either positive or negative but not zero.

171
3.Finite and Infinite game

A game is said to be finite game, if it has a finite number of moves or plays.

A game without any limit on the number of moves or play is referred as infinite game.

Two persons zero sum game (Rectangular game)

A game with only two players in which the gain of one player is equal to the loss of other
player is called a two person sum games represented by pay off matrix in rectangular
form

Assumption of two persons zero game

1. There are two players having opposite interest


2. No. of strategies available to each player is finite resulting in pay off
3. Amount won by one player is exactly equal to the amount lost by the other.

Limitations of game theory

1. A player may have infinite number of strategies. But we assume that there are
onlyfinitenumber of strategies.
2. Game theory ignores the presence of risk and uncertainly
3. The assumption that the gain of one person is the loss of opponent need not be true in
all situations
4. The assumption that the two persons involved in the game have equal intelligence
may not be true always.

Maximinprinciple

Here the maximizing player (A) lists worst possible pay off of all his potential strategies
and chooses the strategies which corresponds to the best. This is called maximum
principle .

Minimax principle

In this case, the minimizing player (B) lists his maximum losses from each strategy and
selects the strategy which corresponds to the least. This is called Minimaxprinciple.

Saddle point

A saddle point of a pay off matrix is the position in pay off matrix where the maximin
coincides with minimax. Pay off at the saddle point is the value of game

Example

172
Player B

3 2 4
Player A s−2 1 −3t
0 −2 3
Maximin = Max of row min = 2
Minimax – Minimum of column max = 2
Maximin = Minimax = 2 (A1, B2)
Saddle point is (A1, B2)
Value of game = 2

Solve the game whose pay off matrix is given as

Problem 1

Solve the game whose Pay off matrix is given below

Player B

−2 15 −2
Player A s−5 −6 −4t
−5 20 −8
Value of game is -2 (A1, B1) (A1, B3)
Best strategy for player A is I
Best strategy of player B is I or III
Problem 2

Solve the game whose pay off matrix is given by

15 2 3
Player A s 6 5 7t
−7 4 0
Row min 2 , 5, -7
Column max = 15,5,17
Max min= Minimax = 5 at (A2 B2)
Saddle point at (A2, B2)
Value of game = 5

Hence the solution to the game is

1. Optimum strategy for player is A2


2. Optimum strategy for player B is B2

173
Problem 3

Solve the game whose pay off matrix

9 3 1 8 0
6 5 4 6 7
u v
2 4 3 3 8
5 6 2 2 1

Saddle point = Minimax = Max min = 4


Value of game = 4
Strategy of player (A2, B3)
Strategy of player A is A2
Strategy of player B is B3
Problem 4

A company management and labout union are negotiating a settlement. Each has the
strategies

1. Aggressive approach
2. Logical approach
3. Legalistic approach
4. Conciliatory approach

The cost to the company for every paise of strategy choices are given below.

40 30 24 70
50 20 16 20
u v
80 4 20 10
−10 8 22 0
Arrive at the decision what is the value of game
Maximin=Minimax = 24 at (A1, B3)
Saddle point (A1, B3)
Union strategy = Aggressive approach
Companysstrategy Legalistic approach
Value of game = 24
Solution of mixed strategy problem
When there is no saddle point for a game problem, the minimax = maximin principle
cannot be applied to solve that problem. The strategies thus used to solve the problem is
called mixed strategy

Solution to mixed strategy problems can be solved by the following methods

1. Probability method
174
2. Linear programming method
3. Graphical method
Probability Method
This method is applied when there is no saddle and the pay off matrix has two rows and
two columns only. The player may adopt mixed strategies with certain probabilities.
Here the problem is to determine the problem of different strategies of both players
expected value of the game

$ '
Consider the following pay off matrix

l m
w x
Let p be the probability of A using strategy A1, probability for A using strategy A2 is 1-p
Let q be the probability of B using the strategy .probability of B using strategy B2 is (1-q)
LX yO X y X O
Value of game V = 0L*X4 q = 0L*X4 p= 0L*X4
0y O4 0y*O4 0y*O4

Problem 5

Solve the following game

3 5
l m
4 1

There is no saddle point for this point since maximum and minimax

A1 = p A2 = 1-p

B1 = q B2 = 1-q

X O
P = 0L*X4 =0
0y*O4 * 4 0 * 4
=3/5

1-p =

X y
q = 0L*X4 =0
0y*O4 * 4 0 * 4
=4/5

1-q =

LX yO E
Expected value of game = 0L*X4 0y*O4
= =

z |
{ {
Strategy of A = (p, 1-p) =( , )

+
{ {
Strategy of B = (q, 1-q) = ( , )

175
}
{
Value of game =

Problem 6 (Feb 2016-10 Marks )

Solve the following pay off matrix. Determine the optimal strategies and value of
game

5 1
l m
3 4

There is no saddle point since minimax and maximin are not equal

A1 = p A2 = 1-p

B1 = q B2 = 1-q

X O
p = 0L*X4 0y*O4
=0 * 4 0 * 4
= =

1-p – 1- =

=4-1/5=3/5
X y
q= 0L*X4 0y*O4

1-q=1-3/5=2/5

~$•€] `• ‚$ƒ] = 0
E
* 4 0 * 4
=

As strategy = (p, 1-p) = ( , )

Bs strategy (q, 1-q) = (3/5,2/5)

Problem 7

For the given pay off matrix, find the saddle point if it exists or not. Find the mixed
strategy for the player.

Player B

3 2
Player A l m
−1 4

There is no saddle point since maximin ≠ minimax

X O
P = 0L*X4 0y*O4
a=3, b = 2 , c= -1, d= 4

*
0 * 4 0 * 4 5
= 1-p =1-5/6 = 1/6

176
X*y
q = 0L*X4 0y*O4
= 1-q =

X O *
Value of game = 0L*X4
0y*X4
= =

Activity Question Problem 8

Solve the following pay off matrix. Determine optimum strategiesand value of game

Player B

2 8
Player A l m
10 6

Dominance theory or dominance principle

Principle of dominance states that if the strategy of a player dominates over another
strategy in all conditions, hence the latest strategy can be ignored because it will not
affect the solution in any way. This is applied in the case of pure strategy and mixed
strategy

Dominance principle is useful in reducing the size of given pay off matrix.

Rule of Dominance

1.If all the elements in the row of a pay off matrix are less than or equal to the
corresponding elements of another row, then the latter dominates and the former is
ignored. (Remove the row having less values)

2. If all the elements in a column of a pay off matrix is greater than or equal to the
corresponding elements of a column, then the latter dominates and so former is ignored
(Remove the column having max values)

3. If the linear contributing of two or more rows (columns ) dominate a row (or
column),then the latter is ignored

case I : If all the elements of a row are less than or equal to the average of the
corresponding elements of two other rows , then the former is ignored

Case II : If all the elements of a column are greater than or equal to the average of
corresponding elements of two or other columns than the former is ignored.

By using dominance property we should try to reduce the size of pay off matrix to 2 x2
matrix

Using dominance property obtain the optimum strategies for both the player determining
the value of game

177
Problem 9

Solve the following game by dominance property

Player B

1 2 4 3 4
2 5 6 7 8
Player A u v
3 6 7 8 7
4 4 2 4 3

All the elements of column IV are greater than or equal to the corresponding elements
column I so we can elements column IV. The resultant matrix

Player B

2 4 3 4
5 6 3 8
Player A u v
6 7 8 7
4 2 8 3

All the elements of row 4 are less than the corresponding elements of row 3. So IVth row
can be deleted so we have

2 4 3 4
s5 6 3 8 t
6 7 9 7

All the elements in column V are greater than the corresponding elements of Column
1,So delete column V

2 4 3
s5 6 3 t
6 7 9

All the elements of row I and II are less than the corresponding elements of row 3, so
delete both rows 1 and 2

Resultant matrix is

3 „6 7 9…

Here the column I dominates II and III. So deleting column II and III

Value of game = 6

Optimum strategy player A choose 3 player B choose 1

Problem 10 UNI Nov 2012 (20 Marks)

178
Solve the game whose pay off matrix is given
1 7 2
6 2 7
6 1 6

Each element is in row 3 is less than or equal to the corresponding elements in row 2.
Hence row 3 is deleted. Reduced matrix is

1 7 2
l m
6 2 7

Every element of column III is more than the corresponding elements of Ist column.
Hence column III is ignored.

The reduced matrix is

1 7
l m
6 2
a= 1, b= 7, c= 6, d= 2

We may solve the problem by probability method

0LX yO4 E 5
V = 0L*X4 0y*O4
=0 * 4 0E*54
= =4

X O 5
P = 0L*X4 0y*O4
= = =

1-p = 1- =

0X y4 E
Q = 0L*X4
0y*O4
= = =

1 -Q =

| z
{ {
Optimum strategy for A is ( , ,)

Optimum strategy for B is (|, |,)

Value of game = 4

Problem No 11 UNI FEB 2015(10 Marks)

Reduce the following game by dominance and find the value of game
3 2 4 0
3 4 2 4
4 2 4 0
0 4 0 8

179
Each element of row I is less than or equal to elements in row III. So row I is deleted.

3 4 2 4
Reduced matrix s4 2 4 0t
0 4 0 8

Each element of row I is less than or equal to elements in row III. So row I is deleted.

3 4 2 4
4 2 4 0
u v
0 4 0 8

Each element of Column I is greater than the elements in Coloumn s III. Hence So column I
is deleted.

4 2 4
2 4 0
u v
4 0 8

Column 1 is greater than the average of column 2&coloumn 3.Hence Column 2&3 dominates
and Column 1 is deleted

Row I is less than or equal to the average row of II and row III

4 0
Hence reduced matrix is l m
0 8
X O A
P= 0L*X4 0y*O4
= = 1-p = 1- =

X y A A
Q = 0L*X4 =0
0y*O4 *A4 0 * 4
= =

1-q = 1- =

;† ‡= + ˆ ‰ Š z| ‰
Value of game = 0;*†4
0‡*=4 | | z
= = =

|
Strategy of A is (z, z)

|
z z
Strategy of B is ( , )

Problem 12

Solve the following game by the principle of dominance

180
8 10 9 14
A s10 11 8 12t
13 12 14 13

B4 is dominated by B1..So eliminate B4

Resultant matrix

8 10 9
s10 11 8t
13 12 14

A1 is dominated by A3. So delete A1

Reduced Matrix

10 11 18
l m
13 12 14

Row A2 is dominated by A3. So delete A2

A3 „13 12 14…

Column 2 is dominated by column I and III

Hence delete column B1 & B3

Hence the value of game is 12

Optimum strategy of A is A3 & B is B12

OR

Average of B2 & B4 dominates B1 . So delete column b1

11 8 „
l m 12 14…
12 14

Value of game = 12

Activity Question 13

Apply dominance rule a solve the following game

1 7 2 −1 −2 8
2 1 0 −2
s6 2 7 t s 7 5 −1t l m
1 0 3 2
5 1 5 6 0 12

Solving the game theory by Graphical method

Graphical method can be applied only if the pay off matrix is of the order 2 x n and m x 2

181
Hence A has only two strategies and B has n strategies. Let the strategies of A be A1 & A2.
Let p and 1-p be the probabilities with which player A uses his pure strategies. Player A will
select the value of p which to maximize the minimum excepted pay off.

For this we draw lines representing the strategies of B, say B1, B2 …. Bn. The lower
boundary of these lines will give the minimum expected pay off and the highest point in this
lower boundary will give the maximum expected pay off of player A and hence the optimum
value of P. Thus we get 2 x2 matrix. From the matrix, we find p & 1-p and expected value of
game.

Solution has m x 2 matrix

Here B has only two strategies say B1 & B2. A has m strategies say A1, A2 …. Am. We
draw two vertical lines B1 & B2. Draw the lines A1, A2 …..Am by joining the respective
joints on B1&B2. The upper boundary of these lines gives the maximum of the loss and the
lowest point of this upper boundary is minimax point. The lines passing through the minimax
point represents the choice of A. This we get a 2 x 2 matrix. Solving this matrix, we get
strategies

A&B

Problem 14.

Solve the following game problem by graphical method

Player B

2 −4 6 −3 5
l m
−3 4 −4 1 0
Player A

By dominance property B5 dominate B1. Hence B5 can be eliminated.

Pay off matrix is

2 4 6 −3
l m
−3 4 −4 1
Player A

This is a 2xn matrix where n=4

182
Lower area bounded by lines B1, B2, B3 , B4 is LMNOP

N is the inter section of B1 and B4. So ignoring B2 and B3.Thus ignoring B2 and B3. Thus
the reduced matrix is obtained by avoiding B2 and B3.

2 3
l m
3 1

A3 optimal strategy for choosing A1 or A2 is given by


P=0
* 4 0 4
= 1-q =

B = strategy for choosing B1 & B4 is given by


Q=0 * 4 0 4
= 1-q =

LX yO E
V = 0L*X4 0y*O44
= 0 54
=

Solution

+ {
‹ ‹
As strategy = ( , )

+ {
Bs strategy are (‹, 0, 0, ‹ )

}

Value of game =

183
Problem 5

Use graphical method top solve the following game

Player Y

6 3 1 0 3
Player X l m
3 2 4 2 1

Draw A1 & A2 = (vertical lines)

Mark points from -3


3 to 6 on A1 & A2. Join the points on A1 & A2 to get lines B1, B2, B3,
B4, B5

Lower area bounded by lines B1, B2, B3, B4, B5 is LMNOP. The maximum point of the area
is N. N is the point of intersection of B3 & B5. So ignoring B1 B2, B4 reduced
reduced matrix.

1 3
A1 = l m
4 1
X O
X2 optimal strategy p = 0L*X4 =0 4 0
0y*O4 4
=

Strategies are given by

X y
Q = 0L*X4
0y*O4 4 0 4
= = = 1-q =

LX yO
Value of game = 0L*X4 0y*O4
= =

184
z |
Solution to game is strategy Xs ({, {)

| z
{ {
StrategyYs (0,0, , 0, )

{
Value of game =

Problem 16

Solve graphically the game, whose pay off matrix of given below.

Player B

6 7
Ž 4 5‘
• •
Player A • 1 2•
• 2 5•
Œ 7 6•

Draw two vertical lines B2 and B1. Mark the values B1 & B2. Then draw A1, A2, A2, A4 ,
A5

Upper boundary LMNO. Lowest point on the upper boundary is N. N is the point of
intersection of lines A4 & A5. The game reduces to

2 5
l m
7 6

185
X O 5 E
P= 0L*X4 0y*O4
= = 5
1-p = 5

X y 5
Q= 0L*X4
0y*O4
= = 11/16 1-q = 5/16

LX yO
V = 0L*X4 0y*O4
= = 47/16

Solution

z
/
Strategy of player A (0,0,0, , 3/16 )

Strategy of B is (11/16,5/16 )

Value of game = 47/16

Problem 17 Activity Question

Solve the following game problem

Player B

2 5
Ž−5 3 ‘
• •
• 0 −2•
•−3 0 •
Œ 1 −4•

+
Value of game =

{ }
As strategy ( |, 0 , 0, 0 |
)

z
Bs strategy (+, +)

186
187
Problem 18(UNI Feb 2015-10
2015 marks)

Use the notion of dominance to simplify the rectangle game with the following pay off. Solve
it graphically

Player K
I IIIII IV
Player
1 L
2
3
All the elements in column I is greater than column. Hence delete column I

4 6 4
4

s2 13 7t
5 17 3

All the elements in column IV is greater than column I

Player K
II B1 IV B2
Player L
188
1
2
3
Solve the problem by graphical method
4

Strategy of L is (0, , 0, , )

Strategy of k is ( , , )

N is the point of intersection of lines A2 and A4

Reduced matrix is

2 7
l m
6 2
a= 2, b= 7, c= 6, d=2

LX yO E 5 A A
V= 0L*X4 =0
0y*O4 * 4 0E*54
= = =

X O 5
P = 0L*X4 y*O4
= 0 * 4 0E*54
= =

1-P=5/9

X y E
Q = 0L*X4 0y*O4
=0 * 4 0E*54
=

1-Q=4/9

189
190
1 +1 +1 0 0 0
B CB YB Y1 Y2 Y3 S1 S2 S3 Ratio
S1 0 1 1 -1 3 1 0 0 -1
S2 0 1 3 5 -3 0 1 0 1/5

∆j
S3 0 1 6 2 -2 0 0 1 1/2
-1 -1 -1 0 0 0

1 1 1 0 0 0
B CB YB Y1 Y2 Y3 S1 S2 S3 Ratio
S1 0 6/5 8/5 0 12/5 1 1/5 0 ½
Y2 1 4/5 3/5 1 -3/5 0 1/5 0 -1/3

2 8 1
S3 0 3/5 24/5 0 4/5 0 -2/5 1 3/4

5 5 5
0 0

191
1 1 1 0 0 0
B CB YB Y1 Y2 Y3 S1 S2 S3
Y3 1 ½ 2/3 0 1 5/12 1/12 0
Y2 1 ½ 1 1 0 ¼ ¼ 0

2 2 1
S3 0 1/5 16/15 0 0 -1/3 -7/15 1

∆j
3 3 3
0 0 0

All ∆j s are positive


Y1=0 y2=1/2, y3=1/2

Z = (1 x ) + (1 x ) + (1 x0) = 1

/ * /
Value of game = =1

Strategy of player B

Q1 = y1 V = x 1 =

Q2 = y2 V = x1=½

Q3 = y3V = 0 x 1 = 0

Optimum strategy of player A

P1= x1v = x 1 =

P2 = x2V = x 1 =

P3 = x3v = 0

|
Solution to game = Asstrategy : (z, z, 0)

Bs strategy (|, |)

Value of game = 1

Problem 23 UNI SDE Dec 2019

Solve the following game Whose pay off matrix is

−5 2 0 7
As 5 6 4 8t
4 0 2 _3

192
Give the values and strategies adapted by A&B

Solution

Column Max=4

Row Min= 4

Saddle point exists

Saddle point= 4

Value of the game=4

Strategy of A&B corresponds to saddle point (A2,B3)

Problem 24

SDE UNI DEC 2019(20 Marks)

Solve the game whose pay off matrix of Player A is given by

1 7 2
s6 2 7 t
5 2 6

Check for saddle point Max Row Minimum=1, MinColoumn Max=7

Saddle point does not exist

Apply Dominance principle

All the elements of row 3 is less than or equal to the corresponding elements of 2ndrow.Hence
3rd row can be deleted

1 7 2
6 2 7

All the elements of column 3 is greater than or equal to the corresponding elements of
istcolumn .Hence 3rdcolumn can be deleted

1 7
Hence the reduced matrix= 6 2

† = | /
Optimal strategy p = 0;*†4 0‡*=4
=0
*|4 0}*/ 4
= 4/10 1-p=6/10

193
† ‡ | }
Q = 0;*†4 0‡*=4
=0 *|4 0}*/ 4
= 5/10 1-q = 5/10

;† ‡=
Value of game = 0;*†4
0‡*=4
= 2- 42/-10 = 4

Summary of game theory

1. If saddle point exists, find the strategies of A and B are those corresponding to the
saddle point
2. When the saddle point does not exist, apply dominance rule and find the reduced
matrix
3. When the reduced matrix
a. 2 x 2 matrix – Apply probability method
b. 2 x m or n x 2 matrix – Apply Graphical method
c. mx n matrix- Apply LP techniques

194
Module V

Decision Theory

Previous University Questions (Part A&Part B)

1. What is the significance of opportunity loss in decision making under risk.(Feb 2016-
5 Marks
2. Distinguish decision under uncertainty and decision under risk Feb 2015-5 Marks
3. Write short Notes on Opportunity loss-June 2014 -5 Marks
4. Explain decision making under risk and decision making under uncertainty -5 marks
Nov 2013
5. Explain the concept decision making under risk-Nov 2012-5 Marks
6. Discussion in details the different decision environment-Nov 2011 –10 Marks
7. Write short notes on a. Decision Tree Approach b .Laplace Transform

Decision theory

Decision is the process of choosing an alternative course of action, when a number of


alternatives exists

Decision theory is a term used to apply to those methods for solving decision problems in
which uncertainty plays a critical role

Components of a decision problem

1. Decision maker
2. Number of alternatives
3. Different states of nature
4. Outcome of decision

Decision maker :Decision makers is charged with responsibility of making the decision

Course of Action (acts):Acts are alternative course ofactions or strategies that are
available to the decision maker. The decision involves selection among two or more
alternative courses of action. The problem is to choose the best of these alternatives to
achieve an objective.

Event (State of nature)

Events are occurrences which affect the achievement of the objective. They are also
called the states of nature. The events constitute a mutually exclusive and exhaustive set
of outcomes which describe the possible behavior of the outcome in which decision is
made.

Outcome : When the decision makes selects a particular act under a particular state of
nature, the result obtained is called outcome.

195
Pay off : pay off can be interpreted as the outcome in quantitative form when the decision
make adopts a particular strategy under a gain or loss of each such outcome. The pay off
can also be based on cost or time.

Opportunity loss (Regret)

An opportunity loss incurred because of the failure to take the best possible decision
,Opportunity losses are calculated separately for each state of nature that might occurs.
Given the occurrence of a specific state of nature. We can determine the possible act.

For a given state of nature

Opportunity loss = pay off that Act – pay off for the best act that have been selected.

Steps involvedin decision theory approach

1. Identify and define the problem


2. Listing of all possible future events called state of nature, which can occur in the
context of decision problem such problem are not under the control of decision makes
because they areeratic
3. Identification of all courses of action which are available to decision maker. The
decision makes has control over these courses of action
4. Expressing the pay off resulting from each pair of course of action and state of nature
5. Apply an appropriate mathematical decision theory model to select the best course of
action from the given list on the basis of some criteria that results in optimal pay off

Types of Decision Making Situations

1. Decision making under certainity

2. Decision making under uncertainty

3 Decision making under risk

1. Decision making under certainty

In this case, decision makes know with certainity the consequences of every alternatives
or decision choice. The decisionmaker presumes that only one state of nature is relevant
for the purpose. He identifies this state of nature takes it for granted and presumes
complete knowledge as to its occurance

Eg. Decision to purchase NSC, or deposit it in NSS

5. Decision making under uncertainity

196
When the decision makes faces multiple states of nature, but he has no means to arrive at
probability values to the likelihood or chance of occurrence of these states of nature, the
problem is a decision problem under uncertainity

Eg., When a new product is introduced in a market. These decisions under uncertainty are
taken with even less information than decisions under risk. The following choices are
available before the decision makes in substitution of uncertainty.

1. Maximax criteria or Minimax criteria of optimism


2. Maximin criterion – criteria of pessimism
3. Laplace criterion (criterion of equally likelyhood)
4. Hurwicz alpha criterion (criteria of realism)
5. Criteria of regret –Minimax criteria
1. Criteria of optimism (Maximax or Minimin )
In thus criterion, the decision makes ensures that he should not miss the opportunity
to achieve the biggest possible profit (maximax) or lowest possible cost ( minimin).
Thus he selects the alternative (decision choice or course of action) that represents the
maximum of the maxima (or minimum of minima) pay off (consequences or
outcomes)
1. Locate the maximum (or minimum) pay off value corresponds to each alternative
course of action) then
2. Select an alternative with the best anticipated pay off value (maximum for profit
and minimum for cost ) This is also called optimistic decision criterion
Eg. Suppose for each act, there are these possible pay off corresponds to the states
pf nature as given in the following decision matrix.

State of matrix Pay off table

A1 A2 A3

S1 220 180 100

S2 160 190 180

S3 140 170 200

• • •
Maximum pay off under each decision A1, A2,A3 are respectively


respectively

Maximum of these maximum is 220, which relates to A. Consequently according to


maximax criteria the decision is to choose A1

2.Criteria of pessimism (Maximin)

In this criteria, the decision makes ensures that he would earn no less (or pay no
more) than some specified amount

197
In this the decision makes lists down the minimum outcome within every course of
action or alternative strategy and then selects the strategy with the maximum number

a. Locate the minimum pay off value in the cost or loss ( or maximization in the case
of profit) corresponding to each alternative strategy
b. Select an alternative with the best anticipated pay off value (minimum for profit
and maximum for cost or loss)

In this criteria ,the decision maker is conservative about the future and always
anticipates worst possibleout come (Minimum for profit and maximum for cost or
loss) it is called a pessimistic decision criterion. This criterion is also known as
waldian criterion
State of
A1 A2 A3
nature
S1 -80 -60 -20
S2 -30 -10 -2
S3 30 15 7
S4 75 80 25

• • •
A 5
Minimum loss incurred

Maxi Min= -20

Hence the decision makes should take a decision for A3. A 3 is the alternative to be
selected.

3. Equally likely decision (Laplace criterion)

Since the probabilities of states of nature are not known , it is assumed that all states
of nature will occur with equal probability ie each state of nature has assigned an
equal probability

The expected pay off is worked out on the basis of probabilities. The act having
maximum expected pay off is selected. Each state of nature has assumed a
probability of 1/n of occurrence where is the no of states of nature.

This criteria is also known as criteria of in sufficient reason , because except in few
cases some information oflikehood of the occurrence of the state of nature is
available.

20 12 25
Events E1s25 15 30t
30 20 22

We associate equal probability for each event is 1/3 so the expected pay off are the
average of pay offs under each set

198
A1 = [20+25+30] = 25

A2 = [12+15+20] = 15.67

A3 = [25+30+22] = 25.67

Since A3 has the highest pay off, A3 is the optimal Act

4. Criteria of realism (Hurwicz criterion)

This criterion suggests that a rational decision maker should be neither completely
optimistic nor permistic and therefore must display a mixture of both.

In this method decision makers degree of optimism is represented by α. The


coefficient of optimum α varies between 0 & 1

When α = 0, total pessimism

Α=1 , total optimism

We find D1, D2 D3 etc connected with strategies where

Di= M i α + (1-α) m1, where Mi is the maximum pay off ith strategy and mi is the
minimum pay offofith strategy

Decision maker will specify the value of α depends upon the level of optimism

Example

Acts

20 12 25
Events s25 15 30t
30 20 22

Assume α = 0.6
For A1 max pay off = 30Minimum pay off = 20
D1 = (0.6 x 30) + (1-0.6) 20 = 26
A2 max pay off = 20 minimum pay off = 12
D2 = (0.6 x 20) + (1-0.6) 12 = 16.8
A3 max pay off = 30, minimum pay off = 22
D3 = (0.6 x 30) + (1-0.6) = 26.8
Since D3 is more, use select the act A3

199
5. Criteria regret or minimax criteria

Regret table
Minimax is just opposite to maximax. Application of the minimax decisions requires
a table of losses instead of gains. These losses are the costs to be increased or
damages to be suffered for each of the alternative act and states of nature .The
minimaxrule minimizes the max possible loss for each course of action (minimax
observation of minimum of maxima)
Under each of the various act, there is a maximum loss and the act that is associated
with the minimum of various maximum loss is the act to be undertaken according to

0 4 10
minimax criteria

Example s 3 0 6t
18 14 1
• • •
A
A3 is the choice selected :

Problem 1 UNI NOV 2013- 20 Marks

The research department of consumer product division has recommended to the


marketing dept to launch a soap with three different perfumes. The marketing
manager has to decide the type of perfume under the following estimated pay off for
the various levels of sales.

Estimated level of sales (units)


Type of
20,000 10,000 2000
perfume
I 250 15 10
II 40 20 5
III 60 25 3

Estimate which type can be chosen under maximum, minimax,


maximinLaplace&Hurwicz alpha (given alpha = 0.6)_

Solution

Pay off table


Level of table I II III
20000 250 40 60
10000 15 20 25
2000 10 5 3

Maximax for each type of perfume

Type I / 250 Types II / 40, Type III / 60

200
Maximum of maximax = 250 which relates to strategy I

Hence decision is to select type I perfume

Minimax criteria

Regret tableTable 2
Level of sales I II III
20000 250-250 250-40 250-60
10000 25-15 25-20 25-25
2000 10-10 10-5 10-3

Regret tableTable 3
Level of sales I II III
20000 0 210 190
10000 10 5 0
2000 0 5 7

Maximum losses from table II

Type I /10 ,Type II /210, Type III/ 190

Minimum of this is 10 which relates to type I hence type I is preferred.

6. Maximin(from table I)

Minimum pay off

Type I/ 10, Type II /5, Type III/ 3

Maximum of these minimum is 10 which relates to I. Hence type I is preferred.

7. Laplace criteria

Pay off table


Type I Type II Type III
250 40 60
15 20 25
10 5 3
Total 275 65 88

Average 91.67 21.67 29.33

Average pay off is more for type I, So type I is preferred

5, Harwizs alpha criterion

201
Given α = 0.6 (1-α) = 0.4

Maximum pay off for type I = 250

Min pay off for Type I = 10

D1 = (0.6x 250) +(1-0.6) 10 = 154

D2 = (0.6 x 40) +(1-0.6)5 = 26

D3 = (0.6 x 60) + (1-0.6)3 = 37.2

D1 is highest . So select type I

Problem 2

A food product company is planning the introduction of a revolutionary new product


with new package to replace the existing product at a much higher pack (s1) or a
moderate change in the composition of existing product with a new packaging at a
small increase in price (S2) or a small change in composition of the existing except
the word “ New with a negligible increase in price (S3) . The three possible states of
nature of extent are

1. High increase in sales (N1)


2. No change in sales (N2)
3. Decrease in sales (N3)

The marketing dept of the company worked out the pay off in terms of yearly new
profits for each of the strategies on these events. This is represented on the following
table .
Strategies State pay off
N1 N2 N3
S1 700 300 150
S2 500 450 0
S3 300 300 300
Which strategy should be executive concerned choose on the basis of
a. Maximin extra (b) maximax criteria
c. Minimax regret criteria d. Laplace criteria
Solution
Rewriting the pay off table (with strategies in columns)
State of nature S1 S2 S3
N1 700 500 300
N2 300 450 300

202
N3 150 0 300
Maximin criteria
Minimum pay for S1, S2, S3
S1/150 , S2/0 , S3/300
Maxima of the minima = 300 which relate to S3.
Hence the executive should choose option S3
B, Maximax criteria
Maximum pay off for S1, S2, S3 are
S1/ 700, S2/500, S3/300
Maximum of the maxima = 700 which relate to S1
Hence the executive should choose option S1
c.Minimax regret criteria
Opportunity loss table
N1 S1 S2 S3
N2 700-700=0 700-500=200 700-300=400
N2 450-300=150 450-450=0 450-300=150
N3 300-150=150 300-0=300 300-300=0

Maximum opportunity loss for S1, S2, S3

S1/150, S2/300,S3/400

Minima of these maxima = 150 which relate to S1

Hence the executive should choose strategy S1, since it minimizes the maximum
strategy

d. Laplace transform

Pay off table


S1 S2 S3
700 500 300
300 450 300
150 0 300
Total 1150 950 900

Average 383.3 316.67 300

Since the average is highest for strategy1 , the executive may select strategy S1

203
Problem 3
A company has an opportunity to computerize with department. However existing
personal has job security union agreement. The cost of three alternative
programmesfor the change over depends upon the attitude of the union are estimated
below.

Attitude of union General retraining Selective retaining Hire new employee


Antagonist 940 920 900
Passive 810 800 820
Enthusiast 700 710 860

Construct the opportunity loss table

Solution
General training Selective training Hiring new employee
Antagonist 940-940=0 940-920=20 940-900=40
Passive 820-810=10 820-800=20 820-820=0
Enthusiastic 860-700=160 860-710=150 860-860=0
Max opportunity loss is S1/160, S2/150, S3/ 40
Minimax of the max opportunity loss is relating withS3 = 40
Hence hiring new employee is the strategy followed since it minimise the
maximum strategy

Decision making under risk


Decision making under risk is a probabilistic decision situation in which more than one state
of nature exists and the decision maker has sufficient information to assign probability values
to the the likely occurrence of each of these states. Knowing the probability distribution of
the states of nature, the best decision is to select that course of action which has the largest
expected pay off values.

The objective is to optimize the expected profit or minimize the opportunity loss

The following were the criterion followed for decision making under risk

1. Expected monitory value (EMV)


2. Expected opportunity loss (EOL)
3. Expected value of profit information (EVPJ)
1. Expected monetary value

Expected monetary value (EMV) for a given course of action is the weighted average pay
off which is the sum of pay off for each course of action multiplied by the probabilities
associated with each state of nature.

EMV (Cause of action Sj = ∑!" – \

m – no of possible states of nature

204
Pi – probability of occurrence state of nature

Pj – pay off associated with the state of nature Ni and course of action Sj

The decision makes then selects from the available alternatives, the act that lead to
optimum expected outcome (act with optimum EMV)

Optimal means maximization in the case of profit or income and minimization in the case
of cost, expenditure etc

Let the pay off table be shown below

30 20
l m
35 30

Let the probabilities for the states of nature S1 & S2 respectively 0.6 and 0.4

Then EMV for A1 = (30 x 0.6) + (35 x 0.4) = 32

EMV for A2 = (20 x 0.6) + (30 x 0.4) = 24

EMV for A1 is greater

Decision maker will choose strategy A1

Expected opportunityLoss (EOL)

The difference between the greater pay off and actual pay off is known as opportunity
loss .Under the criteria, the strategy which has the minimum expected opportunity loss
(EOL) is chosen.

Calculation EOL is similar to that of EMV.

Expected opportunity loss (EOL) is defined as the difference between the highest profit
(or pay off) for a state of nature and actual profit obtained for a particular course of action
taken. EOL is also called expected value of regret.

EOL is the amount of pay off that is lost by not selecting the course of action that has the
greatest payoff for the state of nature that actually occur.

When the probabilities for various states of nature are known, it is possible to calculate
the expected for each course of action

EOL = (x1+p1) + (x2 x p2) + …. Where x1, x2 are losses and p1, P2 …. Are the
probabilities of State of nature. These expected losses are known as expected
opportunity loss (EOL)

205
Consider the following example. Given below is an opportunity loss table. A1 and A2 are
strategies S1 and S2 are states of nature.
A1` A2
S1 0 10
S2 2 -5
Let the probabilities for the state be 0.6 and 0.4
EOL of A1 = (0 x 0.6) + (2x 0.4) = 0.8
EOL of A2 = (100 x 0.6)+ (-5 x 0.4) = 4
EOL for A1 is least. Therefore the strategy A1 may be chosen

Problem 4

ABCD has invented a picture cell phone. It is faced with selecting one alternative out of
the following strategies. Manufacture the cell phone take royalty from another
manufacturer, sell rights for the invention and take a lump sum amount. The probabilities
of demand being high, medium and low are 0.25, 0.44 are respectively. The profit table is
given below
Demand Profitability Manufacturer Royalty Sell rights
High 0.25 20 60 50
Medium 0.40 50 40 50
Low 0.35 -10 20 50

Which alternative should the company select?

Solution
We apply equally likely or laplace criteria for estimating alternatives
Expected pay off of different courses of action
Expected pay off for Manufacturer
E1 = (0.25 x 20 ) + (0.4x 0.50) + (0.35 x -10) = 21.5
Expected pay off for loyalty
(0.25 x 60) + (0.4 x 40) + (0.35 x 20) = 38
Expected pay off for sell rights
(0.25 x 50) + (0.4 x 50) + (0.35 x 50) = 50
Expected pay off is maximum in the case of alternatives sell rights. Hence we may
accept the alternative sell rights as an alternative the company select.

Problem 6
A management is faced ith the problem of choosing one of the products for
manufacturing. The profitability matrix after the market research for the two products as
follows.
Act State of nature
Good Fair Poor
Product A 0.75 0.15 0.10

Product B 0.60 0.30 0.10

206
The profit that the management can make for different levels of market acceptability of
the product are as follows

Good Fair Poor

Product A 0.75 0.15 0.10

Product B 0.60 0.30 0.10

The profit that the management can make for different levels of market acceptability of
the product are as follows.
Good Fair Poor
Product A 35000 15000 5000
Product B 50000 20000 30000
Calculated the expected values of choices of alternative and advise the management

Solution
Product A product B
Pay off x profitability pay off x probability
Good 35000 x 0.75 = 26250 50000 x 0.6 = 30000
Fair – 15000 x 0.15 = 2250 20000 x 0.3 = 6000
Bad 5000 x 0.1 = 500 30000 x 0.10 = -300

EMV = 29000 EMV = 35700

Since the expected pay off (EMV) for product B is greater product B should be
preferred by management
Problem 7
Given below is the regret table
Act Events of state of matrix
A1 350 0 250
A2 0 300 0
A3 450 100 100

Suppose that the probability of events in these table are P(E1) = 0.3 P(E2) = 0.45 P(W3)
= 0.25

Calculate the expected loss of each action and interpret


A1 A2 A3
E1 350 0 450
E2 0 300 100
E3 250 0 100

Calculation of expected loss


A1 A2 A3
Loss in profit Loss x profit Loss x profitability

207
350 x 0.3 = 105 0 x 0.3 = 0.4 450 x 0.3 = 135
0 x 0.45 = 0 300 x 0.45 = 135 100 x 0.45 = 45
250 x 0.25 = 62.5 0 x 0.25 = 0 100 x 0.25 = 25

EOL – 167.50 EOL = 135 EOL = 205

Least EOL is A2. Hence A2 is the optimal Act

208
209
18. A grocery stores with bakerydept is faced with the problem of how many cakes to buy
inorder to meet days demand. The grocer prefers not to sell of goods in competition with
fresh products. Left over cakes are therefore a complete loss. On the other hand, if a
customer desires a cake and all of them have sold, the disappointed customer will be else
where and sales will be lost. The grocer has therefore collected information on past sales
or a selected 100day period as shown in the table below.
Sales per day No., of days probability
25 10 0.1
26 30 0.3
27 50 0.5
28 10 1
Total 100 1.0
Construct the pay of table. What is the original number of cakes that should be bought
each day. Find and interpret EVPI. A cake costs Rs. 0.80 and sells for Rs. 1

Solution
Let A1, A2, A3, A4 stands for strategies and S1, S2, S3, S4 stands for the states of nature

Then A1,A2,A3&A4 represents the stocking 25,26,27,28 cakes and

S1, S2, S3, S4 represents the demand for 25, 26,27, 28 values.

Conditional pay off values and pay value are obtained as shown below.
A2 S1 = 2.6 x 1 – 25 x 0.8 = 6
A3S1 = 27 x 1 – 2.5 x 0.8 = 7 A1S4 = 25 x 28 x 0.8 =2.6
A1S2 = 26 x 1 - 26 x 0.8 = 7 A4S2 = 28 x 1 – 2.6 x 0.8 = 5.2
A1S2 = 26 x 1 – 26 x 0.8 = 5.2 A2S4 = 26 x 1 – 26 x 0.8 = 5.2
A3S2 = 27 x 1- 26 x 0.8 = 6.2 A4S3 = 28 x 1 – 2 7 x 0.8 = 6
A1S3 = 25 x 1 – 2.7 x 0.8 = 3.4 A3S4 = 27 x 1- 26 x 0.8 = 4.6
A3S3 = 27 x 1 – 27 x 0.8 = 5.4 A4S4 = 28 x 1- 28 x 0.8 = 5.6

Pau off table

State of nature A1(25) A2(26) A3(27) A4(28) probability


S1(25) 5 6 7 86 0.1
S2(26) 5.2 5.2 6.20 5.2 0.3
S3(27) 3.4 4.4 5.4 6.4 0.5
S4(28) 2.6 3.6 4.60 5.0 0.1

EMV 4.02 4.72 5.12 6.06

Maximum EMV is act A4 which is equal to 6.06. This according to EMV decision
criteria the store would stock 28 cakes

210
For calculating expected value with perfect information select highest pay off for each
state of nature(ie each row ) and calculate expected values
Pay off Probability Pay off x probability
8.6 0.1 0.86
6.2 0.3 1.86
6.4 0.5 3.20
5.6 0.10 0.50
6.36

Expected value of perfect information


EVPI = 6.36 – 6.06 = 0.30
In the case of cost matrix
EVPI = Min EMV – Expected value with perfect information
(Expected value with perfect information is calculated by selecting the least pay off
of each row

Problem 11

Given below the pay off matrix


Events A1 A2 A3
E1 5 -1 -17
E2 18 28 30
E3 25 30 80

What will be the optimal decision if the criteria followed is

Maximax ,minimax, Maximin , Laplace, EMV, EOI Given probabilities for various
events are 0.3 , 0 .4 &0.30 respectively

Pay off table regret table probability

5 −1 −17 0 6 22
s18 28 30 t s10 0 2 t o.3
25 30 80 55 50 0

• • •
Maximax criteria

A
Maximum pay off =
Maximum of maxima = 80 (which relates to A3) Optimal Act is A3

• • •
Minimax criteria
From regret table max loss =
Minimum of the maxima = 22 which relates to optimal act is A3.

• • •
3.Maximin criteria

E
Minimum pay off from pay off table =
Maximum of these minima = 5 which relates toA1 .optimal act is A1.
Laplace criteria

211
Average pay off of the acts A1 = (5+ 18+ 25) = 16

A2 = (-1+28+30) = 19

A3 = (-17+30+80) = 31

Highest average is 31 which relates to A3

Optimal act is A3

5.EMVcriterian

EMV for various acts (pay off x probability)

A1 = (5 x3 ) + (18 x 0.4) + (25 x 0.3) = 16.2

A2 = (-1 x 3) + (28 x 0.4) + (30 x 0.3) = 25.9

A3 = (-17 x 3) + (30 x 0.4) + (80 x 0.3) = 29.3

Highest EMV is 29.3 which relates to A3

Hence A3 is the optimal Act

6. EOL Criterian

EOL for various acts = Regret x probability

E1 = (0 x 0.3) + (10 x 0.4) + (55 x 0.3) = 20.5

E2 = (6 x 0.3) + (0 x 0.4) + (50 x 0.3) = 10.8

A3 = (22 x 0.3) + (2 x 0.4) + 0. X 0.37 = 7.4

Minimum EOL is 7.4 which relates to A3

Hence A3 is the optimal act

Problem 12
A small ink manufacturer produces a certain type ink at a total average cost of Rs. 3/ per
bottle and then sells at a price of Rs. 5 per bottle. The ink is produced over the week end
and is sold during the following week. According to the past experience the weekly
demand has never been less than 78 or greater 80 bottles in this place.
You are requested to formulate pay off

Solution

212
The different states of nature are selling 78 units, 79 units 80 units respectively are S1,
S2, S3. The different courses of action are selling 78 units 79 units and 80 units
represented as A1, A2, A3

selling price of ink = Rs. 5 per bottle

Cost price of ink = Rs. 3 per bottle

Calculation of pay off


A1S1 = 7.8 x 5 = 78 x 3 = 156
A1S2 = 78 x 5 – 79 x 3 = 153
A1S3 = 78 x 5-80 x 3 = 150
A2S1 = (79x 5) – (78 x 3) = 161
A2S2 = (79 x 5) – (79 x 3) = 158
A2S3 = (79 x 5) – (80 x3) = 155
A2S1 = (80 x 5) – (78 x 3) = 166
A2S2 = 80x 5 – 79 x 3= 163
A3S3 = 80 x 5 – 80 x 3 = 160
Pay off represented as
A1 A2 A3
S1 156 161 166
S2 153 158 163
S3 150 155 160

Problem 13

A firm manufacturers three types of products .The fixed and variable costs are given
belowFixed cost(Rs) Variable cost (Rs)
Product A 25000 12
Product B 35000 9
Product C 53000 7
The likely demand (units of products) is given below
Poor demand = 3000
Moderate demand = 7000
High demand = 110000
If the sale price of each type of product is Rs. 25 .Prepare the pay off matrix
Solution
Let D1, D2, D3 be the poor, moderate and high demand respectively
Then pay off = sales revenue – cost
The calculation of pay off ( in thousand) for each pair of alternative demand (course of
action) and types of product (states of nature) are shown below.
D1A = 3 x 25 – 25 – 3 x 12 = 144
D1B = 3 x 25 – 35 – 3 x 9 =13

213
D1C = 3 x 25 – 53 – 3x 7 = 1
D2A = 7 x 25 – 25 – 7 x 12 = 66
D3C = 11 x 25 – 53 -11 x 7 = 145
Pay off values are shown in the table
Alternative demand
Product type A D1 D2 D3
A 14 66 118
B 13 77 141
C 1 13 145

Problem 14A news paper boy has the following probability of selling of magazine

No. of copies sold Probability

10 0.10

11 0.15

12.1 0.20

13 0.25

14 0.30

Cost of copy is 30 paise and sale price is 50 paise. He cannot return unsold copies. How

many copies should he order?

Solution

We can apply either EMV criterion or EOL criterian. Let us apply EMV criterian for which

we have to calculate pay off number of copies ordered are different courses of action. The

copies

214
215
216
conditional profit table
State of nature :High demand and moderate demand and course of action expand or
modernize
State of nature Expand Moderate
High demand N1 12-8=4 6-5 =1
Moderate demand N2 7-8=-1 5-5=0

The pay off table given below can be written as follows


State of nature Probability Expand No demand
High demand N1 0.35 4 1
Moderate demand N2 0.65 -1 0
Calculation of EMV
EMV (S1) = 0.35 x 4 + 0.65 x -1 = Rs. 0.75million
EMV (S2) = 0.35 x 1+ 0.65 x 0 =Rs. 0.35 million
Thus to maximise EMV, the company must choose the course of action S1
EMV = Rs. 0.75 million

Cause of Conditional
State of nature Probability Weighted
action profit
High demand N1 0.35 S1 A 4x 0.35 = 1.40
0 x 0.65 = 0
Moderate demand N2 0.65 S2 0
EPPJ = 1.40
The optimal EMV is Rs. 0.75 million corresponding to course of action S1
EPVI = EPPI – EMV (S1)
= 1.40 – 0.75 = Rs. 0.65 million

The company could get a perfect information of demand it should consider paying
uptoRs. 0.65 million for an information

Expected value of perfect information in business helps in getting an absolute upper


bound on the amount that should be spent to get additional information on which to base a
given decision.
Opportunity Loss Table
Cause Conditional
Conditiona
State of nature Probability of opportunity loss due
l profit
action to cause of action
High demand N1 0.35 4 1 0 3
Moderate demand
0.65 -1 0 1 0
N2

EOL (S1) = 0.35 x 0 + 0.65 x 1 = Rs. 0.65 million

EOL (S2) = 0.35 x 3 + 0.65 x 0 = Rs. 1.05 million

217
The decision maker seeks to minimize the expected opportunity loss he must select a
cause of action S1 s it produces the smallest expected opportunity loss

Activity Question 16
The manager of a flower shop promises its customers deliverywithin four houses on all
flower orders. All flowersare purchased on the previous day and delivered to Packer 8
Am the next morning. The daily demand for roses as follows.

Dozen of roses 7 8 9 10

Probability 0.1 0.2 0.4 0.3


A manager purchases roses for Rs. 10/- per dozen and sells them for Rs. 30. All unsold
loses are donated to a local hospital. How many dozens of roses should packer order each
evening to maximize its profit which is the optimum expected picture
Ans : (Highest EMV = 1.68 purchase 9 dozen)

Problem 17
The following is a pay off (in rupees) table for three strategies and two states of nature
Strategy (S1) N1 N2
S1 40 60
S2 10 -20
S3 -40 150
Select a strategy using each of the following decision criteria
a.Maximax
b.Minimax regret
c. maximin
d. Minimum risk assuming qui-probable states

Bayesian rule of decision Theory


It is an approach in which decision maker selects a course of action on rational basis by
using subjective evaluation of probability based on experience. Past performance,
judgment etc.

For making use of Bayes principle in the statistical decision problem, the decision maker
has to assign probabilities to each state of nature. These probabilities represent the
strength of decision makes belief is a subjective evaluation regarding the likelihood of
occurrence of various states of nature.

After determining the probabilities, the Bayes principle must be used phase wise. The
three phases are
1. Prior analysis
2. Pre posterior analysis
3. Posterior analysis

218
A decision maker assigns probability to various events which is his subjective evaluation
of probability based on experience of past performance.
performance. When these prior probabilities
are used the procedure is known as prior analysis.

If prior analysis is reveals a high EVPI, additional information are to be obtained. Prior
probabilities maybe then, revised on the basis of these additional information
info .By
applyingbayes theorem of probability the revised probabilities arecomputed. These
probabilities are known as posterior probabilities .A further analysis of the problem
using these post probabilities give new expected pay off. The revised of the problem is
known as posterior analysis.

Preposterior analysis is done to assess the value of sample information as against the
expected of perfect information even before selecting a sample additional information.
This analysis involves of the probabilities
probabilities using Bayesr Theorem.posterior analysis
involves arriving at a decision after revision probabilities.

Decision Trees

Decision tree is one of the devices of representing a diagrammatic presentation of


sequential and multi dimension aspects of a particular decision problem for systematic
analysis and evaluation. Under the method the decision problem, alternative course of
action, state of nature and the likely outcomes of alternatives diagrammatically or
graphically depicted as they are branches and sub branches of horizontal tree.

The decision tree consists of nodes and branches. The nodes are two types, decision nodes
and chance node. Course of action or strategies originate from the decision nodes or main
branches. At the terminal of each main branch, there is a chance node. From these chance
nodes, chance events emanate in the form of subbranches

The respective pay offs and probabilities associatedwith alternative courses and chances
events areshown along with the subbranches.

At the terminal of the sub branches are shown the expected value of outcome

219
Here A1, A2, A3 are strategies E1, E2, E3 are the events and O11, O12, O22, O31 are the
outcomes.

A decision tree is highly useful to a decision maker in multi stage situation which involve
a series of the decision each dependent on the preceding one working backward from
future to present, we are able to eliminate unprofitable branches and determine optimum
decisions. The decision tree analysis allows one to understand, simply by inspection,
various assumptions and alternatives in a graphic form which is much more easier to
understand then the abstract analytical form.

The advantage of the decision tree structure is that complex managerial problems and
decisions of the chain like nature can be systematically and explicitly defined and
evaluated.

Problem 17

There are 40% chance that patient admitted to the hospital is suffering from cancer. A
doctor has to decide whether serious operations should performed or not. If the patient is
suffering from cancer and the serious operation is performed, the chance that he will
recover is 70% otherwise it is 35%. On the other hand, if the patient is not suffering from
cancer and serious operationis performed, the chance that he will recover is 20%
otherwise it is 100%. Assuming that recovering and death are the only possible
results.Construct an appropriate decisiontree . What decision should the doctor take.

Solution
Let P1, probability that patient shall recover from operation= 0.28 + 0.12 = 0.4
P2 = Probability that patient will recover with any operation= 0.14 + 0.60 = 0.74
Since P2 > p1 doctor should not take a decision for operation to the patient .

220
Problem 18

A firm owner is seriously considering of drilling a farm well. In the past, only 70% of the
wells drilled were successful at 200 feet of depth in the area. Moreover on finding no
water at 200 ft, some persons drilled it further upto 250 feet but only 20% struck water at
250 ft. The prevailing cost of drilling is Rs. 50 per foot. The farm owner hs estimated that
in case he does not get his own wells.He will have to pay Rs. 15000 over the next in pvt
term to buy water from the neighbor.The following decisions can be optimal

1. Do not drill any well


2. Drill upto 20 ft
3. If no water is found at 200 ft, drill further 250 ft

Draw an approach decision tree determine the farm owners strategy under EMV approach

221
At D2 point
Decision A. Drill upto 250 ft b. Do not drill
Event a. No water b. Water
Probabilities are 0.2 & 0.8
EMV for drill upto 250 feet
= (12500 x 0.2 ) + (27500 x 0.8) = 24,500
EMV for do not drill =25000 (from the tree)
EMV is smaller for the act drill upto 250 ft. So it is optimal act.
At D point
The decision are drill upto 200 feet and do not drill.Events are same as those of D2 point
probabilities are 0.7 and 0.3

EMV for drill upto 200 feet

(10,000 x 0.7) + (24500 x 0.3) = 14350

EMV for do not drill = 15000 from tree

Optimal decisions is drill upto 200 feet as the EMV is smaller


Therefore combining D1 & D2 optimal strategy is to drill the well upto 200 feet and if no
water is stuck, then further drill upto 250 feet

222
223
224
At point D1
Decisions are (i) Development drug
(ii) Accept proposed it
Events are (i) successful (ii) Not successful
Probabilities 0.6, 0.4
Develop new drug
EMV = Expected PV = (23000 x 0.6) = (0 x 4) = 13800
Accept proposal H
EMV = Expected PV = 12000 x 1 = 12000(Given)
Using EMV criterion, optimal decision at D1 is to develop and market new drug

Assignment questions of OR
All questions are compulsory
1. Describe the various OR software used in an industry
2. Solve the problem by simplex method
Minimise Z = 20x1+ 24x2 + 18x3
Subject to constraints
2x1 + x2 + x3 ≥ 30
X1 + x2 + x3 ≥ 20
X1+ 2x2+ x3 ≥ 20
X1, x2, x3 ≥ 0
3. A company has 3 plants P1, P2, p3 each produces 50, 100, 150 units of a similar
product. There are 5 warehouses W1, W2, W3, W4 and W5 having demands of 100,
70, 50, 40and 40 units respectively. Cost of selling oneunitfrom various plants to
ware houses differ as given by cost matrix below.

Determine the transportation schedule so that cost is minimized.


W1 W2 W3 W4 W5
P1 20 28 32 55 70 50
P2 48 36 40 44 25 100
P3 35 55 22 45 48 150
100 70 50 40 40

4. The manager of a flowershop promises its customers delivery within four houses on
all flower orders. A flowers are purchased on the previous day and delivered to
pacrker by 8 am the next morning

The daily demand for roses as follows

225
Dozen of roses 7 8 9 10

Probability 0.1 0.2 0.4 0.3

A manager purchases roses for Rs,=10/- per dozen and sells them for Rs. 30. All unsold
roses are donated to a local hospital. How many dozen of roses should pacrker order each
evening to maximize the profit. What is the optimum expected purchase.

5. In a railway marshaling yard, goods train arrive at the rate of 30 trains per day.
Assuming the interval arrival time follows an exponential distribution and severe time
(time taken to hump a train) distribution is also exponential with an average of 36
minutes. The line capacity of the yard is to admit 9 trains only (There being 10 lines
one of the which one is ear marked for shunting engine to reverse itself from the crest
of the humps to the rear of train

Calculate the following on assumption that 30 trains on an average are recieved in the
yard

a. Probability that yard is empty


b. Expected no of trains in the yard
c. Average queue length
d. Average waiting time in the system
e. Average waiting time in the queue

6. A small maintenance project consist of the following 10 jobs. Draw an arrow diagram
representing the project

Calculate a. TE, TL values of all events

b. EST, LST, EFT & LFT of all activities


c. Float of all activities

Obtain a. critical activities b. project duration

1-2 2-3 2-4 3-5 3-6 4-6 4-7 5-8 6-8 7-8
4 6 10 8 2 12 4 15 14 18

226
Module V

OR SOFTWARES
Operations research uses various optimization algorithms to help make decisions related
to highly complex problem

Linear programming (LP) and mixed integrated programming (MIP) are often used to
solve these high complex decision making problem

Linear programming with Bounds NCSS statistical soft ware

LP maximizes (or minimize) a linear objective function subject to one or more constraint.
The technique finds broad use in OR and occasionally of use in statistically work

NCSS solves a particular linear programme using a revised dual simplex method
available in the extreme optimization mathematical subroutine package

This technique requires a special data format which will be discussed under the
specification tab

LPI data sheet


Type Logic RHS X1 X2 X3 X4

1 .Open the LPI data sheet


From the file menu of the NCSS data window select
Open example data
Click on the file LP, LNCSS
Click open
2. Open the linear programming with Bounds Window

Using the analysis menu or the procedure navigator, find and select the linear
programming with bounds procedure.

On the menus, select file, then new template. This will fill the procedures with the default
temp plate

Specify the problem


• On the linear programming with bounds window, select the specification tab
• Set type of optimum to maximum
• Double click in the row type column text book. This will bring up the column
selection window

227
• Select type from the best of columns and then click ok. Type will appear in this
box
• Double click in the variable column text box. This will bring up the column
selection window
• Select X1 – X4 from the list of columns and then click ok : x1 – X4 will appear in
the box
• Double click in the labels constraints column text book . This will bring up the
column selection
• Select C label from the list of columns and then click ok. (Label will appear in this
box)
• Double click in the logic column textbook. This will bring up the column select
window)
• Select logic from the list of columns and then click ok. Logic will appear in this
box.
• Double click in the constraint bounds (RHS) columns text book. This will bring
up the various selection window.
• Select RHS from the list of column and then click Ok. RHS will appear in this
box.

Run the procedure

• From the run menu, select Run procedure.


• Alternatively, just click the green run button.

Other softwares used in OR


1. COIN OR : Computational infrastructures for operation research includes
optimization softwares, and heuristics .
2. Gnumeric : A spread sheet with built in solves GLPK and IP- solve
3. GLPK: Linear and integer programming including a parser for a subset of AMPL
4. GOBLIN : Graph object library for network programming problem includes a
C++ library for graph optimization as well as a graph browser and editor
5. Open Forecast
6. FLOPC : An algebraic modeling language implemented as a C++ class library
7. Zimpl : Language to translate the mathematical model of a problem into a linear
(or mixed) integer mathematical program expressed in LP or mps file format
8. Chequer: Routine for cheque searching

228
Theory Questions Part A&part B questions in OR

(5 marks &10 Marks)


1. What is Operations Research
2.Explain the characteristics of OR
3.Describe the functions of OR
4.Explain the various phases of OR
5.State the limitations of OR
6.Explain the role of OR in decision making
7.Explain the various techniques of OR
8.Explain the scope of OR
9. Explain the applications of OR
10. What are the models in OR? What are the advantages and properties of OR
11. Describe the types of models used in OR
12.What is linear programming ? what are the uses of LPP
13.Explain the applications of LPP in management
14.Describe the basic functions of LPP
15.State Diet Problems in LPP
16.Explain how graphically a linear programming problem can be solved
17.Write short notes on Assignment problem
18.Explain Hungarian Algorithm or reduced matrix method
19.State Unbalanced Assignment problem
20.State Travelling sales man problem and Routing problem
21.State Transportation problem and what are the uses
22.State the basic assumptions in LPP
23.Write short notes on
a. Feasible Solution
b.Basic Feasible Solution
c.Optimal solution
24.Explain the steps for solving transportation problem
25.State the various methods of finding basic feasible solution in Transportation problem
and compare them
26. Write short notes on

229
a. .North West Corner Rule
b .Least cost entry method
c .Vogels Approximation method
d .MODIs test of optimality
27.State Unbalanced Transportation problem
28Explain degeneracy in transportation problem.How can be controlled
29.State the difference between Transportation problem and Assignment problem
30.What is meant by Simplex method. Describe the steps involved in solving Simplex
method
31.Write short notes on
A .Duality in Simplex Method
B .Artificial variables
C .Big M Method
32.Explain the special types of LPP problem
33.Write short notes on Degeneracy in Simplex Method
34.Write short notes on Sensitivity Analysis
35.State the objectives of Queuing Theory
36.What are the applications of Queing Theory
37.State the role of Queuing Theory in Management
38 Explain the types of Queuing system
39.Explain the customer behavior in queing system
40.State the characteristics of Queing Model
41.Explain with examples service cost for multiple channel system
42.Write short notes on Monte Carlo Simulation
43.Explain Traffic Intensity
44.Write short note on Kendels notation for queing
45.Explain Simulation modeling
46. Define Simulation Give two applications areas where Simulation Technique is
suitable
47.What are the uses of simulation
48.What is game theory? Explain the scope of game theory
49.What is meant by game

230
50.Write short note on Pay off matrix
51.What is meant by value of game
52. Explain the different types of game
53.what are the limitations of game theory
54.State Maxi Min Principle and Minimax principle used in game theory
55.Write short notes on Saddle point
56.Explain Mixed Strategy ?Explain how the solution of mixed strategy be determined
57.State Dominance Theory of value of game
58.What is decision theory
59.What is the significance of opportunity loss in decision making under risk
60.Distinguish between decision making under uncertainty and decision under risk
61.Explain the steps involved in decision theory
62.Write short notes on Opportunity loss
63.Discuss in detaild the different decision environments
64.Write short notes on criteria of Optimism and Pessimism
65.Explain Laplace criteria in Decision making
66.ExplainHurvizCriterian
67.What is meant by regret table
68.Write short notes on EMV &EOL
69.What is meant by expected value of perfect information
70. Explain Bayesian role of decision making approach
71.Explain the objectives of Net Work Analysis
72.ExplainNetworkAnalysis.What are the uses
73.Explain the phases of Project management
74.What is meant by float? Describe the different types of float
75.What are the uses of float
76.What is critical path method(CPM) Explain the steps involved in CPM
77.Write short notes on PERT
78.Differentiate between PERT and CPM
79. Explain the steps involved in construction of PERT
80 .State the applications of PERT
81 .Write short notes on crashing of Projects

231
82. Write short notes on Resource Smoothening
83.Write short notes on Resource leveling
84.Explain the terms Optimistic, Pessimistic ,most likely time estimates in project
Management
85.Describe the different soft wares used in Operations Research
86 .Critically comment on the assumption on which PERT/CPM analysis is done for
projects
87.Write short notes on Decision Tree Approach
88.Wrie short notes on Project variance in PERT

232

You might also like